You are on page 1of 120

St. Louis Review Center, Inc.

Eneelsus Bldg. Gen. Luna St., Davao City


Tel. no. (082) 224-2515 or 222-8732
ED-TECH
POST TEST
1. Which of the following statements is correct
about the domains of educational technology?
A. Design is the production stage while
development is the planning stage.
B. Both the design and development are the
planning stage,
C. Evaluation
is
synonymous
with
implementation.
D. Utilization is the action phase.
2. Ms. Cruz was hired in a well-equipped school
but she has to start preparing her
instructional materials before classes begin.
Which of the following is a systematic process
in preparing her materials?
A. design utilization evaluation development
B. design development utilization
evaluation
C. development design utilization
evaluation
D. development utilization evaluation
design
3. Ms. Briones is planning to integrate
technology in her Mathematics class. Which of
the following would be her second step?
A. set the objectives
B. analyze the learners
C. utilize the materials with showmanship
D. evaluate the performance of the students
4. Which of the following should Ms. Gomez
primarily consider in determining her teaching
and
learning
objectives
and
use
of
instructional media?
A. the learner
C. the instructional activity
B. the teacher D. the instructional strategy
5. Which is the best reason why teachers state
the objectives before using instructional
media?
A. To be able to practice how to operate the
equipment.
B. To determine which media to use best.
C. To prepare the materials beforehand.
D. To secure available materials.
6. Ms. Villegas is thinking of an educational
technology that can relay information clearly
to her class. Which principle will guide her in
the selection of the material?
A. interest
C. cost effectiveness
B. meaningfulness
D.
communication
effectiveness
7. Mrs. Zinampan presented real samples of
rocks when she discussed the different forms
of rocks. What principle in the selection of
instructional material did she apply?
A. interest
C. cost effective
B. B authenticity
D. responsiveness
8. Which of the following is a limitation of
conventional technologies in teaching and
learning?
A. They pose problems on storage..
B. They are less abstract and more concrete.
C. They are readily available in the
environment, around school, and in the
home.
D. They
provide
hands-on
learning
experiences and emphasize real-world
E. applications
9. which of the following is not a contribution of
technology to the learning process? .
A. The quality of learning can be improved
B. The delivery of instructions can be more
interesting

C. The role of the teacher can be changed


into a demonstrator.
D. The method of teaching and learning
becomes more interactive
10. In what way can instructional aids foster
learning?
A. Reinforce learning
B. Entertain students
C. Take the place of the teacher
D. Holds students in the classroom
11. With the pervasiveness of technologies
nowadays, a learner-centered instruction can
be promoted. Which of the following
statements
support
this
approach
to
teaching?
I. It focuses on transformation of facts.
II. It supports the use of lecture and drill
methods.
III. It gives emphasis on collaboration and
authentic assessment.
IV. Students work on tasks determined and
controlled by the teacher.
A. I and II only C. II and IV only
B. I and III only D. III and IV only
12. Prof. Villamins students use cooperative
learning, inquiry based and project-based
learning approaches in creating their digital
unit plans. What can be developed among the
learners through these approaches?
A. repetition and active learning
B. repetition & information delivery
C. information processing and active learning
D. construction of knowledge and information
exchange
13. Which of these technologies are arranged
from the most symbolic to multisensory?
A. real objects, print, audio-visual materials,
and visual materials
B. visual materials, audio visual materials,
print and computers
C. visual
materials,
print,
audio-visual
materials and realia
D. print,
audio,
visual
materials,
and
computers
14. Which group of technologies has the highest
degree of abstraction?
A. book, imaginative literature, programmed
instruction
B. digital video, film, versatile compact disc
C. video, pictures and television
D. realia and computer
15. Mrs. Soriano, a Grade V teacher prefers to use
textbooks than other instructional materials.
What could be her reason for using it?
A. Textbooks can be easily duplicated.
B. Textbooks quickly become updated.
C. Textbooks address the needs of diverse
students.
D. Textbooks contain most of the materials
they need to learn in the course.
16. It is impractical to bring real objects to the
classroom so Ms. Simangan constructed a
threedimensional visual instead. Which of the
following did she construct?
A. cartoon
C. graphic
B. chart
D. model
17. If a teacher wants to teach her pupils the skill
to organize and integrate related concepts,
which of the following is the most appropriate
graphic organizer to use?
A. timeline
C. venn diagram
B. fishbone
D. semantic webbing
18. Which graphic organizer is used to show how
a series of events interact to produce a set of
results again and again?
A. Series of events chart
C. cycle
B. Web
D. timeline
19. Which instructional aid requires pupils to
verbalize?
A. graphic
C. . model

B. diorama
D. . video
20. Which of the following is inappropriate in
using printed visuals such as charts, graphs,
and drawings?
A. Provide written or verbal cues to highlight
important aspects of visuals.
B. Allow the students to pass the materials
from one person to another.
C. Use materials that everyone can see.
D. Present the material one at time.
21. Under what category will a globe as an
instructional material fall?
A. Realia
C. solid model
B. mock up
D. cutaway model
22. Prof. Agustin would like to provide hands-on
experience on the expansion and contraction
of matter. Which of the following materials
would be the best to use?
A. models
C. realias
B. pictures
D. slides
23. Ms. Sarah finds the chalkboard an effective
instructional material up to present. However,
just like any other materials, it also has its
limitations. Which one is it?
A. It allows spontaneity, speed and change.
B. Absent students cannot keep up with their
assignments.
C. It is valuable for emphasizing the major
points of the lesson.
D. It can be used for displaying pictures and
important clippings.
24. With which learning style group are
manipulatives MOST effective?
A. Master style group
B. Interpersonal style group
C. Understanding style group
D. Self- expressive style group
25. Which does a pupil use when s/he sings a
concept to a familiar tune in order to help
himself commit the concept to memory?
A. rap
C. pop
B. jingle
D. lullaby
26. Prof. Arcilla would like to use audiocassette
tape in teaching a lesson in English. In which
activity is audiocassette tape very effective in
the teaching-learning process?
A. in developing listening skills
B. in teaching creative writing
C. in composing poems
D. in building concepts
27. Romalyn is going to discuss about The ADDIE
Model to a big class. She is planning to use a
technology by which parts of her presentation
could be partly hidden to make it more
exciting and interesting. What do you think
shall she use?
A. model
C. transparency
B. realia
D. video
28. Marife wants to make a presentation material
wherein more additional transparent sheets
with information can be placed over a base
transparency. Which one should she make?
A. cut-out
C. silhouette
B. puppet
D. overlay
29. Which one is used with 2D and 3D materials?
A. Opaque projector
C. digital projector
B. overhead projector D. slide projector
30. After watching the film, Muro Ami, the
students of Mrs. Tamaray are expected to
show a demonstrative proof of what they have
learned. How is the technology used in
thissituation?
A. entertainment
C.
instructional
B. informational
D. entertainment and
informational
31. Self made charts and illustrations serve as
universal aid for bringing fascinating and
exciting experiences in the classroom. To tap

the optimum potentials of these materials,


which of the following should be avoided?
A. Giving due consideration to lettering.
B. Presenting materials with accurate facts.
C. Giving more importance to austerity over
legibility.
D. Focusing on the main idea of the lesson
presented.
32. Kamyl used overhead transparencies when
she presented her assigned topic to class.
What type of educational technology are
transparencies?
A. printed material
C.
projected
material
B. graphic material
D.
non-projected
material
33. Which instructional material/s is/are MOST fit
in contextualized learning?
A. TV
C. pictures
B. Slides
D. field trip
34. Ms. Villanueva wants to teach the students
the performance of a certain skill such as
dancing. Which technology would be the most
appropriate and convenient to use?
A. film
C. television
B. video
D. printed material
35. Slides are miniature transparencies. They can
be created with simple cameras and simple
equipment. They display color in a realistic
manner. However, they also have some
limitations. Which one is it?
A. They can be easily updated and revised.
B. They can be adapted to group or to
individual use.
C. They can get out of sequence if handled
individually.
D. They can be combined with taped
narration for greater effectiveness.
36. Mrs. Santos used a film clip in teaching
science concepts to her Grade Six class.
However, she found out that it was
inefficiently used in the classroom. When is a
technology considered inefficient?
A. When it makes viewing more interesting.
B. When it increases the time to master the
lesson.
C. When it helps attain the objectives of the
lesson.
D. When it enhances understanding of new
lesson.
37. Prof. Manantans lesson in EPP is about
Pagtatanim ng halaman to her students.
How can she make her lesson more
interesting and meaningful?
A. Have a viewing activity about the lesson.
B. Have them read their EPP book.
C. Give them a collaborative work.
D. Let them listen to a gardener.
38. Prof. Delos Santos would like her students to
give more accurate observations about plants
in the environment. Which technique would
help her attain her objective?
A. Bring them to the garden.
B. Bring actual plants to class.
C. Show colorful pictures to the class.
D. Let the class read books about the topic.
39. Which of the following should be avoided in
presenting visuals?
A. Show visuals with an element of suspense.
B. Shut off the overhead projector when
explaining lengthily.
C. Present all the materials simultaneously to
hold the learners interest.
D. Erase any writing on the chalkboard or
whiteboard when you no longer need it.
40. After listing down the advantages and
disadvantages of computers, Mrs. Muoz
decided to purchase a computer for her class.

Which do you think is the last consideration in


purchasing the equipment?
A. Computers can make her more efficient.
B. Computers
can
be
a
form
of
entertainment.
C. Computers can enhance teaching and
learning.
D. Computers can be used for interactive
presentations.
41. Marnel prepares his school research works
using computer to submit his requirements on
time. Does the computer make him productive
and efficient? Why?
I. Yes, because it can generate its own data.
II. Yes, because it can make ones work easier.
III. Yes, because it can perform tasks fast and
accurately.
A. I and II
C. II and III
B. I and III
D. I, II and III
42. Prof. Aguinaldo would like to integrate
technology in writing a friendly letter. How
can he do it effectively?
A. Let the pupils surf a friendly letter from
the Internet.
B. Have the pupils write a friendly letter and
send it through an email.
C. Have the pupils forward a downloaded
friendly letter to others via email.
D. Let the pupils write a friendly letter using
word processing and have it critiqued by
E. their peers.
43. Which of the following is known for its
strength of giving immediate feedback?
A. video
C. digital encyclopedia
B. story book
D.
computer-assisted
instruction
44. Which of the following computer-based
instructional material can be used to learn
new concepts?
A. games
C. simulation
B. tutorial
D. drill and practice
45. 45. Prof. Natividad would like to create a
presentation material for his lesson on the
types of computer-assisted Instruction. Which
tool should he use?
A. communicative tool C. productivity tool
B. Informative tool
D.
situating
tool
46. Prof. De Guzman uses an online learning
approach by which content provides links to
information at other locations and serves as a
focal point for a distance education
experience. Which of the following does he
use?
A. computer-aided instruction
B. web-based instruction
C. self-paced program
D. teleconferencing
47. Mr. Villena searches for related literature by
accessing several databases in the library
computer that is connected with other
computers that have databases. How is this
termed?
A. CD ROM search
C. mechanical
search
B. computer search
D.
online
search
48. Which pair of tools provide synchronous
communication?
A. chatroom and email
B. email and bulletin board
C. video conferencing and blogs
D. instant messaging and chatroom
49. Should Mrs. Reyes allow her pupils to surf the
Internet in creating a group newsletter during
her English class? Why?
A. No,
because
pupils
may
just
be
exchanging messages via email.
B. No, because the pupils might open
undesirable websites.

C. Yes, to allow the pupils to chat with their


friends.
D. Yes, as long as it is used effectively.
50. Which of the following should you ask yourself
in evaluating the content of an instructional
material?
A. Do the materials reinforce learning
effectively?
B. Are the materials of high technical quality?
C. Does the content match the curriculum?
D. Is it appropriate for the students?
51. Which of the following statements does NOT
describe educational technology?
i. It includes hardware and software.
ii. It refers to the efficiency of teachers in
using computers
iii. It is the development, application, and
evaluation of systems, techniques and aids to
improve human learning.
A. i only
C. Both ii and iii
B. ii only
D. Both i and iii
52. What should Mr. Asuncion determine first in
the selection of media in teaching?
A. needs of the students C. technique to be
used
B. availability of the media D. objectives of
the lesson
53. Which is the most important reason why
teachers preview materials to be used in
class?
A. To gain confidence in using them.
B. To encourage viewers to be more focused.
C. To avoid potential problems that might
occur while materials are in use.
D. To ensure appropriateness of the materials
with the objectives and target audience.
54. After Ms. Raca planned her lesson in English,
she found out that the materials at hand do
not match her objectives. Which is the best
thing that she can do?
A. Modify the available materials.
B. Teach the lesson the following day.
C. Change the objectives to match with the
available materials.
D. Carry out the lesson as planned and use
the materials at hand.
55. Prof. Balagtas used worksheets, manipulatives
and models in teaching math to help her
students understand the lesson and love the
subject. What did she bear in mind when she
used these materials?
A. appropriateness
C. breadth
B. balance
D. variety
56. Ms. Torres always makes sure that text,
animation and color do not confuse students
in her presentation materials. Which principle
is applied?
A. simplicity
C. responsiveness
B. variety
D.
cost
effectiveness
57. Mrs. Reyes, a librarian, informed the students
as well as the teachers that several software
are available for classroom instruction and
individual learning. Which material is she
referring to?
A. Computers
C. Television set
B. CD-ROM
D. VCD and
DVD players
58. Susans mother tongue is a vernacular. Which
of the following materials would be the most
efficient and effective material to learn a
second language?
A. interactive multimedia
B. pictures and print materials
C. audio compact discs and radio
D. printed materials and real objects
59. Computer
can
be
a
good
tool
for
individualized instruction. Which of the
following aspects can be a deterrent for its full
utilization in the classrooms?

A. economic
C. social
B. physical
D. technical
60. With the increasing use of educational
technology inside the classroom, what roles
are expected of the teacher?
A. facilitator
C. knowledge giver
B. demonstrator
D.
source
of
information
61. Which of the following technologies are
properly classified?
A. computers, compact discs, film, television
B. imaginative literature, book, programmed
instruction
C. versatile compact disc, printed material,
diagram, sketches
D. digital video, phonograph, compact discs,
radio, audio tape
62. Which of the following technologies are
arranged from the most concrete to the most
abstract?
A. motion pictures, verbal symbols, visual
symbols, radio, realias
B. realias, visual symbols, television, motion
pictures, still pictures
C. realias, motion pictures, still pictures,
visual symbols, verbal symbols
D. verbal symbols, still pictures, visual
symbols, models, motion pictures
63. Which is the best way to present instructional
materials?
A. concrete
->semi-concrete->
abstract>semi-abstract
B. semi-concrete-> concrete -> abstract->
semi-abstract
C. abstract->semi-abstract-> semi-concrete> concrete
D. concrete ->semi-concrete-> semi-abstract
-> abstract
64. Which of the following technologies provide
iconic experiences?
A. videos and computer
B. books and periodicals
C. audio and audio materials
D. printed and verbal symbols
65. How can Prof. Ubia best promote the use of
multimedia in teaching Science to her
coteachers?
A. Sell multimedia at low cost.
B. Demonstrate its use to them.
C. Explain the literature supporting its use.
D. Convince the principal to require the use
of technology.

66. There are countless things in the environment


that you and your students can use to learn
from such as trees, globes, pebbles, blocks
etc. These real objects and models are really
effective if they are utilized properly. Which of
the following is incorrect about the use of real
objects and models?
A. Familiarize yourself with the object or
model.
B. Allow passing of a single object around the
class.
C. Make sure that objects/models are large
enough to be seen by the whole class.
D. Encourage students participation through
questioning and having students
E. decide the next step.
67. Aaron constructed a three dimensional
material to simulate the circulation of blood.
Which of the following did he construct?
A. A solid model
C. mock-up model
B. cutaway model
D.
crosssectional model

68. Which is a two-dimensional representation of


the earths geographic and/or political
features?
A. globe
C. mock-up
B. map
D. model
69. You asked your students to illustrate what
they have understood from what they have
read. Which of the following non-projected
visuals are you referring to?
A. printed visuals
C. models
B. graphics
D. realias
70. Which software should Dr. Balagtas to
manipulate numerical data in the computer?
A. Spreadsheet
C. word processing
B. desktop publishing D. multimedia
71. Prof. Silva uses projected visuals such as OHP
in presenting her lesson. What could be her
main reason in using such an educational
technology?
A. The materials are readily available.
B. Most visuals can be obtained at no cost.
C. It is more abstract than any other visuals.
D. She can easily prepare her own
transparencies in advance.
72. Ms. Pacheco showed a segment of matter in
sine skwela to her pupils without a follow-up
activity. Thus, the pupils got low in the test.
What does this imply?
A. TV makes viewing enjoyable.
B. TV promotes mastery of the lesson.
C. TV induces alienation on the part of the
learners.
D. TV is effective when learners attain the
lesson objectives.
73. Which activity is closest to the real thing?
A. hear
C. watch a demonstration
B. view images D. perform in a presentation
74. Your department would like to purchase a
computer set as your project. Which of the
following advantages of computer will be your
last consideration in purchasing it?
A. It can enhance the teaching and learning
process.
B. It can be used for interactive presentation.
C. It can be used for research activity
D. It can be used for entertainment.
75. Prof. Orencia will have a digitized presentation
to pre-service teachers. Which of the following
will make her presentation appealing and
effective?
A. Observe maximum use of animations and
graphics together.
B. Apply as many computer effects per slide
as possible.
C. Reinforce textual information with graphic
organizers.
D. Use as many color as possible.
76. Why are computers increasingly becoming
pervasive in schools nowadays?
A. Schools advocate the use of computers.
B. They increase efficiency and productivity.
C. Anybody can operate computers without
formal training.
D. Students have access to computers in
school and at home.
77. There are several reasons why teachers are
reluctant in using electronic media in the
teaching-learning process. Which is the most
common reason?
A. The difficulty in integrating them in the
curriculum.
B. The limited exposure of teachers to new
equipment.
C. Their incompatibility to diverse needs of
the learners.
D. The
excessive
availability
of
local
technology in the community.
78. With the number of senses to be stimulated
as criterion, which one should be first in
thelist?

A. multi sensory aid


C. visual aid
B. audio-visual aid
D. audio aid
79. Which of the following is considered in terms
of technical quality of a material?
A. stereotyping
C. color and size of
text
B. vocabulary level
D. students
achievement
80. Which statement is true about the opaque
projector and overhead projector?
A. An opaque projector allows more flexibility
than an overhead projector.
B. An overhead projector allows more
flexibility than an opaque projector.
C. Opaque and overhead projectors can
instantaneously project 3D visuals well.
D. The series of still visuals in an opaque
projector are arranged in a fixed pattern
but not in an overhead projector.
81. A grade II teacher wanted to show the parts of
a seed by using a large, wooden seed visual
aid with detachable cotyledons and tiny seed.
Under what classification does wooden
structure fall?
A. assembly model
C. realia
B. cutaway model
D. solid model
82. Which term refers to a model which is
constructed so as to emphasize a particular
part or function?
A. audio recording
C. mock-up
B. simulation
D, realia
83. Which is the best use of computers to
students like you?
A. They are used for chatting and surfing the
net.
B. They
are
used
for
research
and
collaboration.
C. They are used for playing online games.
D. They are used for watching movies.
84. Which
statement
makes
technology
ineffective in student learning?
A. It develops higher thinking skills.
B. It prepares students for the workforce.
C. It enhances students collaborative skills.
D. It decreases achievement in content
learning.
85. You plan to use instructional materials to a big
class-size. Which of these will you not use?
A. pictures
C. 27-inch television
B. projection device D. computer with LCD
projector
86. Computers can be classified according to the
roles they play namely communicative tool,
informative tool, and constructive tool. What
is the other role of computes in the options
below?
A. instructional tool
C. utility tool
B. situating tool
D, application tool
87. Which of the following categories of CAI will
you use in your class if your objective is to
increase proficiency in a newly learned skill or
refresh an existing one?
A. tutorial
C. simulation
B. drill and practice .
D. Instructional game
88. Which of the following is an ineffective use of
presentation software?
A. Darken the room
B. Use appropriate pacing
C. Read directly from the slides.
D. Allow interaction with the learner.
89. Which of the following is NOT an example of
communicative tool?
A. multimedia encyclopedia
B. teleconferencing
C. electronic mail
D. chat
90. Which is a characteristic of the teaching
machines of B. F. Skinner?
A. It does not need any feedback.
B. It requires teachers assistance.

C. It is meant for a collaborative work.


D. It allows a student to learn at his/her own
pace.
91. Why is one-way delivery of information a
misuse of communication tools?
A. because the teacher expects the student
to study more
B. because it requires activities that focus on
thinking than responding
C. because it enables the users to focus more
on higher level cognitive activities
D. because this kind of practice lessens
interaction capabilities of communication
tools
92. Internet consists of thousands of connected
computer networks around the world. Which
term does NOT refer to Internet?
A. A. NET
C. Cyberspace
B. B. Online
D.
Information
Superhighway
93. Your class adviser is planning to have an
asynchronous communication with your
classmates. Which technology tools can she
use?
A. chat and blog
B. chat and instant messaging
C. blog and video conferencing
D. electronic bulletin board and email
94. In your computer subject, you allow your class
to chat as a part of your motivation before
discussing them the roles of computer. How is
chat used in this context?
A. Communicative tool C. Application tool
B. Informative tool
D.
Situating
tool
95. Your mother wanted to finish her long
dreamed course but she wanted to do it at
home during her free time. How could you
help your mother in pursuing her dream?
A. Encourage her to hire a helper so that she
can attend regularly to her class.
B. Give up your study so that your mother
can attend her classes.
C. Enroll her to the school where you
enrolled.
D. Enroll her in distance education
96. The following statements are true about
computer
conferencing.
Which
is
an
exception?
A. It refers to live student interaction with an
expert.
B. It is also known as discussion forum or
bulletin board.
C. It also refers to online class discussions,
forums or debates
D. It permits two or more individuals to
engage
in
asynchronous
text-based
dialogue.
97. Which instructional tool application will you
introduce to your class if your objective is to
help them find and use information resources
available in the internet?
A. Webquests
C. Scavenger Hunt
B. Hybrid course
D.
Distance
education
98. Maryjane is looking for an organized
instructional program in which the teacher
and learners can be physically separated.
Which of the following will she choose?
A. Distance Education
B. Uniform Resource Locator
C. Web Quests
D. Computer-Based Instruction
99. Prof. Ruscoe would like to show Rizals
museum to the students but due to financial
constraint, she couldnt bring them there.
What
should
she
do
to
make
the
teachinglearning process more realistic?
A. Conduct a virtual tour.
B. Use DVD with less resolution.

C. Show pictures of the museum to the whole


class.
D. Go to the museum and relate all
observations made.
100.
Which of the following should you avoid if
you were asked to evaluate the effectiveness
of an instructional game after using it in
teaching a lesson in high school science?
A. Present problems which are relevant to
learning objectives.
B. Allow learners to select different content
materials.
C. Provide
a
cooperative
learning
atmosphere.
D. Provide a scoring system.
***** THE END *****
POST TEST
ED-TECH
1. Which of the following statements is correct
about the domains of educational technology?
1.
2.
3
4
5
6
7
8
9
10
11
12
13
14
15
16
17
18
19
20
21
22
23
24
25
26
27
28
29
30
31
32
33
34
35
36
37
38
39
40
41
42
43
44
45
46
47
48
49
50

D
B
A
A
B
D
B
A
C
A
B
D
D
A
D
D
D
C
A
B
C
C
B
C
B
A
C
D
A
C
C
C
D
B
C
B
C
A
C
B
C
D
D
B
C
B
D
D
D
B

51.
52
53
54
55
56
57
58
59
60
61
62
63
64
65
66
67
68
69
70
71
72
73
74
75
76
77
78
79
80
81
82
83
84
85
86
87
88
89
90
91
92
93
94
95
96
97
98
99
100

B
A
D
A
D
A
B
A
A
A
B
C
D
A
B
B
C
B
B
A
D
D
D
D
C
B
B
A
C
A
A
C
B
D
A
B
B
C
A
D
D
B
D
A
D
A
C
A
A
B

FILIPINO
POST TEST
I. Panuto: Bilugan ang titik ng wastong
sagot.
1. Agad na sumigaw ang bata ____ makitang
dumating ang kanyang kapatid
a. ng
b. nang
2. Ang mga mag-aaral ay nagkasundo _____ sa
iminungkahi ng guro.
a. din
b. rin
3. Ang
bawat
tao
_____
ay
kailangang
isakatuparan ang kanyang mithiin sa buhay.
a. Daw
b. raw
4. Ang kirot ay unti-unti ______ nawawala.
a. ng
b. nang
5. Ayon kay Jose Rizal, ang mga bata _____ ang
siyang pag asa ng bansa.
a. daw
b. raw
6. Sa Sabado _____ gabi mawawalan ng
kuryente.
a. ng
b. nang
7. Hindi na nakaramdam ng gutom si Kuya mula
_____ siya ay natulog.
a. ng
b. nang
8. _____ dalang pusa ang Inay nang umuwi.
a. May
b. Mayroon
9. Maya-maya ay sisingaw _____ ang amoy ng
Patay.
a. din
b. rin
10. Ang dunong ay kailangan ng tao ngunit
kailangan _____ niya ang tulong ng Maykapal.
a. din
b. rin
11. Sino
ba
ang
sumisigaw
_____
at
nagtatakbuhan ang mga tao.
a. doon
b. roon
12. _____ tainga ang lupa, may pakpak ang balita.
a. May
b. Mayroon
13. Ang bawat tao sa mundo ___ ay dapat
magkaunawaan para sa kanilang ikabubuti.
a. Daw
b. raw
14. Ang Pilipinas ay malakas ____ tulad ng
Singapore kung karapatan ang Pag-uusapan.
a. din
b. rin
15. Unti- unti ____ humuhupa ang kanyang galit.
a. Ng
b. nang
II. Ibinigay ang kahulugan ng salitang may
salungguhit.
16. Narinig ko ang alawat ng mga bata sa silid ng
mag-asawang Maria at Jose.
a. ingay
c. sigaw
b. mahinang alingawngaw d. tawanan
17. Ang Itay ay alimbuyaw nang dumating kanina.
a. Aburido
c. Patakbo
b. Masaya
d. sumigaw
18. Si Tj ay isang anluwagi nang mapangasawa ni
Luisa.
a. Guro
c. karpintero
b. katulong
d. pulis
19. Kakarampot ang nakuha kong ulam sa mesa.
a. marami
c. malalaki
b. katiting
d. mamhahaba
20. Alumpihit ang Itay habang hinihintay ang Inay.
a. Kabang-kaba
c. Di-mapalagay
b. Siyang- siya
d.
Tuwangtuwa
21. Iyon ang kinamihasnan ng babaeng iyon sa
bundok kaya di-makaunawa sa iyo.
a. natutuhan
c.
napagaralan
b. kinagawian
d. nagustuhan
22. Ang alipustahin ang mga dukha ay di kanaisnais na pag-uugali.
a. layuan
c. talikdan

b. apihin
d. kagalitan
23. Nakita kong pakimod na sumagot ang babae
sa dalaga nang mag-usap sila.
a. Paismid
c. patawa
b. Pangiti
d. pasigaw
24. Isang indihente ang tumawag ng aking pansin
dahil sa nakakatawang ayos nito.
a. Maralita
c. mag-asawa
b. Matanda
d. paslit
25. Naging Cum Laude si Memi dahil siya ay
nagsunog ng kilay gabi-gabi.
a. nagbubunot
c. nag-aahit
b. puspos sa pag-aaral d. nag-aayuno
26. Parang balat-sibuyas ang kutis ng babaeng
ito.
a. namumula sa bilog c. mahaba at payat
b. napakaputi at malinis
d. pino at
malambot
27. Kapit-tuko sa isat-isa habang naglalakad ang
magkasintahang Heart at Echo.
a. away nang away
b. mahigpit na magkahawak-kamay
c. malayo ang agwat
d. patakbo
28. Ang langitngit ng mga bintana ay gumigising
nang lubos sa katahimikan ng silid-aralan.
a. Alatiit
c. dekorasyon
b. kulay
d. sira
29. Mataginting na tinanggap ng batang paslit
ang pangaral ng guro.
a. maingay
c. mapayapa
b. pasigaw
d. paismid
30. Ang paswit ay sa aso, ang Oo ay sa tao.
a. Palo
c. buto
b. Sipol
d. sigaw
III.
Ibinigay ang tamang sa mga tulang
bayani
31. Ang tawag sa mga tulang bayani
a. dalit
c. senakulo
b. epiko
d. duplo
32. Tinaguriang Joseng Batute ng Pilipinas
a. Jose Garcia Villa
c. Jose Corazon de
Jesus
b. Francisco Baltazar
d. Modesto de Castro
33. Mga sagisag na ginamit ni Rizal
a. Piping Dilat
c. Pudpod at Plaridel
b. Dolores Manapat
d. Dimasalang at
laong laan
34. Ang prinsipe ng makatang Tagalog
a. Modesto de Castro
b. Francisco Baltazar
c. Fernando bagong Lanta
d. Jose Garcia Villa
35. Siya ay tinaguriang Ama ng Balarila ng
Wikang Pambansa
a. Jose Villa Panganiban
c.
Severino
Reyes
b. Lope K. Santos
d.
Rafael
Palma
36. Isang dula noong panahon ng Hapon na
isinulat ni Francisco Soc Rodrigo
a. Panibugho
c. Panday Pira
b. Sa pula, Sa Puti
d. Luha ng
Buwaya
37. Ama ng Katipunan
a. Emilio Jacinto
c. Apolinario Mabini
b. Andres Bonifacio
d. Marcelo H.
del Pilar
38. Isang uri ng panitikan na nagsasaad ng
simulain ng mga bagay o tao sa daigdig
a. tula
c. alamat
b. tibag
d. maikling kwento
39. Pinakabantog at pinakamahalagang awit na
nasulat ni Francisco Baltazar
a. Senakulo
c. duplo
b. epiko
d. Florante at Laura
40. Ang kauna-unahang Pilipinong manlilimbag
a. Marcelo del Pilar
c. Jose Maria
Panganiban

b. Tomas Pinpin
d. Emilio Aguinaldo
41. Ang tawag sa ating unang alpabeto
a. Alpabetong Romano c. Kartilya
b. Alibata
d. Romanisasyon
42. Ang taong may memorya fotograpica
a. Jose Maria Panganiban
b. Jose Garcia Villa
c. Jose Corazon de Jesus
d. Jose Rizal
43. Ama ng Dulang Pilipino
a. Julian Balmaceda
c. Lope K. Santos
b. Severino Reyes
d.
Emilio
Jacinto
44. Siya ay tinaguriang Joseng Sisiw
a. Jose Villa Panganiban
c. Jose dela Cruz
b. Pedro Paterno
d. Modesto
de Castro
45. Ang kilalang epiko ng mga Muslim
a. Hudhud
c. Hinalawod
b. Darangan
d. Bantugan
46. Ang kauna-unahang aklat na nalimbag sa
pilipinas
a. Pasyon
c. Doctrina
Christiana
b. Barlaan at Josaphat d. Florante at Laura
47. Ama ng Wikang Pambansa
a. Emilio Aguinaldo
c. Aurelio Tolentino
b. Manuel L. Quezon
d. Florante at Laura
48. Ang Orator ng Pagbabago
a. Graciano Lopez Jaena
c. Urbana at
Feliza
b. Mariano Ponce
d. Jose Buhain
49. Isang dulang nagwagi ng kauna-unahang
Gatimpalang Palanca
a. Medusa
c. Urbana at Feliza
b. Tibag
d. Hulyo 4, 1946 A.D
50. Ang may-akda ng Ang Cadaquilaan ng Dios
a. Emilio Aguinaldo
c. Julian Felipe
b. Marcelo H. del Pilar d. Lopez Jaena
51. Baston ni Adan Hindi mabilang-bilang
a. buhok
c. dahon
b. ulan
d. palay
52. Ang dalawaY tatlo na,
Ang maitim ay maputi na
Ang bakod ay lagas na
a. aso
c. matandang tao
b. kalabaw
d. punong kahoy
53. Ang anak ay nakaupo na
Ang inay gumagapang pa
a. kalabasa
c. sanggol
b. saging
d. aso
54. Kung araw ay bumbong
Kung gabi ay dahon
a. saging
c.
atip
ng
bahay
b. banig
d. paying
55. Isang reyna
Nakaupo sa tasa
a. kandila
c. kasoy
b. kapa
d. santol
56. Dalawang magkaibigan,
unahan nang unahan.
a. trak
c. bibig
b. paa
d. mata
57. Dalawang bolang sinulid
Umaabot hanggang langit
a. bola
c. lobo
b. mata
d. saranggola
58. May ulong walang mukha
May katawan, walang sikmura
Namamahay nang sadya
a. pako
c. upo
b. palito ng posporo
d. talong
59. Bumili ako ng alipin
Mataas pa sa akin
a. payong
c. sombrero
b. atip
d. bahay
60. Tubig sa ining-ining
Di mahipan ng hangin

61.
62.
63.
64.
65.

a. ilog
b. balon

c. ulan
d. dagat

a. eskursiyon
b. iskursiyon

c. exkursion
d. excursion

a. scout
b. escout

c. iskawt
d. skawt

a. colisiyon
b. kolisiyum

c. koliseum
d. coliseum

a. Istadyum
b. Stadium

c. Estadyum
d. estadium

a. Matematika
c. matimatika
b. Mathematica
d. matemateka
66. Karamihan sa mga sugapa ay mula sa wasak
na tahanan.
a. malaki ang sita ng bahay
b. maliit lamang ang bahay
c. magkahiwalay ang magulang
d. walng magulang
67. Matagal na lumagay sa tahimik si Marcia. Ang
ibig sabihinay _______.
a. matagal na namatay c. hindi na nagpakita
b. nag-asawa na
d. nanganak
na
68. Alin ang salawikain sa sumusunod:
a. Nasa Diyos ang awa
Nasa Tao ang gawa
b. Di-maliparang uwak
c. May puno walang bunga
May dahon walang sanga
d. Nag-bubuhat ng sariling bangko
69. Ang bagong alpabetong Filipino ay may ______
ng letra.
a. 20
b. 24
c. 28
d. 30
70. Ito ay bahagi ng aklat na makikita sa likod. Ito
ay talaan ng lahat na mahalagang paksa
kasama ang pahina. Ang mga paksa ay
nakasulat sa paalpabeto.
a. Talatuntunan
c. Talahulugan
b. Talatinigan
d.
Talaan
ng
nilalaman
71. Isang kuwento ng ang gumagapang ay mga
hayop na kumikilos at nagsasalita na parang
tao.
a. Parabola
c. kuwento
b. pabula
d. alamat
72. Ito ay isang uri ng dula na nawawakas a
pagkamatay ng pangunahing tauhan.
a. komedya
c. melodrama
b. epiko
d. trahedya
73. Isang kuwento hango sa banal na kasulatan
na umaakay sa tao sa matuwid na landas ng
buhay. Ito ay may aral.
a. Anekdota
c. parabola
b. alamat
d. sanaysay
74. Isang tagisan ng mga talino sa pamamagitan
ng katwiran sa pamamaraang patula.
a. balagtasan
c. tula
b. talumpati
d. duplo
75. Si severino Reyes na lalong kilala sa tawag na
Lola Basyang ay higit na kilala sa larangan ng:
a. dulaan
c. pag-awit
b. pagtula
d. balagtasan
76. Ang Kumitang ay isang uri ng awiting bayan.
Ito ay may karaniwang inaawit sa:
a. paghaharana
c. paghehele
b. pakikidigma
d. pamamangka
77. Ang senakulo ay isang panrelihiyon; ito ay
naglalayon na
a. ipaala ang kapanganakan ni Hesukristo
b. ipakita ang pagkakapatiran ng mga
Kristiyano at Muslim
c. magsalarawan
ngmga
pinagdaanang
buhay at kamatayan ni Hesukristo
d. magligtas sa mga kasalanan

78. Sa akda niyang Guryon, ipinalintulad ni


Idelfonso Santos ang Guryon sa:
a. buhay ng tao c. anyo ng pagpapalipad
b. tibay ng pisi
d. hanging habagat
79. Unupo si Itim, sinulot ni Pula, heto na si Puti
na bubuga-buga. Ito ay halimbawa ng isang:
a. bugtong
c. alamat
b. salawikain
d. kuwentong bayan
80. Ang sinuman ay makabubuo ng matibay na
lubid kung pagsasamahin ang sinulid. Ang
ibig sabihin ng kasabihang ito ay:
a. Mahirap magkaisa ang mga tao.
b. Madali ang gumawa ng lubid kung may
sinulid
c. Kailangan natin ang lubid sa ating mga
Gawain.
d. Magkakaroon tayo ng lakas kung tayoy
magkakaisa.
81. Ang taong nagigipit sa patalm kumapit. Ano
ang ibig sabihin ng salawikaing ito:
a. Ang kaligtasan ng taong nagigipit ay sa
tapang ng dibdib
b. Susuungin ng tao kahit ani mang panganib
upang malunasan ang kanyang problema
c. Malapit sa panganib ang mga taong
nagigipit.
d. Huwag makiharap sa taong nagigipit
sapagkat siya ay siguradong galit
82. Alin sa sumusunod ang hindi tuluyang anyo
ng panitikan?
a. korido
c. kuwentong bayan
b. alamat
d.
maikling
kuwento
83. Tukuyin kung anong uri ng panitikan ang
Isang bayabas, pito ang butas.
a. Sawikain
c. Salawikain
b. Idyoma
d. Bugtong
84. Ano man ang tibay ng piling abaka ay wala
ring lakas kapag nag-iisa. Isinasasaad ng
salawikaing ito ang kahalagaan ng :
a. pagkakaisat pagtutulungan
b. tibay ng dibdib at lakas kahit nag-iisa
c. pagkakaroon ng lakas kahit nag-iisa
d. pagpapalakas ng loob lalot nag-iisa
85. Sa akin lipain doon nagmula
Lahat ng pagkain nitong ating bansa
Ang lahat ng tao mayaman o dukha
Silay umaasa sa pawis kot gawa.
Ano ang ipinahihiwatig ng saknong?
a. Lahat ng pagkain ay sa magsasaka
nagmumula
b. Lahat ng magsasaka ay may lupang
sinasaka
c. Lahat ng taot bagay ay galling sa lupa
d. Lahat ng umaasa sa biyayang galling sa
magsasaka.
86. Ang kaibigan ko ay isa lamang maralita.
a. mangmang
c. mabait
b. maliliit na tao
d. mahirap
87. Si Nena ay inaruga ng kanyang lola mula pa
noong siyay maulila.
a. pinabayaan
c. inalagaan
b. pinamigay
d. kinuha
88. Palasak na ang desenyong iyan.
a. pambihira
c. magastos
b. pangkaraniwan
d. wala sa
moda
89. Ang mga salbahe ay kinamuhian niya
a. kinakalinga c. kinatatakutan
b. kinukumusta d. kinasusuklaman
90. Ang mga kawal na lumabag sa utos ay
binigyan ng babala.
a. sundalo
c. kusinero
b. kaibigan
d. pulis
91. Nangangamba ka ba na hindi ka niya
pagbibigyan?
a. nasisiyahan
c. nababanas
b. natatakot
d. naiinis
92. Ang pagpunta sa Saudi Arabia ay di-gawang
biro.

a. Madali
c. mahirap
b. masayang Gawain
d. maayos
93. Si Miguel ay sumakabilang buhay na noong
Linggo.
a. nagpaalam
c.
nagpunta
sa
siyudad
b. namatay
d.
nagbayad
ng
utang
94. Bakit mukhang Biyernes Santo si Marko.
a. malungkot
c. mukhang masaya
b. lumuluha
d. tumatawa
95. Nakaririmarim ang nangyaring sakuna sa
dagat.
a. nakalulungkot
c. nakaiinis
b. nakatatakot d. nakapangingilabot
96. Ang dayuhang siyang pinakamatalik na
kaibigan ni Rizal ay
a. Austin Craig
c. Otley beyer
b. Ferdinand Blumentritt
d.
Don
Eulogio Despujl
97. Dahil sa tulong at pagmamalasakit ni
a. Dona Aurora A. Quezon
b. Tandang Sora
c. Luz B. Magsaysay
Sa kapakanan ng mga sinalanta ng sakuna,
siyay tinaguriang Ina ng Kruz na Pula.
98. Kung ano ang Urbana at felisa sa mga
tagalog ang
a. Lagda
c. Bidasari
b. Maragtas
d. Hudhud
ay siya naman sa mga Bisaya.
99. Sa mga tauhan ng Noli Me Tangere ni Rizal, si
a. Basilio
c. Capitan Tiago
b. Elias
d. Simon
Ang nagligtas kay Ibarra sa kapahamakan
100. Ang aklat ng mga tinipong tula sa Tagalog
ni Lope K. Santos ay pinamagatang
a. Damdamin
c. Tungkos ng Alaala
b. Puso at Diwa
d. Mga Dahong Ginto
***** THE END *****
WORK HARD, DREAM HARDER

filipino
1
2
3
4
5
6
7
8
9
10
11
12
13
14
15
16
17
18
19
20
21
22
23
24
25
26
27
28
29

B
B
B
B
B
A
B
A
B
A
A
A
B
A
B
B
A
C
B
C
B
B
A
A
B
D
B
A
C

51
52
53
54
55
56
57
58
59
60
61
62
63
64
65
66
67
68
69
70
71
72
73
74
75
76
77
78
79

B
C
A
B
C
B
B
B
C
B
B
C
B
A
A
C
B
A
C
A
B
D
C
A
A
B
C
A
A

30
31
32
33
34
35
36
37
38
39
40
41
42
43
44
45
46
47
48
49
50

B
A
C
D
B
B
B
B
C
D
B
A
A
B
C
B
C
B
A
D
B

80
81
82
83
84
85
86
87
88
89
90
91
92
93
94
95
96
97
98
99
100

D
B
A
D
A
D
D
C
B
D
A
B
C
B
A
D
B
A
D
D
B

HUMAN GROWTH AND DEVELOPMENT


POST-TEST April 2010
Multiple Choices:
1. The process by which certain potentials are
inherited
from
the
parents
for
his
development
a. Life
c. Heredity
b. Birth
d. Character
2. This theory states that there are 8 basic
development stages that the individual has to
pass through his life
a. Learning Theory
b. Psychoanalytic Theory
c. Psychosocial Theory
d. Cognitive Development
3. Transition age from childhood to adulthood
where rapid physical changes and sex
maturity occur resulting in changes in ways of
feelings, thinking and acting.
a. Puberty
c. Early adulthood
b. Adolescence
d. Stage V
4. Modifying an existing scheme after an
individuals interaction with the environment,
resulting in the creation of a new scheme.
a. Assimilation
c. Recognition
b. Interaction
d. Accommodation
5. Theory stating that a persons behavior can
be motivated by urges towards self
satisfaction.
a. Psychoanalytic Theory
b. Cognitive development theory
c. Psychosocial Theory
d. Moral development theory
6. The ability of a child to conceptualize the
retention and preservation of the same
quantity under various changes.
a. Recognition
c. Assimilation
b. Reversibility
d. Conservation
7. Refers to the idea that no individual are
exactly the same or alike.
a. Cognitive theory
c.
Individual
differences
b. Exclusivity theory
d.
Emotional
quotient
8. He is known as the Father of Modern I.Q. Test
a. Lewis Terman c. Laurence Kohlberg
b. Erick Erickson
d. Martin Lesley
9. Intellectual appreciative Experience is
a. base on the premise that all learning has
emotional correlates
b. obtained in the field of music, art and
literature

c. the acquisition and retention o acts and


information
d. assumes that human activities are based
on stimulus and response
10. These statements imply that children at the
early learning stage consider parents and
teachers as authorities and models.
a. Parents and teachers should always
coordinate childrens activities
b. Parents should enforce strict discipline at
home and teachers in school
c. Parents and teachers should be the role
models at all times
d. Parents and teachers should always
consult each other with regards the childs
intellectual development
11. Any change in the behavior of an individual
a. Learning
c. Change
b. Response
d. Development
12. Which of the following principles IS NOT
considered under Classical Conditioning by
Ivan Pavlov?
a. Excitation
b. Adhesive Principle
c. Stimulus Generalization
d. None of the above
13. The reinforcement of a persons responses by
presentation or removal of rewards and
punishment.
a. Operant conditioning c. Feedback Principle
b. Transfer of learning d. Discipline
14. This stimulation of action best explains the
behavior of an individual to take what he
perceives to be the shortest route to his goals.
a. Recognition
c. Response
b. Assimilation
d. Motivation
15. The process by which an individual acquires
the social and cultural heritage o the society
where he belongs.
a. Socialization
c. Integration
b. Internalization
d. Acquisition
16. Philosophy of educations main function.
a. Aid the leaner to build his own personal
philosophy
b. Definition o goals and setting of directions
from which education
c. Educations carries on a lifetime cycle
d. Provision
of
academic
background
prerequisite to learning
17. According to Froebel, kindergarten is also
known as ____________?
a. children have fun and enjoyment
b. Garden where children could grow
c. He learning Center for Life
d. Where new beginnings begin
18. Which of the following statements is given
emphasis by humanistic education?
a. The great works of man such as the
classics should be enjoyed.
b. Man
should
learn
the
different
philosophies of education
c. Build a man who is distinctly civilized,
educations and refined
d. Develop man into a thinking individual
19. A teacher who advocates the pragmatic
philosophy of education believes that
experience should follow learning, thus, she
has to?
a. require her student mastery of the lessons
b. encourage her students to memorize facts
c. equip her students with basic skills and
abilities
d. provide her student with opportunities to
apply their skills and abilities
20. How are institutions of learning encouraged to
set higher standards over and above the
minimum requirement for state recognition?
a. Scholastic achievement
b. Faculty development
c. Academic freedom

d. Voluntary accreditation
21. The period of physical, especially sexual, and
mental maturation which is characterized by
rapid somatic growth is known as
a. infancy
c. puberty
b. early childhood
d. adulthood
22. Claustrophobia is an irrational fear of
a. Darkness
c. closed space
b. strangers
d. height
23. An eye defect characterized by clear vision in
one dimension but unfocused vision on the
other is called
a. myopia
c. hyperopia
b. astigmatism
d. presbyopia
24. Which of the following statements does not
apply to adolescents?
a. they desire the approval of their peers
b. they seek dependence on their parents
c. they have a marked sex development
d. none of the above
25. As young people mature, society expects
them to develop competencies and assume
social roles in a conventional manner.
a. expectation of parents
b. influence of peers groups
c. influence of formal education
d. cultural demands
26. The founder of the theory of psychology called
psychoanalysis was
a. Lock
c. Freud
b. Hume
d. leibnitz
27. When the learner reaches a point where no
further improvement can be expected, he is in
a so-called
a. development crisis c. regression
b. learning plateau
d. depression
28. Regarding the sexual maturation o boys and
girls, teachers should bear in mind that:
a. girls mature at a late stage than boys
b. girls mature at an earlier stage than boys
c. boys and girls mature at the same time
d. there are no marked differences in heir
time of maturity
29. Rationalization is used by student who
a. always give explanation or reason for their
failures rather than own their faults
b. like to take the blame for their faults
c. bribe their elders with promises
d. substitute words for deeds
30. Which of the following is true of Abnormal
Psychology?
a. it studies the cause of personality defects
b. it measures the accomplishments of the
individual
c. it
concentrates
on
the
scholastic
performance of the individual
d. it investigates the educational background
of the individual
31. Which of the following is a continuous
variable?
a. weight
c. nationality
b. sex
d. race
32. Which of the following is true about ones IQ?
a. it remains fairly constant
b. it is highly changeable
c. it is affected by attitude
d. it is never constant
33. Transfer of training easily takes place if the
activities involved
a. Are different
b. Have identical element
c. Occur in the same place
d. Vary in difficulty
34. When the learner is well-motivated, he
performs his task
a. with indifference
c. with arrogance
b. with disinterest
d. with enthusiasm
35. A six-year-old child who has a mental age of
eight years has an IQ of

a. 120
b. 130
c. 132
d.
133
36. The ratio obtained by dividing mental age by
chronological age times 100 is called
a. derived quotient
b. deviation
c. intelligence quotient or IQ
d. intelligence ratio
37. Which of the following was written by Plato?
a. Sic et Non
c. The Republic
b. The School and Society
d. Emile
38. Who among those below asserted that
Education is for complete living
a. Dewey
c. Kant
b. Spencer
d. Froebel
39. The right of an educational institution and its
faculty to prescribe the methods/strategies of
teaching refers to:
a. building style
b. choice of curriculum
c. academic freedom
d. co and extra curricular program
40. The 1987 Constitution provides that religious
institution can be given
a. with the students consent
b. with the parent/guardian approval
c. with mayors permit
d. with the schools support
41. Public schools in the Philippines are the
contribution of which colonizer?
a. American
c. Japanese
b. British
d. Spanish
42. Hardship allowance is given to a teacher when
a. hes assigned in a depressed area
b. hes given additional teaching load
c. hes in lahar area
d. hes assigned in a hazardous area
43. The ability for quantitative learning of the
relations of facts taken from newspaper
readings, letter writing and the like is called:
a. functional literacy
c.
Knowledge
outcome
b. adjustment learning d. Social competence
44. A teacher who gives a uniform assignment to
be worked out by all learners in Arithmetic is
not observing a characteristic of a good
assignment.
Which
characteristic
is
overlooked?
a. It should be definite
b. It should be stimulating
c. It should emphasize the essential
d. It should provide for individual differences
45. If a student ask a question which the teacher
does not have a ready answer, the latter
should:
a. dismiss the question as irrelevant
b. offer a bluff
c. admit the fact that he doesnt know the
answer
d. ask volunteers to answer the question and
do research on it later.
46. The heredity traits acquired by a person in his
lifetime;
a. are transmissible to his offspring
b. reappear in his future grandparent
c. Have no influence on the offspring
d. Become recessive traits
47. When student are given a chance to settle
differences of opinion by discussion, they
develop:
a. fair play
c. irritants
b. tolerance
d. sociability
48. The schools responsibility towards teenagers
gang age is:
a. provide the gang all the freedom it needs
b. gives classroom activities to give direction
to out-of-school youth activities
c. supervise gang activities

d. set up norms of conduct or the member of


the gang
49. In an intelligence test, a 13-year old girl got a
score equivalent to that of a 15-year old. This
means:
a. that the girl must be accelerated
b. that the girl is 2-years older mentally
c. that the girl has a chronological age of 15
d. that she has a mental age of 13
50. Which statement is not necessary to achieve
the learners interest in a learning activity?
a. the activity must lead to a practical end
b. the activity must be within the ability of
the learner
c. the activity must fill a need recognized by
the learner
d. the learner must have the experience that
will furnish the background for the activity
51. He is responsible for the theory which
recognizes the importance of developing
multiple intelligence
a. Jean Piaget
c. Frederick Freobel
b. Howard Gardner
d. Sigmund Freud
52. The need to recognize and develop special
sensitivity to language, thus helping the
learners to use the right word, phrase and/ or
graph to grasp new meaning refers to
a. visual intelligence
c. feelings
sensitivity
b. linguistic intelligence
d. jargon
53. The sensitivity to tone and pitch, allowing
one to produce musical scoring is intelligence
in?
a. musical
c.
quantitative
exercises
b. verbal ability
d.
qualitative
analysis
54. Ones ability to do abstract reasoning and
manipulate symbols refers to what type of
intelligence?
a. musical
b. personality identification
c. mental ability
d. mathematical-logical
55. The ability to perceive how objects are related
in order to mentally perceive what is seen,
thus creating concrete visual images from
memory refers to?
a. visual-spatial intelligence
b. musical
c. language
d. logical reasoning
56. The capacity to analyze ones feelings and
thus be able to understand and be able to
know the motives of other peoples actions.
a. spatial
c. logical
b. personal
d. diametric
57. The type of intelligence which enables a
person to understand other persons feelings,
behavior and motivation.
a. emotional
c. social intelligence
b. spatial
d.
quantitative
and
qualitative
58. The type of intelligence which characterizes
actress, actors, mimes, dancers and people of
the Arts?
a. bodily-kinesthetic
c. research
b. scientific
d. emotions
59. An emerging thrust in determining ones
personality,
whether
pleasant
or
unwholesome, this type of personality
measurement is the wholesomeness of ones
virtues, i.e., values, relationships with other,
adjustments to varying situations, behavior an
motivations
a. emotional quotient (E.Q.)
b. intelligence quotient (I.Q.)
c. maladjustment personality

d. anticipated behavior
60. It is a measurement of personality which is
the result by dividing the mental age by the
chronological age.
a. emotional quotient (E.Q.)
b. intelligence quotient (I.Q.)
c. multiple Intelligence
d. forecasted behavior quotient
61. The teacher must be aware that both heredity
and environment represent complex factors,
exerting many specific influences on an
individuals growth. Which of the following
statements best represents the influence of
heredity and environment?
a. Heredity counts; environment is less
important.
b. If the environment is changed, heredity
becomes less important.
c. The relative influences of heredity and
environment can vary widely in an
individuals growth.
d. In the long run, both tend to cancel each
others influences
e. None of the above
62. The best possible way to measure the
influence of heredity is by:
a. keeping the environment constant.
b. Ignoring the environment
c. Studying only fraternal o normal capability
d. Studying only identical twins of normal
capability
e. Doing none of the above
63. Educators who contributed to the open
education movement includes:
a. Neill and piaget
c.
Bruner
and
Silberman
b. Kohl and kozol
d. All of the above
64. A childs social skills can be measured by:
a. direct observation and parent-teacher
conferences
b. psychological test
c. adaptive behavior scales
d. A and C above
65. A teacher uses behavioral modification
techniques in his classes. Which of the
following student behaviors would he find
most difficult to change?
a. Aggressive tendencies toward classmates
b. Poor habits in organizing work materials
c. Interrupting a speaker
d. Abandoning a project before it is finished

c. Left-right directional confusion


d. Speech aphasia
70. Students with secondary reading problems
have capacity to read, but are non-readers
because of:
a. auditory problems
b. congenital defects
c. visual-acuity impairment
d. environmental or emotional actors
71. If a teacher accepts Maslows theory on the
hierarchy of needs, he or she will probably
structure objectives to:
a. meet
both
the
physiological
and
intellectual needs of students
b. eliminate testing
c. eliminate extrinsic motivations
d. maintain a certain anxiety level for
increased competition
72. The knowledge explosion has led to crowding
more and more information into curriculum
courses. A likely result is that:
a. the textbook will no longer be the main
instructional medium in many classes
b. the child may spend more time in school
c. the teacher may have to rely more on the
se of multimedia materials
d. all of the above
73. During the learning process the teacher has
most control over:
a. the learners
b. the learning environment
c. the learning process
d. the behavior of the learners
74. Which of the following conditions does NOT
contribute to a climate psychologically suited
to learning?
a. The teacher acts like a real person.
b. The teacher makes all of the decisions
about students learning activities.
c. The teacher accepts students as they are
d. The teacher shows trust in students
decisions

66. Learning-disabled
children
most
characteristically have:
a. low IQ
b. poor socio-economic backgrounds
c. an average level of intelligence
d. minimal brain damage
67. Which of the following is true about educable
mentally retarded children?
a. Their IQ range between 50 and 70
b. They have short attention spans and
experience difficulty in generalizing
c. Their reading, writing, and arithmetic skills
cannot be improved
d. A and B above
68. Which of the following is characteristics of a
dyslexic child
a. Mirror writing
b. listlessness
c. Below-average intelligence
d. Hyperactivity
69. Primary reading retardation is presumed to be
neurologically based, related to parietal lobe
dysfunction?
a. Inability to relate sound to letter symbols
b. Inadequate
auditory
information
processing

75. William Glasser advocates the frequent use of


classroom meetings, with teacher and
students sitting in a small circle. Which one of
the following types of discussion would NOT
be appropriate in such a setting?
a. An educational-diagnostic conference on
the learning weaknesses of individual
students.
b. An open-ended meeting for the purpose of
exploring and discussing students ideas
about the curriculum
c. A
social-problem-solving
meeting
to
resolve teacher or student problems
elating to the school, the class, or any
individual member.
d. A sensitivity-training meeting for the
purpose of helping students ace their
school-related problems and learn how
their actions can affect others
76. Which of the following does NOT represent a
teachers contribution to the emotional
environment of the classroom?
a. A strident, compelling voice.
b. A sustained sense of expectation where
student achievement is concerned
c. A well-written lesson plan

d. A sense of humor in a tense situation


77. According to Jones, student commitment to
accomplishing a learning goal depends on all
of the following EXCEPT:
a. how interesting the goal is
b. how likely it seems that the goal can be
accomplished
c. what degree of challenge the goal
presents
d. whether the learner will be able to tell if
the goal has been accomplished
e. whether materials are ready assembled
for undertaking the goal
78. The
teacher
who
understands
the
adolescents need to conform will:
a. use sarcasm as a disciplinary device
b. disregard unique responses in discussion
and on examinations
c. establish a learning climate that fosters
feelings of security
d. lecture students on their weakness o
character
79. The best public relations agents for a school
are the:
a. pupils
c. PTA members
b. Teachers and pupils d. principals
80. The structured curriculum is in decided
contrast to the child-centered curriculum,
which:
a. emphasizes fundamental education
b. is changeable and is built around student
interest and needs
c. is oriented to the needs of a democratic
society.
d. Utilizes the theory of mental discipline
81. According to Bruner, teacher working with
young children should
a. Push the children to maximum cognitive
development as rapidly as possible
b. Present all information verbally so the
children will listen well
c. Present new material from the concrete to
the abstract
d. Present new information from the abstract
to the concrete
82. from the educational viewpoint, intelligence
is:
a. an abstract concept
b. a trait that can be manipulated
c. good judgment
d. a form of behavior
83. Every taxonomy of educational objectives:
a. describes increasingly difficult learning
activities
b. describes levels of goals for learner
development
c.
suggest evaluation measure for teacher
use
d. Classifies learning outcomes

84. A mathematics teacher following Gagnes


theory of learning believes that:
a. learning can take place under all
conditions
b. learning is mainly a mater of accurate
discrimination
c. learning takes place only when the
student is in a receptive state
d. learning is reinforced chiefly by classical
conditioning
85. Under which of the following conditions is a
childs IQ more likely to increase?
a. If the emotional climate in the classroom
improves
b. If the child is given a large research
project.

c. If the child enjoys problem solving and is


given ample opportunity for it
d. If A and C are true
86. Intelligence is the basis of education.
Education is the effective means for national
development, hence, a country spends a large
portion of its budget for the systematic
training of the learner to attain full
development
Why is education one major concern of
every c
country? Because
a. intelligence has many facets
b. intelligence is useful in testing
c. intelligence is a safe gauge for budgetary
allocation
d. intelligence test when carefully conducted,
can help in determining need for future
facilities for national building
87. There are no two individuals who are the
same. Individual differences, when early
recognize and provided for, enable the
teacher to provide different motivations and
approaches in guiding the learning process.
Each pupil differs physically, mentally, socially
and emotionally from other children. Unless
the teacher provides for this nature of the
learner, no amount of modern approaches in
teaching can elicit favorable results.
a. The paragraph highlights the need or
motivating learning
b. Individual differences is an important
consideration in guiding the learner
c. The above paragraph focuses on teacherpupil relationship
d. It takes about the nature of the learning
process
88. Robert Craig, et al, wrote of the phase of
steps in every learning process. These
include: 1.) the focusing of attention to the
stimulation at hand, 2.) the interplay of the
learner and the social factors that surround
him, 3.) the acquisition of a new response or
behavior he gives to the new learning and 4.)
Retention which presupposes that the new
learning is acquired.
The above paragraph emphasizes
a. the learning process
b. the steps/phase of how individuals learn
c. the manifestations of learning
d. why learning is a difficult process
89. Approaches in teaching change from time to
time depending on the traditional of
sophistication attached to the course being
taught. Some mentors believe that the tie
tested ways to teaching is effective. Other are
easily carried away to use modern approaches
in imparting new subject matter. It maybe
safe to conclude that once results are realized
in teaching, no specific method can be
considered the one-and-only method to use.
When teaching a subject area, it is safe to
a. stick to the traditional way
b. be modern and most recent
c. get results in teaching
d. to try any method as they are all theories
after all

90. In the early 1980s programmed teaching


became popular in helping teachers to
provide for individual differences in learners.
The chunks of the subject matter which are
divided into units are supposed to help the
learner master the lesson, since it is simply to
understand the frame of the lessons. No test o
mastery of the units are done because the

purpose is to provide information on certain


subject matter
Would you as a teacher use programmed
instruction if you handle a subject on
Values Education?
a. yes, definitely
b. no, not important for the subject matter
c. I dont know
d. Why not if the subject matter calls for it
91. The data/subject matter to teach are gathered
in different ways, These include historical
sources like surveys, systematic observations,
experimentation, interviews, etc. to be
reliable and valid, the data collected must be
organized, properly analyzed and interpreted.
From these processes, some conclusion or
generalization are done to reveal certain
relationships like cause & effect. Data
gathering involves:
a. tedious and serious study
b. easy does it
c. data gathered are tested and filed, then
verified before being used
d. no follow-up needed
92. Heredity and environment play important
roles in the function of human beings. DNA or
Deoxyribonucleic Acid is the biological
(heredity)
band
of
our
genes.
Our
environment includes the house, school and
the community where we live. Whether we
become successful or a failure will depend on
the interplay of both nature and nurture.
If heredity and environment affect the
individual, thus, we can conclude that
a. both actors play equal roes in ones life
b. one factor, either heredity or environment
exerts more influence than the other
c. neither factor is important
d. nurture and nature are the same
93. The first systematic philosopher to work in
the field of education was
a. Socrates
c. Plato
b. Aristotle
d. Rousseau
94. The first state in the worlds history where all
human capabilities were allowed to develop
freely
a. Rome
c. Sparta
b. Athens
d. Germany
95. They are the most practical, pragmatic people
who absorbed themselves in the management
of their state affairs
a. Spartans
c. Romans
b. Athens
d. Chinese
96. Invented the first system of writing in the
orient
a. Phoenicians
c. Greeks
b. Chinese
d. Romans
97. first to introduce the use of printing press in
the Philippines
a. Romans
c. Greeks
b. Chinese
d. Japanese
98. conducted the worlds first civil service test
a. Greeks
c. Chinese
b. Romans
d. English
99. To develop the capacity of man only for war
was the educational aim of the ancient
a. Romans
c. Athenians
b. Spartans
d. Chinese
100.
To produce a young man who would be
charming in person and graceful in manner,
e.g. a beautiful soul in a beautiful body is the
educational aim of education of the
a. Romans
c. Spartans
b. Athens
d. Italians

***** THE END *****


WORK HARD, DREAM HARDER
human growth and development
1
c
51
c
2
c
52
b
3
b
53
a
4
d
54
d
5
a
55
a
6
d
56
b
7
c
57
c
8
a
58
a
9
a
59
a
10
c
60
b
11
a
61
c
12
d
62
d
13
c
63
d
14
d
64
d
15
a
65
a
16
b
66
c
17
a
67
d
18
c
68
a
19
d
69
d
20
d
70
d
21
c
71
a
22
c
72
d
23
b
73
b
24
b
74
b
25
d
75
a
26
c
76
c
27
b
77
e
28
b
78
c
29
a
79
b
30
a
80
b
31
a
81
c
32
a
82
a
33
b
83
b
34
d
84
c
35
d
85
d
36
c
86
d
37
c
87
b
38
b
88
b
39
c
89
c
40
b
90
b
41
a
91
a
42
c
92
a
43
d
93
c
44
d
94
b
45
a
95
c
46
a
96
b
47
b
97
b
48
b
98
c
49
b
99
b
50
a
100
b

LICENSURE EXAMINATION FOR TEACHERS (LET)


Refresher Course
Content Area: MATHEMATICS
Focus: ANALYTIC GEOMETRY
Prepared by: Daisy de Borja-Marcelino
Competencies:
1. Solve problems involving coordinates of a point, midpoint of a line segment, and
distance between two points.
2. Determine the equation of the line relative to given conditions: slope of a line given its
graph, or its equation, or any two points on it.
3. Determine the equation of a non-vertical line given a point on it and the slope of a line,
which is either parallel or perpendicular to it.
4. Solve problems involving
a. the midpoint of a line segment, distance between two points, slopes of lines,
distance between a point and a line, and segment division.
b. a circle, parabola, ellipse, and hyperbola.
5. Determine the equations and graphs of a circle, parabola, ellipse and hyperbola.
I. The Cartesian Plane
Below is a diagram of a Cartesian plane or a rectangular coordinate system, or a coordinate plane.

The two axes separate the plane


into four regions called quadrants.
Points can lie in one of the four
quadrants or on an axis. The points on
the x-axis to the right of the origin
correspond to positive numbers; while
to the left of the origin correspond to
negative numbers. The points on the yaxis above the origin correspond to
An ordered pair of real numbers, called thepositive
coordinates
of a point,while
locatesbelow
a pointthe
in the
numbers;

Cartesian plane. Each ordered pair corresponds to exactly one point in the Cartesian plane.

The following are the points in the figure on the right:


A(-6,3), B(-2,-3), C (4,-2), D(3,4), E(0,5), F(-3,0).

For numbers 1-2, use the following condition: Two insects M and T are initially at a point
A(-4, -7) on a Cartesian plane.
1. If M traveled 7 units to the right and 8 units downward, at what point is it now?
Solution: (-4+7, -7-8) or (-3,-15)
2. If T traveled 5 units to the left and 11 units downward, at what point is it now?
Solution: (-4-5, -7-11) or (-9, -18)
II. The Straight Line
A. Distance Between Two Points

( x1 x2 ) 2 ( y1 y2 ) 2
A. The distance between two points (x1,y1) and (x2,y2) is given by
Example: Given the points A(2,1) and B(5,4). Determine the length AB.

2 5 2 1 4 2

9 2 3 2

9 9 18

92

3 2

Solution: AB =
or
or
.
Exercises: For 1-2, use the following condition: Two insects L and O are initially at a point (-1,3) on
a Cartesian plane.
1. If L traveled 5 units to the left and 4 units upward, at what point is it now?
A) (-6, 7)
B) (4, 7)
C) (-6, -1)
D) (4, -7)
2. If O traveled 6 units to the right and 2 units upward, at what point is it now?
A) (7, 5)
B) (5,5)
C) (-7, 5)
D) (-5, -5)
St. Louis Review Center-Inc-Davao Tel. no. (082) 224-2515

15

3. Two buses leave the same station at 9:00 p.m. One bus travels at the rate of 30 kph and the
other travels at 40 kph. If they go on the same direction, how many km apart are the buses at
10:00 p.m.?
A) 70 km
B) 10 km
C) 140 km
D) 50 km
4. Two buses leave the same station at 8:00 a.m. One bus travels at the rate of 30 kph and the
other travels at 40 kph. If they go on opposite direction, how many km apart are the buses at 9:00
a.m.?
A) 70 km
B) 10 km
C) 140 km
D) 50 km
5. Two buses leave the same station at 7:00 a.m. One bus travels north at the rate of 30 kph and
the other travels east at 40 kph. How many km apart are the buses at 8:00 a.m.?
A) 70 km
B) 10 km
C) 140 km
D) 50 km
6. Which of the following is true about the quadrilateral with vertices A(0,0), B(-2,1), C(3,4) and
D(5,3)?
i) AD and BC are equal
ii) BD and AC are equal
iii) AB and CD are equal
A) both i and iii

B) ii only

C) both ii and iii

D) i, ii, and

iii
7. What is the distance between (-5,-8) and (10,0)?
A) 17
B) 13
C) 23

D) -0.5

B. Slope of a line

y1 y2
x1 x2

a) The slope of the non-vertical line containing A(x 1,y1) and B(x2,y2) is

or

y y
m 2 1
x2 x1

.
b) The slope of the line parallel to the x-axis is 0.
c) The slope of the line parallel to the y-axis is undefined.
d) The slope of the line that leans to the right is positive.
e) The slope of the line that leans to the left is negative.
C. The Equation of the line
In general, a line has an equation of the form ax + by + c = 0 where
numbers and that a and b are not both zero.

a, b, c are real

D. Different forms of the Equation of the line


General form: ax + by + c = 0.
Slope-intercept form: y = mx + b, where m is the slope and b is the y-intercept.

y y1 m( x x1 )

Point slope form:

Two point form:


line.

where (x1, y1) is any point on the line.

y y
y y1 2 1 ( x x1 )
x2 x1

where (x1, y1) and (x2, y2) are any two points on the

x y
1
a b

Intercept form:

Reminders:
A line that leans to the right has positive slope. The steeper the line, the higher the slope
is.

where a is the x-intercept and b is the y-intercept.

p
q
r
The slopes of lines p, q, r are all positive. Of the three slopes, the slope of line p is the
lowest, the slope of r is the highest.

St. Louis Review Center-Inc-Davao Tel. no. (082) 224-2515

16

A line that leans to the left has negative slope. The steeper the line, the lower the slope is.

The slopes of lines t, s, u are all negative. Of the three slopes, t is the highest, while u has the
lowest (because the values are negative.)
Exercises

5x - 4 y +12 = 0
1. What is the slope of
A)1.25

?
B) -1.25

C) 0.8

D) -0.8

2. What is the slope of x = -9?


A) 4
B) 1

C) 0

D) undefined

3. What is the slope of y= 12?


A) 7
B) 1

C) 0

D) undefined

x y
+ =1
4 9
4. What is the slope of

0.4

0.4

A)
2.25

B) 2.25

C) -

D)

E. Parallel and Perpendicular lines


Given two non-vertical lines p and q so that p has slope m 1 and q has slope m2.
If p and q are parallel, then m1 = m2.
If p and q are perpendicular to each other, then m1m2 = -1.
F. Segment division
Given segment AB with A(x1,y1) and B(x2,y2).

M(

x1 x2 y1 y2
,
)
2
2

The midpoint M of segment AB is

AB
If a point P divides

r1
r2
in the ratio

AP r1

PB r2
so that

rx r x
x 1 2 2 1
r1 r2

be obtained using the formula


G. Distance of a point from a line

, then the coordinates of P(x,y) can

r y r y
y 1 2 2 1
r1 r2

and

The distance of a point A(x1,y1) from the line Ax + By + C = 0 is given by

d
Exercises

Ax1 By1 C
A2 B 2
.

1. Write an equation in standard form for the line passing through (2,3) and (3,4).
a. 5x y = -13
b. x 5y = 19
c. x y = -5
17

d. x 5y =

2. Write an equation in slope intercept form for the line with a slope of 3 and a y-intercept of 28.
a. y = 3x + 28
b. y = 0.5x + 28
c. y = 3x + 28
d. y
= 3x + 21
3. Write the equation in standard form for a line with slope of 3 and a y-intercept of 7.
a. 3x y = 7
b. 3x + y = 7
c. 3x + y = 7
d. 3x + y = 7
4. Which of the following best describes the graphs of 2x 3y = 9 and 6x 9y = 18?
a. Parallel
b. Perpendicular
c. Coinciding
d. Intersecting
5. Write the standard equation of the line parallel to the graph of
through (0,1).
a. x + 2y = 2
b. 2x y = 2
c. x 2y = 2
St. Louis Review Center-Inc-Davao Tel. no. (082) 224-2515

x 2y 6 = 0 and passing
d. 2x + y = 2
17

6. Write the equation of the line perpendicular to the graph of x = 3 and passing through (4, 1).
a. x 4 = 0
b. y + 1 = 0
c. x + 1 = 0
d. y 4 = 0
7. For what value of d will the graph of 6x + dy = 6 be perpendicular to the graph 2x 6y = 12?
a. 0.5
b. 2
c. 4
d. 5
III. Conic Section
A conic section or simply conic, is defined as the graph of a second-degree equation in x
and y.
In terms of locus of points, a conic is defined as the path of a point, which moves so that its
distance from a fixed point is in constant ratio to its distance from a fixed line. The fixed point is
called the focus of the conic, the fixed line is called the directrix of the conic, and the constant
ratio is called the eccentricity, usually denoted by e.
If e < 1, the conic is an ellipse. (Note that a circle has e=0.)
If e = 1, the conic is a parabola.
If e > 1, the conic is hyperbola.
A. The Circle
1. A circle is the set of all points on a plane that are equidistant from a fixed point on the
plane. The fixed point is called the center, and the distance from the center to any point of
the circle is called the radius.
2. Equation of a circle
a) general form: x2 + y2 + Dx + Ey + F = 0
b) center-radius form: (x h)2 + (y k)2 = r2 where the center is at (h,k) and the radius
is equal to r.
3. Line tangent to a circle
A line tangent to a circle touches the circle at exactly one point called the point of
tangency. The tangent line is perpendicular to the radius of the circle, at the point of
tangency.
Exercises
For items 1-2, use the illustration on the right.

y
2

1.5

1. Which of the following does NOT lie on the circle?


a. (3,-1)
b. (3,0)
c. (2,-1)
d. (3,-2)

0.5

2. What is the equation of the graph?

y 2 x 3 1

a.

( y 1) 2 x 3 1

c.

-3.5

-3

-2.5

-2

-1.5

-1

-0.5

( y 1) 2 x 3 1

b.

( y 1) 2 x 3 1

0.5

1.5

2.5

3.5

-0.5

-1

-1.5

-2

d.

B. The Parabola
1. Definition. A parabola is the set of all points on a plane that are equidistant from a
fixed point and a fixed line of the plane. The fixed point is called the focus and the fixed line is
the directrix.
2. Equation and Graph of a Parabola
a) The equation of a parabola with vertex at the origin and focus at (a,0) is
The parabola opens to the right if a > 0 and opens to the left if a < 0.

y2 = 4ax.

b) The equation of a parabola with vertex at the origin and focus at (0,a) is
The parabola opens upward if a > 0 and opens downward if a < 0.

x2 = 4ay.

c) The equation of a parabola with vertex at (h , k) and focus at (h + a, k) is


4a(x h).
The parabola opens to the right if a > 0 and opens to the left if a < 0.

(y k)2 =

d) The equation of a parabola with vertex at (h , k) and focus at (h, k + a) is


4a(y k).

(x h)2 =

e) The parabola opens upward if a > 0 and opens downward if a < 0.


f)

Standard form:

g) General form:

(y k)2 = 4a(x h) or (x h)2 = 4a(y k)


y2 + Dx + Ey + F = 0, or x2 + Dx + Ey + F = 0

St. Louis Review Center-Inc-Davao Tel. no. (082) 224-2515

18

3. Parts of a Parabola
a) The vertex is the point, midway between the focus and the directrix.
b) The axis of the parabola is the line containing the focus and perpendicular to the
directrix. The parabola is symmetric with respect to its axis.
c) The latus rectum is the chord drawn through the focus and parallel to the directrix
(and therefore perpendicular to the axis) of the parabola.
d) In the parabola y2=4ax, the length of latus rectum is 4a, and the endpoints of the latus
rectum are (a, -2a) and (a, 2a).

In the figure at the right, the vertex of the parabola is the origin,

LL'
the focus is F(a,o), the directrix is the line containing

CC '
the axis is the x-axis, the latus rectum is the line containing

x2
The graph of

16
y
3
The graph of (y-2)2 = 8 (x-3).

C. Ellipse
1. An ellipse is the set of all points P on a plane such that the sum of the distances of P from two
fixed points F and F on the plane is constant. Each fixed point is called focus (plural: foci).
2. Equation of an Ellipse
a) If the center is at the origin, the vertices are at ( a, 0), the foci are at ( c,0), the
endpoints of the minor axis are at (0, b) and
2

b2 a2 c2
,

then the equation is

x
y
2 1
2
a
b
.
b) If the center is at the origin, the vertices are at (0, a), the foci are at (0, c), the

b2 a 2 c2
endpoints of the minor axis are at ( b, 0) and
2

, then the equation is

x
y
2 1
2
b
a
.
c) If the center is at (h, k), the distance between the vertices is 2a, the principal axis is

b2 a 2 c2

( x h) 2 ( y k ) 2

1
a2
b2

horizontal and
, then the equation is
.
d) If the center is at (h, k), the distance between the vertices is 2a, the principal axis is

( y k ) 2 ( x h) 2

1
a2
b2

b2 a2 c2
vertical and

, then the equation is

St. Louis Review Center-Inc-Davao Tel. no. (082) 224-2515

19

y
B(0,b)

(c, ba )

V(-a,0)F(-c,0) O

(c, ba )

F(c,0) V(a,0) x
2

(c, ba )

(c, ba )

4.

B(0,-b)

Parts of an Ellipse
For the terms described below, refer to the ellipse
shown with center at O, vertices at V(-a,0) and V(a,0),
foci at F(-c,0) and F(c,0), endpoints of the minor axis
at B(0,-b) and B(0,b), endpoints of one latus rectum

b2
a

at G (-c,

H (c,

b2
a
) and G(-c,

) and the other at

b
a

b
a
) and G(c,

).

a) The center of an ellipse is the midpoint of the segment joining the two foci. It is the
intersection of the axes of the ellipse. In the figure above, point O is the center.
b) The principal axis of the ellipse is the line containing the foci and intersecting the
ellipse at its vertices. The major axis is a segment of the principal axis whose endpoints

V 'V
are the vertices of the ellipse. In the figure,
is the major axis and has length of 2a
units.
c) The minor axis is the perpendicular bisector of the major axis and whose endpoints are

B' B
both on the ellipse. In the figure,

is the minor axis and has length 2b units.

G' G
d) The latus rectum is the chord through a focus and perpendicular to the major axis.

2b 2
a

H'H
and

are the latus rectum, each with a length of

(0, 3)
(-4,9)
5

(-5,0)

(4, 9)
5

(4,0)

(-4,0)

(5,0)

O
(-4,- 9)
5

(0, -3)

(4,- 9)
5

y
(2,6)

(-6,4)

St. Louis Review Center-Inc-Davao Tel. no. (082)


(-8,1) 224-2515

(8,5)

(2,1)

(12,1)

O
(8,3)
(2,-4)

20

( x 2) 2 ( y 1) 2

1
100
25

x2 y2

1
25 9
The graph of

The graph of

4. Kinds of Ellipses
a) Horizontal ellipse. An ellipse is horizontal if its principal axis is horizontal. The graphs above
are both horizontal ellipses.
b) Vertical ellipse. An ellipse is vertical if its principal axis is vertical.
D. The Hyperbola
1. A hyperbola is the set of points on a plane such that the difference of the distances of each
point on the set from two fixed points on the plane is constant. Each of the fixed points is
called focus.
2. Equation of a hyperbola
a) If the center is at the origin, the vertices are at ( a, 0), the foci are at ( c,0), the endpoints

x2 y2

1
a 2 b2

b2 c 2 a 2

of the minor axis are at (0, b) and


, then the equation is
.
b) If the center is at the origin, the vertices are at (0, a), the foci are at (0, c), the

y2 x2

1
a2 b2

b2 c2 a2

endpoints of the minor axis are at ( b, 0) and


, then the equation is
.
c) If the center is at (h, k), the distance between the vertices is 2a, the principal axis is

( x h) 2 ( y k ) 2

1
a2
b2

b2 c 2 a 2

horizontal and
, then the equation is
.
d) If the center is at (h, k), the distance between the vertices is 2a, the principal axis is

( y k ) 2 ( x h) 2

1
a2
b2

b2 c2 a2
vertical and

, then the equation is

2. Parts of a hyperbola
For the terms described below, refer to the hyperbola shown which has its center at O,
vertices at V(-a,0) and V(a,0), foci at F(-c,0) and F(c,0) and endpoints of one latus rectum at

b2

a
G (-c,

b2
a
) and

G(-c,

b2

a
) and the other at H (c,

b2
a
) and

H(c,

).

a) The hyperbola consists of two separate parts called branches.


b) The two fixed points are called foci. In the figure, the foci are at ( c,0).
c) The line containing the two foci is called the principal axis. In the
figure, the principal axis is the x-axis.
d) The vertices of a hyperbola are the points of intersection of the
hyperbola and the principal axis. In the figure, the vertices are at ( a,0).

St. Louis Review Center-Inc-Davao Tel. no. (082) 224-2515

21

V 'V
e) The segment whose endpoints are the vertices is called the transverse axis. In the figure
is the transverse axis.

b2 c2 a2
f) The line segment with endpoints (0,b) and (0,-b) where

is called the conjugate

axis, and is a perpendicular bisector of the transverse axis.


g) The intersection of the two axes is the center of the hyperbola .
h) The chord through a focus and perpendicular to the transverse axis is called a latus rectum. In

G' G
the figure,

is a latus rectum whose endpoints are

G (-c,

b2
a

b2
a
) and

G(-c,

) and has a

2b 2
a
length of

3. The Asymptotes of a Hyperbola


Shown in the figure on the right is a hyperbola
with two lines as extended diagonals of
the
rectangle shown.

These two diagonal lines are said to be the asymptotes of the curve, and are helpful in

x2 y2

1
a 2 b2
sketching the graph of a hyperbola. The equations of the asymptotes associated with

b
y x
a
are

and

y
are

y 2 x2

1
a 2 b2

b
y x
a

a
x
b

. Similarly, the equations of the asymptotes associated with

a
x
b

and

y
(6,9)

(-9,6)

F(0,6)

(9,6)
3y x 0

F(-6,0) (-3,0) O

(0,3)
(3,0)

F(6,0)

x
(0,-3)
3y x 0

(6,-9)

F(0,-6)

St. Louis Review Center-Inc-Davao Tel. no. (082) 224-2515

22

x2 y2

1
9 27
The graph of

y2 x2

1
9 27
.

The graph of

PRACTICE EXERCISES
Directions: Choose the best answer from the choices given and write the corresponding letter of
your choice.
For items 1-5, use the illustration on the right.
1. Which of the following are the coordinates of A?
a. (1,2)
b. (2,1)
2. What is the distance between points M and T?

61

M
c. (-3,3)

d. (2,-3)
x

2
1
1
-10

-3 -2 -1

51

-3
-2

a.
units b. 6 sq. units
c.
units d. 8 units
3. Which of the following points has the coordinates (-3,-1)
a. M
b. A
c. T
d. H
4. Which of the following is the area of the triangle formed with vertices M, A and H?
a. 5 sq. units b. 10 sq. units
c. 5 units
d. 10 units
5. Which of the following is the equation of the line containing points A and T?
a. y= 2
b. x=2
c. y+2x=3
d. y-2x+3=0 x
6. Suppose that an isosceles trapezoid is placed on the Cartesian plane as shown
On the right, which of the following should be the coordinates of vertex V?
a. (a,b)
b. (b+a, 0)
c. (b-a,b)
d. (b+a,b)
7. The points (-11,3), (3,8) and (-8,-2) are vertices of what triangle?
a. Isosceles
b. Scalene
c. Equilateral
d. Right

T
y
O(0,b)

0 D(a,0)
E(b,0)

8. What is the area of the triangle in #7?


a. 40.5 sq units
b. 41.8 sq units
c. 42 sq units
d. 46.8
units
9. Which of the following sets of points lie on a straight line?
a. (2,3), (-4,7), (5,8) b. (-2,1), (3,2), (6,3) c. (-1,-4), (2,5), (7,-2) d. (4,1), (5,-2), (6,-5)

sq

3
7
10. If the point (9,2) divides the segment of the line from P 1(6,8) to P2(x2,y2) in the ratio r =
,
give the coordinates of P2.
a. (16,12)
b. (16, 15)
c. (14,15)
d. (12,12)
11. Give the fourth vertex, at the third quadrant, of the parallelogram whose three vertices are (1,-5), (2,1) and (1,5).
a. (-3,-2)
c. (-3,-4)
c. (-4,-1)
d. (-2,-1)
12. The line segment joining A(-2,-1) and B(3,3) is extended to C. If BC = 3AB, give the
coordinates of C.
a. (17,12)
b. (15,17)
c. (18,15)
d. (12,18)
13. The line L2 makes an angle of 600 with the L1. If the slope of L1 is 1, give the slope of L2.
a. (3 + 20.5)
b. (2 + 20.5)
c. (2 + 30.5)
d. (3 + 30.5)
14. The angle from the line through (-4,5) and (3,m) to the line (-2,4) and (9,1) is 135 0. Give the
value of m.
a.7
b. 8
c. 9
d. 10
15. Which equation represents a line perpendicular to the graph of 2x + y = 2?
a. y = -0.5x 2
b. y = 2x + 2
c. y = 2x 2
d. y
= 0.5x + 2
16. Which of the following is the y intercept of the graph 2x 2y + 8 = 0?
a. -4
b. -2
c. 2
d. 4
17. Which of the following may be a graph of x y = a where a is a positive real number?
a.
b. y
c.
d.
y
y

x
y
2

1.5

18. Write an equation in standard form for a line with a slope of 1 passing through (2,1).
a. x + y = 3
b. x + y = 3
c. x + y = 3
d. x y = 3
b.
For items 19-22, use the illustration on the right.
T
19. Which of the following are the coordinates of A?
1

0.5

-3.5

-3

St. Louis Review Center-Inc-Davao Tel. no. (082) 224-2515

-2.5

-2

-1.5

-1

-0.5

-0.5

-1

-1.5

-2

0.5

1.5

23

2.5

3.5

a. (1,1)
b. (1,-1)
c. (-1,1)
d. (-1,-1)
20. What is the distance between points A and H?

61
a.

units

b. 6 sq. units

51
c.
units
d. 8 units
21. Which of the following points has the coordinates (-2,-2)?
a. M
b. A
c. T
d. H
22. Which of the following is the equation of the given graph?

y x2 2

y x 2 2

a.

y x 2

b.

y x2 2
.

c.

d.

.
23. Which of the following is the equation of the line containing points M and T?
a. y= 2
b. x=2
c. y-2x-2=0
d. y+2x+2=0

x2 8 y

x3

24. What is the shortest distance of


from
a. 1 unit
b. 2 units
2

?
c. 3 units

d. 8 units

y
x

1
12 4
25. Which of the following is a focus of
a. (0,-4)
b. (-4,0)
2

c. (0,4)

d. (4,0)

x
y

1
4
9
26. What are the x-intercepts of
a. none

b.

3
c.

d.

27. Which of the following is a graph of a hyperbola?


a.

b.

y
15

15
10

10
5

- 25

-20

- 15

-10

-5

-30

0
-30

10

15

20

25

-25

-20

-15

-10

-5

30

10

15

20

25

30

-5

-5
-10

-10
-15

-15

c.

d.
y

2
2
1 .5
1.5
1
1
0 .5

0.5

0
-3 .5

-3

-2 .5

-2

-1 .5

-1

- 0. 5

0. 5

1. 5

2 .5

3 .5

-3.5

-3

-2.5

-2

-1.5

-1

-0 .5

-0.5

0.5

1.5

2.5

3.5

-0.5

-1

-1

-1 .5

-1.5

-2

-2

28. Which of the following is an equation of an ellipse that has 10 as length of the major axis and
has foci which are 4 units away from the center?

y2 x2

1
25 9
a.

y2 x2

1
9 16

y2 x2

1
5
3

b.

y2 x2

1
16 25

c.

d.

For items 29-31, consider the graph on the right.


29. Which of the following is the equation of
the graph?

100 y 2 25 x 2 2500

10

a.

100 x 2 25 y 2 2500

b.

100 y 2 25 x 2 2500

-30

-20

-10

10

20

c.

100 x 2 25 y 2 2500

-10

d.
30. What are the x-intercepts of the graph?
St. Louis Review Center-Inc-Davao Tel. no. (082) 224-2515

24

30

a. none

c.

10

b.

d.

31. What kind of figure is shown on the graph?


a. circle
b. ellipse

c. hyperbola

32. Which of the following is the center of the graph


shown on the right?
a. (0,0)
b. (0,10)
c. (10,0)
c. (0,-10)
33. Which of the following is a focus of the graph
shown on the right?
-30
a. (0,0)
b. (0,10)
c. (0,5)
c. (0,-10)

d. Parabola
y
10

0
-20

-10

34. What is the area of the shaded region?


a. 4 units
b. 4 square units
c. 16 units
d. 16 square units

10

20

30

-10

LICENSURE EXAMINATION FOR TEACHERS (LET)


Refresher Course
Content Area: MATHEMATICS
Focus: ADVANCED ALGEBRA
Prepared by: Daisy de Borja-Marcelino
Competencies: Show mastery of the basic terms, concepts and equations in Advanced Algebra
involving radicals, rational exponents and functions. Solve, evaluate and manipulate symbolic and
numerical problems in above areas by applying fundamental principles and processes.
KEY IDEAS
Some Helpful Tips in Answering the LET
1

Read the question/s or the items carefully and understand what they say.
St. Louis Review Center-Inc-Davao Tel. no. (082) 224-2515

25

2
3
4
5
6
7
8

Determine what is/are wanted or what is/are asked for.


Find out what is/are given and which data are needed to solve the problem.
Reason out what processes (operations) to apply and the order in which they are to be
applied.
Summarize the problem by means of an open number sentence.
Compute carefully. Check each step in the computations.
Decide the reasonableness of the result.
Check the result by seeing to it that the result satisfies all the conditions of the problem.

RATIONAL EXPONENTS

an
If a is a real number and n is any positive integer, the symbol
denotes the nth power of
a. The real number a is called the base and n is called the exponent. In symbols,

a n a a a ... a
.
n factors

3 3 3 3 34
Examples:

a)

2m 3 2m 2m 2m .
or 81.

1

2
c)

math

b)

1 1 1 1
.
2 2 2 2

1.31.3 1.3 2 .
d)

math math

2 m m 2 m .
2

e)
.
e)
Note that any base raised to the power of 1, is just the base. Moreover, any base raised to
the power of 0 is 1, while 00 is indeterminate.

9 0 1.

41 4.

Examples: a)

c) (-m)1 = - m.

b)

h op e
2

e)

j o y

38.23 0 1

1
.

f)

2 3

5 1

d) z0 = 1.

1 0 1 1
m a a
4
4

j 2o3 y 5

g)

h)

Laws of Exponents
true.

If a and b are real numbers and m and n are positive real numbers, then the following are

n m

a m a n a mn

a nm
.

If

If

a
a mn
n
a

a0

mn
and

, then

and

am
1
n m
n
a
a

a0

nm

ab n a nb n

, then

If

a

b

b0

an
n
b

, then

If

a
a0 1
n
a

a0

, then

2 3

2 2 2
3

4.
6

Examples: a)

3 2

25 32.
c) (3 x 4 )2 = 32 x 42.

b)

32
1.
32

35
352 33 27.
32

d)

e)

f).

3
1
1
325 33 3 .
5
3
3
27
Exercises
1. In the expression 8m5, 5 is called the ________.
A. base
B. coefficient
C. constant
0

25 m
20 m n s

0
5m 10s 6 n8 m 10

2. Evaluate

D. exponent

3 14 0
0

St. Louis Review Center-Inc-Davao Tel. no. (082) 224-2515

26

m 2 20
12

A. 5

4 -2

B. 23m n s

3a 4b 4

( 3a ) 4

C.

D. undefined

4b 4

3. Anthony wrote
=
+
. Which of the following is his misconception?
A. It is possible to factor out the exponent.
B. It is possible to distribute exponents over a sum.
C. The exponents should be multiplied with the base.
D. The coefficient inside the parentheses should be added.
2

5r 4
3
3s

25r 6
?
9s 5

4. Is

equal to
Why or why not?
A. Yes, for the exponents inside the parenthesis and the numerical coefficients should be
added to 2.
B. No, for the exponents inside the parenthesis and the numerical coefficients should be
subtracted from 2.
C. Yes, for the exponents inside the parenthesis should be added and the numerical
coefficients should be
raised to 2.
D. No, for the exponent inside the parenthesis should be multiplied by 2 and the
numerical coefficients should
be raised to 2.

x12
x3
5. Explain why
is not equal to x4?
A. Because the exponents should be added.
B. Because the exponents should be multiplied.
C. Because the numerator should be divided by the denominator.
D. Because the exponent of the numerator should be subtracted by the exponent of the
denominator.
6. Which of the following is the product of x5y3z and x2y4z2?
A. 2x102y122z2
B. x10y12z2
C. 2x72y72z3
D. x7y7z3

a 3 2a 5 11a.

5a
7. Give the product of

and

5a 10a 55a 2
4

5a 3 10a 6 55a

A.

C.

5a 10a 55a
4

B.

5a 3 10a 5 55a

D.

2 2
3

8. Marlon claims that


is equal to
. Is he correct? Why or why not?
A. Yes, for the exponents inside the parenthesis and the numerical coefficients should be
added to 5.
B. No, for the exponents inside the parenthesis should be multiplied by 2 and the
numerical coefficients should be raised to 5.
C. No, for the exponents should be multiplied.
D. Yes, for the exponents should be added.
The mentioned laws of exponents also hold when m and n are positive rational numbers.
Examples: The following are true if there is no zero denominator.

2
2 5
7

5
m3 m3 m3 3 m3

1
1 1

1
3 2 3 2 3 2 2 31 3

a)

5
5

2
3
1
3

2 1

3 3

b)

1
3

c)

5
7
3
7

5 3

7 7

d)

2
7

2
3

e e 23 23 e 0 1
2
e 3
e)

3x y
2

3 4

314 x 24 y 34 34 x 8 y12

f)

St. Louis Review Center-Inc-Davao Tel. no. (082) 224-2515

27

2
5

art

xy

5y

art 5

2
xy 5

g)
Negative Exponents

x 4 1 1 1
11 y
11 y 5 y
6y
3y
x 4 x 4 4 x 4 x 2
.

h)

1
an
If a is a nonzero real number and n is any rational number, then a-n =
.
To simplify algebraic expressions with negative exponents is just to express the given
expression into an equivalent quantity where the exponents become positive.
Examples: Simplify the following such that they only have positive exponents.

1
2

a)

3
4

2m 4

b)

1
3

1
2

c)

3
4

2m 4

1
1

2m 4

Solution: a)
b)
c)
Examples: Simplify the following such that they only have positive exponents. Assume nonzero
bases and no denominator is zero.

16 2
4 4

m 4
m 3

a)

b)

c)

m 4 g 3
g 1h 3

16 2 m 4 at 3
4 4 m 3a 2t 3
e)

f)

2 2

16 2
4 4

a)

4 4
1
4 4
.

1
3 2

1
)
2

( 3)(

1
)
2

5
2

3
2

x 2
h)

1
x 2 2
=

1
x 4x 4

x 2 x 2
or

or

1
1

x 2 x 2

x 2 x 2
is equal to

Note that

g 3 1 h 3 g 4 h 3
4
m4
m

5
2

=
2

= 1.

=
( 5)(

1
m

m 4 g 3 1
1
1 3 4 g 3 1 h3
1 g
h
m

g)

m 4 3 m 43 m 1

4 4 m 3 aa 2t 3 4 4 m3 a 3t 3 a 3
4 43 .
16 2 m 4t 3
4 mt
m

h)

d)

e)

x 2 2

t 3
t 3
16 2 m 4 at 3
4 4 m 3 a 2t 3

f)

b)

m 4 g 3
g 1h 3

3 2

m 4
m 3

a1 2 a1 2 a 3

c)

d)

g)
Solution:

a
a 2

t 3
t 3

a
a 2

1
x 4x 4

1
1 1

2
x
4x 4

but

is not equal to

a
b
Whenever the exponent of a base is in rational form
always be expressed in radical form.

where b 0, the expression can

RADICALS

St. Louis Review Center-Inc-Davao Tel. no. (082) 224-2515

28

an
If n is a positive integer and a is a real number for which
n

is called a radical, and

The symbol
n

is defined, then the expression

1
n

is a radical sign, the number a is the radicand and n is the index of the

radical

.
3

Examples: a) In the expression

, the number 3 is called the index and 5 is the radicand.

27

1
4

27

b) The expression
can be written as
.
When a radical notation has no index, it is understood that n=2 or we are going to extract
the square roots of the radicand.
2

1
2

49 49 .

3 2 3 3.

Examples: a)

b)

Simplified radicals
An expression with radicals is simplified when all of the following conditions are satisfied.
Exponents of the radicand and index of the radical have no common factor except 1.
The radicand has no factor raised to a power greater than or equal to the index.
All indicated operations have been performed (if possible).
No denominator contains a radical.
The radicand has no fractions.

m
n

Radical notation of
n

If a is a real number, m is an integer and


is a real number, then
Examples: Write each exponential expression using radical notation.
1

a3

m4

52

a)

b)

a
Solution: a)

1
3

m
.

am

c)

a
n

an

3
4

b)

( 4 m ) 3
=

5
.

3
2

c)

5
2

53
=

53
or

Examples: Write each radical expression using exponential notation and simplify.
3

22
a)

27

b)

22
Solution: a)

22
=

1
2

x6

c)

27

b)

(27)

1
3

=
n

1
3 3

c)

x6

x2

x3
=

an = a

If n is an even positive integer, then


n

and

if n is an odd positive integer then

a =a
n

.
Examples: Simplify each of the following and give all the roots.
4

625
a)

169x 6

81

b)

c)

d)

Solution:

625 252 25
a)

b)

81

34

St. Louis Review Center-Inc-Davao Tel. no. (082) 224-2515

3.

29

27 x 6 y 9

132 x 3

169x 6
c)

d)

33 x 2 y 3
3

27 x 6 y 9

13x3.
3

=3x2y3.

For all real numbers a and b, and positive integers m and n for which the indicated roots
are real numbers, the following are true.

( a )( b ) =
n

.
n

a mn a

m n

ab

a
b

where b is not equal to zero.

Examples: The following are true.

3 5 15.
a)

b)

3
12 x 6 x 12

81x 24 81x 8 81x

c)

Examples: Simplify each of the following.


3

8
125

a)

32 y 5
243

b)

( 2)
2 2 .
8
8
3

125
5
5
5
53
3

Solution:

a)

32y 5
243

b)

16 2 y 4 y

4
81 3

24 2 y 4 y
4

3 3
4

2 y4 2 y

3 3

2y

2y

2 y 4 54 y
3

Operations on Radical Expressions


Addition and Subtraction
Radicals with the same radicand and the same index are called like radicals. Like radicals
are added or subtracted by using the distributive property of real numbers. Moreover, only like
radicals can be combined.
3

Examples: a) Give the sum of

,3

15
b) Evaluate 10

,2

15
+3

and 4

7
.

15
-

2 6

24
perimeter.

c) If the lengths of the sides of a triangle is

Solution: a)

+3

+2

15

15

7
+4

15

7
=10

cm,

cm and 4cm, give its

7
.

15

b) 10
+3
= 12
.
c) Given a triangle, its perimeter is determined by adding the lengths of its sides.
Hence,

46

2 6

24
cm +

+ 4cm =

2 6
cm +

2 6
=

cm + 4cm

2 6
cm +

4 6
cm+ 4cm =

cm + 4cm.

Multiplication of Radicals
n

Note that

ab
=

a nb

allows multiplication of radicals with the same index.

Examples: Give the product of the following in simplest form. Take only the positive roots.
St. Louis Review Center-Inc-Davao Tel. no. (082) 224-2515

30

2 3

6 8 9 10

a)

(3 7

b)

2 3 m 2 [3

c)

6 9

6 8 9 10

Solution: a)

m 2 (3 m 33 m 2 )

8)

8 10 54 80 54 16 5 54 4 5 216 5

m 3 3 m 2 ]

b)

8 ) (3 7

2 3 m 2 m 2 3 3 m 2 m 2 2 3 m 3
=

m4

-(6)

2m 6m 3 m
=

(3 7
c)

9 7 8

8 ) 9 7 2 82

8 ) (3 7
+

=63-8=55.

Exercises

34 5
1. In the expression
A. base

, 5 is called the ________.


B. index
C. radicand

D. root

2
5

2. Write

as a radical expression.
2

35

A.
5

32

B.

5y

25 3

C.

D.

3. Write

as an exponential expression.
1
3 5

3 5

A.

5y

5y

B.

4. The sides of a triangle measure 3


triangle?

15

D.

15
m, 5

15
m and

15
cm

3
3 5

C.

15

A. 5

5y

5 5y3

B. 8

m. What is the perimeter of the

15
cm

C. 10

15
cm

D. 12

cm

3
5. The side of a square measures 4

3
A. 4

m. Give its perimeter.

3
m

B. 8

3
m

C. 16

3
m

D. 20

FUNCTIONS
A relation is a set of ordered pairs (x, y) such that for every first element x, there
corresponds at least one y. The set of all first elements is called the domain of the relation,
whereas the set of second elements is the codomain of the relation.
A function is a relation such that for every first element x of the ordered pair (x, y), there
corresponds a unique second element y. The set of all first elements is called the domain of the
function, whereas the set of second elements is the range of the function.
Tests for a Function
There are some tests that can determine whether a relation is a function or not. We have a
function if no two pairs in the set consisting of ordered pairs have the same first components.
Examples: a) The relation {(Mr. Cruz, Mark), (Mr. Cruz, Mary), (Mr. Gonzales, Art), (Mrs. Tan, Alice),
(Miss Peralta, Niko)} is not a function because more than one ordered pair have the same first
component- Mr. Cruz.
b) The relation {(-1,0), (0, 1), (1, 2), (2, 3), (3, 4)} has no ordered pair that has the same
first component. Thus, it is a function. Moreover, its domain is the set {-1,0,1,2,3} and its range is
{0,1,2,3,4}.
c) Consider the relation {(-3, -9), (-2, -4), (-1, -1), (0, 0),(1, -1), (2, -4), (3, -9)}. This
relation is considered a function because it has no ordered pair that has the same first component.
A relation may be described by a set of ordered pairs. A function is described by a set of
ordered pairs with no two pairs having the same first components.
Example: The table below shows the relation of the distance traveled by a car for a given length of
time.
d = rt
Number of Hours
1
2
3
4
5
6
7
8
9
10
Distance
60
120
180
240
300
360
420
480
540
600

St. Louis Review Center-Inc-Davao Tel. no. (082) 224-2515

31

It can be observed that the distance traveled depends upon the number of hours or time.
We say that distance is a function of time. The relation can be expressed as d = rt or d = 60t in
this particular example.
Example: The area of a square is a function of the length of its side.
A = s2
Side
1
2
3
4
Area
1
4
9
16

5
25

If a relation described by an equation or defined by a rule, a functional relationship exists if


a change in the independent variable x causes a change in the dependent variable y.
Another method of showing the relationship between the elements of two sets is by means of an
arrow diagram.

Example: An arrow diagram for the relation y = 8x, where x is in set


of whole numbers from 0 to 3, is shown on the right.
1

16

24

0
Example: An arrow diagram for the relation {(x, y) y = x2 + 2} -1
is
shown on the right where x = -3, -2,-1,0,1,2,3 and y = 2,3,6,11.

-2
2

-3
3

11

A relation described by an arrow diagram is a function if:


1
2

there exists a one-to-one correspondence between the elements of the two sets
there exists a many-to-one correspondence between the elements of the two sets.

Another method of identifying a function from a mere relation is through its graph. A graph
of a relation is a function if a vertical line is drawn through the graph will intersect the graph in no
more than one point.
The most fundamental way to graph a function is to plot points. Once the behavior of the
graph of the function becomes familiar, graphing becomes easier. Note that in graphing functions,
we include all possible real numbers in the domain.
We may start by creating a table of values in order to find out the behavior of the function.
It is very important to choose different numbers to get a clear picture of the graph. That is, it is
helpful to generate as many points possible.

1
f x, y y 3 x
2
Example: Graph

1
f x, y y 3 x
2
Solution: We note that
intercept

can be expressed in terms of slope and m = 3 and

0.5
b=
X
f(x)

. By plotting of points, we consider first a table of values.


-2
-1
0
1
-6.5
-3.5
2.5

2
5.5

3
8.5

-0.5

St. Louis Review Center-Inc-Davao Tel. no. (082) 224-2515

32

y
8

0
-14

-12

-10

-8

-6

-4

-2

10

12

14

-2

-4

-6

-8

Note that the when a vertical line is drawn through the graph, it will intersect the graph in
no more than one point. Hence, the given relation is a function.
Example: If we have f(x) = x, this function is called an identity function defined by f = {(x, y) y
= x}.
Let us graph the identity function by assigning to x the values 0, 1, 2. Hence, the
corresponding values of y are 0, 1 and 2 respectively. Moreover, the line is determined by the
following points.
X
-2
-1
0
1
2
3
f(x)
-2
-1
0
1
2
3
We now have the graph on the right.

y
8

0
-14

-12

-10

-8

-6

-4

-2

-2

If f is defined by f = {(x, y) y = b} where the range of the function f consists of one


number, then f is a constant function whose graph is a straight line parallel to the x-axis.
-4

-6

Example:

-8

Graph f(x) = 4.

Solution: We now have first a table of values.


x
f(x)

-2
4

-1
4

0
4

1
4
6

2
4

3
4

0
-7

-6

-5

-4

-3

-2

-1

-1

-2

y2 x
Non-example: Graph

-3

Solution: We now have first a table of values.


x
y

0
0

1
1

1
-1

4
2

4
2

y
4

0
-3

-2

-1

10

11

12

-1

-2

St. Louis Review Center-Inc-Davao Tel. no. (082) 224-2515


-3

-4

33

10

12

14

Observe that the graph of the said set of points shows that if any vertical line drawn
through the graph intersects the graph at more than one point. Hence, it is not a function.
The graph of a relation shows a function if any vertical line drawn through the graph
intersects the graph at no more than one point.
Non-examples: The graphs below do not define functions.
5

2.5

3
1.5

2
1

-7

-6

-5

-4

-3

-2

-1

0.5

0
-8

0
-4.5

-1

-4

-3.5

-3

-2.5

-2

-1.5

-1

-0.5

0.5

1.5

2.5

-0.5

-2
-1

-3
-1.5

-4

PRACTICE EXERCISES
Directions: Choose the best answer from the choices given. Write the corresponding letter of
your choice.
1. In the expression 9m3, 3 is called the ________.
A. base
B. coefficient
C. constant

D. exponent

1
3
77 77

2. Evaluate

72
A. 7

B.

2
7

77

C.

D.

m2n 2

3. Which of the following is equal to

mn3
A. 0

B. 1

C.

2
2

A.

D.

3
4

4. Which of the following is equal to


4

m 2 n3

24

B.

23

C.
3

D.

34m

5. Give the index of the expression


A. 1
B. 2

C. 3

D. 34 m

888tm
6. In the expression
A. Index

, 888tm is called the __________.


B. Radicand
C. Radical sign

D. Exponent

5
5
7. Which of the following is equal to

5
A. 0

B. 1

C.

3 3
8. The length of a rectangle is
the rectangle?
B.

D.

2 2
m and its width is

5 6
A. 10 m2

2 5

m. Which of the following is the area of

9 6
m2

C.

6 6
m2

St. Louis Review Center-Inc-Davao Tel. no. (082) 224-2515

D.

m2

34

5
9. Rosalinda was asked to get the perimeter of a rectangle whose width is 2

mm and whose

5 5
length is

mm. Which of the following should be her answer?

7 10

10

A.

mm

B. 14

mm

10 5

C. 14

mm

D.

mm

42

10. Is the sum of


,
, and
equal to
? Why or why not?
A. No, because the indices should be multiplied.
B. No, because the terms should not be combined for these are not like radicals.
C. Yes, because the radicands should be added and the indices should be copied.
D. Yes, because the terms have no coefficient and the radicands should be added.
3

3x 2

11. Write as an exponential expression:

3x

3x

1
2 3

2 3

A.

B.

C.

7
12. The sides of a triangle measure 2

7
A. 5

D.

7
m, 3

28
m and

7
m

3x 3

3 3x 2

B. 7

m. Give the perimeter of the triangle.

7
m

C. 9

7
m

D. 35

22
13. The side of a square measures 5

22

cm. Give its perimeter.

22

22

11

A. 10
cm
B.15
cm
C. 20
cm
D. 20
cm
14. A secretary can type 324k words in 32k minutes. How many words can she type in a minute?

1

3
A. 322k

B. 326k
7

8x y
27

2 x y x
2

1

3

C.

23

D.

15. Simplify

23k

2x y
2

2 3

A.

2x y

8x3 y 3 x
27

B. -

C.

D.

h 3o 4 p 3e 5
16. Which of the following is the index of the expression
A. 1
B. 2
C. 3
3

17. In the expression


A. Index

D. 4

8m
, 8m is called the __________.
B. Radicand
C. Radical sign

D. Exponent

3 3

4
5

18. Which of the following is equal to


6
10

A.

6
10

C.

3
3
19. Which of the following is equal to

B. 1
2
3

35

B.

A. 0

2
5

3 25
D.

4
5
4
5

?
4

35

35

C.

D.

1
3

x y

20. Evaluate

St. Louis Review Center-Inc-Davao Tel. no. (082) 224-2515

35

x2 y
A. 0

B. 1

x3 y

C.

D.

21. Which of the following sets of ordered pairs describes a function?


A. {(-1, 4), (2, 8), (2, 20), (9, 36), (-3, -12)}
B. {(24, 6), ( 20, 5), (16, 4), (12, 3)}
C. {(1, 2), (-1, 3), (1, 4), (2, 3), (2, 4)}
D. {(1, -1), (-1, 1), (4, -2), (4, 2), (9, 3), (9, -3)}
22. Which of the following sets of ordered pairs DOES NOT describe a function?
A. {(-1, 4), (2, 8), (5, 20), (9, 36), (5, -12)}
B. {(-3, -2), (-2, -1), (-1, 0), (0, 1)}
C. {(24, 6), ( 20, 5), (16, 4), (12, 3)}
D. {(1, 1), (1, 2), (1, 4), (2, 3), (2, 4)}
23. Which of the following sets of ordered pairs describes a function?
A. {(0.5, 4), (1, 8), (1.5, 20), (2, 36), (-1, -12)}
B. {(4, 6), ( 4, 5), (16, 4), (3, 3)}
C. {(-1, 2), (-1, 3), (1, 4), (2, 3), (2, 4)}
D. {(0.25, -1), (0.5, 1), (0.75, -2), (1, 2), (9, 3), (9, -3)}
24. Which of the following sets of ordered pairs DOES NOT describe a function?
A. {(1.5, 1), (2, 1), (2.5, 1), (3, 1), (3.5, 1)}
B. {(-3, -2), (-2, -1), (-1, 0), (0, 1)}
C. {(24, 6), ( 20, 5), (16, 4), (12, 3)}
D. {(9, 1), (9, 2), (9, 4), (9, 3), (9, 4)}
25. Which of the following describes a function?
A.
x
y

1
-2

1
-4

2
-6

4
-8

4
-10

6
-12

6
-14

1
-2

3
4

5
-6

7
8

9
-10

11
12

13
-14

1
2

2
4

3
6

1
8

2
10

3
12

4
14

1
0.5

2
1

3
1.5

4
2

5
2.5

1
3

2
3.5

B.
x
y
C.
x
y
D.
x
y

26. Which of the following describes a function?


A.
x
y

-1
2

-2
4

-3
6

-4
8

-5
10

-6
12

7
14

-1
2

1
4

-2
6

2
8

3
10

3
12

4
14

1
-2

-2
-4

3
-6

-1
8

-2
10

-3
12

4
14

1
-0.5

2
-1

3
-1.5

4
-2

5
-2.5

1
-3

2
-3.5

B.
x
y
C.
x
y
D.
x
y

27. Which of the following arrows or mapping diagrams DOES NOT specify a function?
A.

St. Louis Review Center-Inc-Davao Tel. no. (082) 224-2515

B.

36

1
2
3
4

6
7
8
9

-2
-1
0

C.

-2
2
-3
3
-4
4

9
4
0

D.

2
3

16

28. Which of the following arrows or mapping diagrams specifies a function?


A.

B.

2
4
6

1
4
5

C.

D.

1
2

8
4
2
0

16
C.
7

29. Which of the following arrows or mapping diagrams DOES NOT specify a function?
A.
B.

6
7
8
9

6
7
8
9

C.

-1
0
1

9
4
0

D.

-2
2
-3
3
-4
4

-4

-9

12

-16
4

30. Which of the following graphs is a function?


St. Louis Review Center-Inc-Davao Tel. no. (082) 224-2515

37

A.

C.
3

2.5
1.5

2
1

1.5
0.5

0
-3.5

-3

-2.5

-2

-1.5

-1

-0.5

0.5

1.5

2.5

3.5

0.5

-0.5

0
-1

-3.5

-3

-2.5

-2

-1.5

-1

-0.5

0.5

1.5

2.5

3.5

-0.5

-1.5

-1

-2

B.

D.

-1.5

2.5

1.5
1
1

0.5

-3.5

-3

-2.5

-2

-1.5

-1

-0.5

0
-3

-2

-1

0
0

0.5

1.5

2.5

3.5

-1

-0.5

-1

-2

-1.5
-3

31. Which is NOT a graph of a function?


A.
B.
100
80
60
40
20
0
-6 -5 -4 -3 -2 -1
-20 0
-40
-60
-80
-100
-120

C.

D.
30

25

20

0
-4

15
1

-3

-2

-1

-2

10

-4

-6

0
-6 -5 -4 -3 -2 -1 0
-5

-8

-10

-10

30
25
20
15
10
5
0
-6 -5 -4 -3 -2 -1 0
-5

-10

32. Which of the following is NOT a graph of a function?


A.

C.

2.5

1.5

1.5

1
0.5

0
-3.5

0.5

-3

-2.5

-2

-1.5

-1

-0.5

0.5

1.5

2.5

3.5

-0.5

-1

-0.5

-1

0
-1.5

0.5

1.5
-1.5

-2

-0.5

B.

D.
y

0.5

0
-1.5

-1

-0.5

0.5

1.5

-0.5

0
-3

-2

-1

-1
-1

-1.5
-2

-2
-3

St. Louis Review Center-Inc-Davao Tel. no. (082) 224-2515

38

33. Which of the following graphs represents a function?


A.

C.

1
1

0
-3

-2

-1

3
-3

-2

-1

-1
-1

-2
-2

-3
-3

C.

D.
y

2
4

1
3

0
-3

-2

-1

-1
1

-2

0
-3

-2

-1

-3

34. Which of the graphs below best represents the following scenario? Chok was at home when he
decided to visit his friend. After a few hours, he was already at his friends house which is a few
kilometers away from his home. He stayed there for a few hours and went back home.
A.

C.

Distance

Distance

Time

Time
B.

D.

Distance

Distance

Time

Time

35. Which of the graphs below best represents the following scenario? Avel is running at a steady
rate and then comes to a hill, which causes him to run at a slower rate. Once he reaches the top
of the hill, he runs down the hill very fast. Upon reaching the bottom of the hill, he resumes his
original pace.
A.

B.

Speed

Speed

Time

St. Louis Review Center-Inc-Davao Tel. no. (082) 224-2515

Time

39

C.

D.

Speed

Speed

Time

Time

Content Area: MATHEMATICS


Focus: Probability and Statistics
Prepared by: Daisy de Borja-Marcelino
St. Louis Review Center-Inc-Davao Tel. no. (082) 224-2515

40

LET Competencies:
Counting Techniques
Experiment: any activity that can be done repeatedly (e.g. tossing a coin, rolling a die).
Sample space: the set of all possible outcomes in an experiment.
Example: In rolling a die, the sample space is S = {1, 2, 3, 4, 5, 6}.
Sample point: an element of the sample space.
Example: In rolling a die, there are 6 sample points.
Counting Sample Points
1. Fundamental Principle of Counting (FPC)
If a choice consists of k steps, of which the first can be performed in n 1 ways, for each of
these the second can be performed in n 2 ways, for each of these the third can be
performed in n3 ways, and for each of these the kth can be made in n k ways, then the
whole choice can be made in n1n2n3nk ways.
Example: In how many ways can two dice fall?
Ans. : 6 6 = 36 ways
2. Permutation
Permutation is an arrangement of objects wherein the order is important.
a. Linear Permutation
If n objects are to be arranged r objects at a time, then the number of distinct arrangements is
given by
n

Pr =

n!
, n r
.
( nr ) !

Example: In how many ways can the first, second and third winners may be chosen with 10
contestants?
10P3 =

10 !
=10 9 8=720 ways
( 103 ) !

b. Circular Permutation
If n objects are to be arranged in a circular manner, then the number of distinct arrangements
is (n - 1)!
Example: In how many ways can 6 people be arranged around a circular table?
Answer: (6 - 1)! = 5!
c. Permutation with Repetitions
The number of permutations of n things of which n1 are one of a kind, n2 second of a kind, , nk of
a kth kind is

n!
n1! n2 !...nk !
Example: How many different permutations are there in the word WAGAYWAY if all letters are to be
taken?

8!
2 !3!2 !
Answer:
3. Combination
Combination is the arrangement of objects regardless of order. In other words, the order of
arranging the objects is not important. If n objects are to be arranged r at a time, the number of
distinct combinations is given by:
n

Cr =

n!
,n r
.
r ! ( nr ) !

Example: In how many ways can a committee of 3 be chosen from 7 persons?

7!
3!4 !
Answer:
Probability
Probability: the likelihood of occurrence of an event.
St. Louis Review Center-Inc-Davao Tel. no. (082) 224-2515

41

If E is any event, then the probability of an event denoted by P(E) has a value between 0
and 1, inclusive. In symbols,
0 P (E) 1
If P(E) = 1, then E is sure to happen.
If P(E) = 0, then E is impossible to happen.
Moreover, if the probability that E will not happen is P(E), then P (E) + P (E) = 1.
Theoretical Probability
Theoretically, the probability of an event E, denoted by P(E), is defined as

n( E )
n(S )
P(E) =
where n(E) = number of favorable outcomes
n(S) = number of possible outcomes
Exercises
1. A coin is flipped once.
a. How many possible outcomes are there?
b. What are these outcomes?
c. How many of these outcomes consist of a face facing up?
d. What is the probability that a tail faces up?
2. A spinner can land on any of the numbers 1 through 12 with equal likelihood. What is the
probability that the spinner lands on a/an
a. odd number?
b. number greater than 12?
c. prime number?
d. number divisible by 3 or 4?
e. number divisible by 3 and 4?
f. number between 3 and 9?
3. A box contains cards printed with the letters of the word PHILIPPINES, so that P is one 3
cards, H is on one card, and so on. A card is drawn from the box.
a. How many possible outcomes are there?
b. What are these outcomes?
c. Are these outcomes equally likely?
d. What is the probability that the card drawn is
i. a P
ii. a H
ii. a I
iv. not a P
STATISTICS
Statistics is the branch of mathematics used to summarize quantities of data and help
investigators draw sound conclusions. Its two main branches are descriptive statistics and
inferential statistics.
A sample is a specified set of measurements or data, which is drawn from a much larger
body of measurements or data called the population.
Kinds of Sampling
1. Random sampling techniques are used to ensure that every member of the population has
an equal chance of being included in the sample. A random sample is said to be representative
of the entire population. The two methods of random sampling are lottery method and the use
of the table of random numbers.
2. Systematic sampling is a technique which selects every nth element of the population for the
sample, with the starting point determined at random from the first n elements.
3. Stratified random sampling is a technique of selecting simple random samples from
mutually exclusive groupings or strata of the population.
Graphical Representations of Data
Graphs are used in mathematics to show relationships between sets of numbers. Graphs
are useful in the field of statistics because they can show the relationships in a set of data.
1. Histogram - a graphical picture of a frequency distribution consisting of a series of vertical
columns or rectangles, each drawn with a base equal to the class interval and a height
corresponding to the class frequency. The bars of a histogram are joined together, that is, there
are no spaces between bars.
2. Bar Chart- uses rectangles or bars to represent discrete classes of data. The length of each
bar corresponds to the frequency or percentage of the given class or category. The categories are
in turn placed in either horizontal
3. Frequency Polygon- a special type of line graph, where each class frequency is plotted
directly above the midpoint or class mark of its class interval and lines are then drawn to connect
the points.
4. Pie Chart- an effective way of presenting categorized (qualitative) distributions, where a circle
is divided into sectors - pie-shaped pieces - which are proportional in size to the corresponding
frequencies or percentages.
5. Pictogram- known as picture graph where picture symbols are used to represent values.
MEASURES OF CENTRAL TENDENCY
A measure of central tendency is a single, central value that summarizes a set of
numerical data. It describes a set of data by locating the middle region of the set.
St. Louis Review Center-Inc-Davao Tel. no. (082) 224-2515

42

Measure
s
of
Central
Tendenc
y
Mean

Definition

The sum of the data


divided by the
number of data

How to find

Ungrouped data:

x
N

Grouped Data:

Median

The middle number


of the set when the
data are arranged in
numerical order

xf
N

Ungrouped data:
The middle for the

( N 2+ 1 )t h score
Grouped Data:

Advantages

Disadvantages

A single, unique
value that is
representative
of all the scores

Not appropriate
for skewed
distribution as it
is affected by
extreme scores
or outliers

Stable from
group to group
May be used in
further
computations

More stable
from group to
group than the
mode
Appropriate for
skewed
distribution

Mdn = L +

Not necessarily
representative
of all scores
Unstable from
group to group
Cannot be used
in further
analyses

n
cf
2
i
f

( )
Mode

The number that


occurs most
frequently in the
data

Ungrouped data:
The most frequent
score

Easy to obtain

Not necessarily
representative
of all scores

Grouped Data:
Cannot be used
The class mark of
in further
the class interval
analyses
with the highest
frequency
Example: The scores for five students on a quiz are 40, 20, 30, 25, and 15. To find the mean score
for this group of students, we first find the sum of the scores:
40+20+30+25+15=130
We then divide the sum by 5, the number of scores:

130
26
5
.
Example: Determine the median for the set of scores
82,
81,
80,
87,
20
Solution: We arrange the scores in Example 1 in order fro lowest to highest:
20,
80,
81,
82,
87
The middle number in this arrangement is 81 and it is the median.
Exercises
1. The mean score on a set of 10 scores is 71. What is the sum of the 10 test scores?
2. The mean score on a set of 13 score is 77. What is the sum of the 13 test scores?
3. The mean score on four of a set of five scores is 75. The fifth score is 90. What is the sum
of the five scores? What is the mean of the five scores?
4. Two sets of data are given. The first set of data has 10 scores with a mean of 70, and the
second set of data has 20 scores with a mean of 80. what is the mean for both sets of data
combined?
St. Louis Review Center-Inc-Davao Tel. no. (082) 224-2515

43

MEASURES OF VARIABILITY
A measure of variation or variability describes how large the differences between the
individual scores. The common measures of variability are range and standard deviation.
Measure
s
of
Variation

Definition

How to find

Advantages

Disadvantages

Range

The difference between


the highest score and
the lowest score

Ungrouped
data:
R = HS - LS

Easy to compute

Unstable

Gives a unique
value

Not
representative
of the set of
data

Grouped Data:

Standard
deviation

The square root of the


variance of the set of
data

R = Upper Limit
of the Highest
Class Interval Lower Limit of
the Lowest Class
Interval
Ungrouped
data:
S

( x x )

to

Not
used
in
further
computations
Most stable
Gives a unique
value

Most
representative

Grouped Data:
S

Easy
understand

Affected
by
extreme scores
More difficult to
compute
and
understand

Used in further
computations

f ( x x )
n

Exercises
1. On a quiz, the following scores were made in a class of 10 students: 72, 83, 86, 97, 90, 70, 65,
71, 80, 86. For this set of scores, give the
a. mean
b. median
c. mode
d. range
e. midrange
f. standard deviation
2. Jao, Dhei, Nelfe, Dada and Ched are all in the same statistics class. Their scores for the first two
exams in the class are listed in the accompanying table. The first exam had a mean of 84 and a
standard deviation of 6, whereas the second exam had a mean of 78 and a standard deviation of
4.
__________________________________
Exam 1
Exam 2
__________________________________
Jao
84
78
Dhei
90
74
Nelfe
66
78
Dada
78
70
Ched
84
78
__________________________________
a.
b.
c.
d.
e.

Who improved on the second exam?


Who improved the most on the second exam?
Who did not improve on the second exam?
Considering both exams, who did the poorest?
Who performed the same on both exams?

PRACTICE EXERCISES
1. In a political science survey, voters are classified into six income categories and five education
categories. In how many different ways can a voter be classified?
A. 11
B. 25
C. 30
D. 45
2. The number of permutations of the word probabilities is
A. 13
B. 2!3!
C. 13!
D. 13!/(2!3!)
3. The number of ways seven visitors can be seated on a round table is
A. 6!
B. 7!
C. 6!7!
D. 6!/2
St. Louis Review Center-Inc-Davao Tel. no. (082) 224-2515

44

4. In a certain town, 40% of the people have brown hair, 25% have brown eyes, and 15% have
both brown hair and brown eyes. A person is selected at random from the town. If he has brown
eyes, what is the probability that he does not have brown hair?
A. 3/8
B. 2/5
C.
D. none of these
5. A die is tossed. If the number is odd, what is the probability that it is prime?

2
3

3
4

2
5

1
3

A.
B.
C.
D.
6. An accounting professor can assign grades of A, B, C, D, or F to students examinations. In how
many ways can the professor assign grades to three different student examinations?
A. 120 ways
B.125 ways
C. 25 ways
D. 50 ways
7. A bag contains 15 red beads, 30 white beads, 20 blue beads, and 7 black beads. If one of the
beads as drawn at random, what is the probability that it will be white or blue?

25
36

15
42

13
40

17
52

A.
B.
C.
D.
8. A whole number is chosen at random from the whole numbers from 1 to 50. What is the
probability that it is an even square?
A. 0.07
B. 0.06
C. 0.02
D. 0.09
9. A pair of dice is tossed. If the numbers appearing are different, find the probability that the sum
is even.

A.

1
2

B.

2
3

C.

2
5

D. none of these

10. If repetitions are not permitted, how many 3 digits number less than 400 can be formed from
the digits 2, 3, 5, 6, 7 and 9?
A.30
B.126
C.40
D. 162
11. In how many different ways can a true-false test be answered if there are five items, assuming
that the student give an answer for each item.

52

25

A.5!
B.2!
C.
D.
12. There are five musical numbers in a program. The number of ways these numbers can be
presented is

5!/ 3!2!

25
A. 5
B.5!
C.
13. The numbers of permutations of the word probabilities is

A.13

B. 13!

D.

13!/ 2!3!

C.2! 3!

D.

14. The number of ways seven students can be seated on a round table is
A. 6!
B.6! 7!
C.7!
D. 6! /2
15. How many different signals, each consisting of 6 flags hung in a vertical line, can be formed
from 4 identical red flags and 2 identical blue flags?

4!2!

6 4!

A. 6!
B.
16. The probability of getting a black heart is

C.4! 2!

D. 6!

14

1 52

A.0
B.
C.1
D.
17. In a certain town, 40% of the people have brown hair, 25% have brown eyes, and 15% have
both brown hair and brown eyes. A person is selected at random from the town. I he has brown
eyes, what is the probability that he does not have brown hair?

38

12

25

A.
B.
C.
For items 18-20: The scores of 10 students in a Math quiz are as follows:
Student
Score

A
18

B
32

C
10

D
15

E
27

F
18

G
23

H
28

D. None of these
I
29

J
30

18. What is the median score?


A. 18
B. 20
C. 23
D. 25
19. What is the mode of the scores?
A. 18
B. 20
C. 23
D. 25
20. What is the range of scores?
A. 18
B. 22
C. 23
D. 25
21. Jojo aims to get an average of 90 in 5 unit tests. The results of the first four unit tests are as
follows: 89, 92, 86, and 91. What should his grade in the fifth unit be for his average to be at least
90?
A. 91
B. 92
C. 93
D. 94
St. Louis Review Center-Inc-Davao Tel. no. (082) 224-2515

45

LICENSURE EXAMINATION FOR TEACHERS (LET)


Refresher Course
Content Area: MATHEMATICS
Focus: Calculus
Prepared by: Daisy de Borja-Marcelino
LET Competencies:

Solve for the roots of a given quadratic equation


Solve problems on quadratic equations
Determine an equation given a set of roots which are imaginary/complex numbers
Perform operations involving exponential and logarithmic functions
Solve for the solution set of a given inequality
Determine the rth term of the expansion (a + b)n
Solve problems involving variations
Determine the number of positive and negative roots of a given polynomial

Equations
An equation that contains at least one variable is called an open sentence. Equations

b&c
above are examples of open sentences. In equation b, only -1 makes the sentence true or
satisfies the equation. However, more than one number might satisfy an equation. For example,

x2 4 0
+2 and -2 satisfy the equation
. Any number that satisfies an equation is called a
solution or root to the equation. The set of numbers from which you can select replacements for
St. Louis Review Center-Inc-Davao Tel. no. (082) 224-2515

46

the variable is called the replacement set. The set of all solutions to an equation is called the
solution set to the equation. To solve an equation means to find all of its solutions.
QUADRATIC EQUATION
An equation of the form ax2 + bx + c = 0 where a
quadratic equation.

0, a ,b, and c are constants, is a

ROOTS OF QUADRATIC EQUATIONS


To solve a quadratic equation means to find the value of x (unknown) that will satisfy the
given equation. The values of x that will make the equation true are called the roots or solution of
the quadratic equation.
Methods of Finding the Roots of a Quadratic Equation
1. Factoring (Use this method if the quadratic equation is factorable)
Example: Determine the roots of x2 -8x - 15 = 0
Solution: Factoring the left side of the equation,
(x - 5) (x 3) = 0

2. Quadratic Formula

Equating each factor to zero,


(x - 5) = 0
(x 3) = 0
x=5
x=3

b b 2 4ac
2a

The quadratic formula is


BINOMIAL FORMULA
To obtain the terms of the binomial expansion (a + b)n, we use the binomial formula:

a n na n 1 b

n (n 1)a n 2 b 2 n (n 1)( n 2)a n 3 b 3

... nab n 1 b n
2!
3!

(a + b)n =
THE rth TERM OF THE EXPANSION (a + b)n

n (n 1)( n 2)...( n r 2)a n r 1 b r 1


(r 1)!
rth term =
EXPONENTIAL FUNCTIONS

The exponential function f with base b is denoted by f(x) =


bx, where

Properties of f(x) = bx

b > 0 , b

1, and x is any real number.

f has the set of real numbers as its domain.

f has the set of positive real numbers as its range.

f has a graph with a y-intercept of (0,1).

f is a one-to-one function.

f has a graph asymptotic to the x-axis.

f is an increasing function if b>1 and f is a decreasing function if 0<b<1.


An exponential function has a constant base and a variable exponent.

The Natural Exponential Function


St. Louis Review Center-Inc-Davao Tel. no. (082) 224-2515

47

f ( x) e x
For all real numbers x, the function defined by
is called the natural exponential
function. Note that e is an irrational number and its accurate value to eight places is 2.71828183.
EXPONENTIAL EQUATIONS
An equation where the unknown quantity appears in an exponent is called an exponential
equation.
SOLVING EXPONENTIAL EQUATIONS
To solve an exponential equation is to find the value of the unknown quantity in the given
equation.
LOGARITHMIC FUNCTIONS

For x > 0, b > 0, and b


Note: only if ay = x.

1, we have y = log a(x) if and

1. If the base of the logarithm is not indicated it is understood that the base is 10.
2. If the base of the logarithm is the number e, then it is called a natural logarithm and it is
written as f(x) = ln x.

f x log b x

Properties of

f has the set of positive real numbers as its domain.

f has the set of real numbers as its range.

f has a graph with a x-intercept of (1,0).

f is a one-to-one function.

f has a graph asymptotic to the y-axis.

f is an increasing function if b>1 and f is a decreasing function if 0<b<1.

Remarks

Logarithmic functions are the inverse of exponential functions.

We can use the rules of exponents with logarithms.

The two most common logarithms are called common logarithms and natural logarithms.
Common logarithms have a base of 10, and natural logarithms have a base of e.

Equation in exponential form can be rewritten in logarithmic form, and vice versa.

y log b x
The exponential form of

by x
is

Example1: The exponential equation 72 = 49 may be written in terms of a logarithmic equation as


log7 (49) = 2.

1
93
Example 2: The exponential equation 9-3 =

1
729
or

may be written in terms of a logarithmic

729

equation as log9

= -3

Basic Properties of Logarithms


Property 1 : loga (1) = 0 because a0 = 1.
St. Louis Review Center-Inc-Davao Tel. no. (082) 224-2515

48

Example 1: In the equation 220 = 1, the base is 22 and the exponent is 0. Remember that a
logarithm is an exponent, and the corresponding logarithmic equation is log 22 (1) = 0, where the 0
is the exponent.

2

5
Example 2: In the equation

2
5
= 1, the base is

and the exponent is 0. Remember that a

log 2
5

logarithm is an exponent, and the corresponding logarithmic equation is

1 = 0.

Property 2: loga (a) = 1 because a1 = a


Example 3: In the equation 71 = 7, the base is 7, the exponent is 1, and the answer is 7. Since a
logarithm is an exponent, and the corresponding logarithmic equation is log 7 7 = 1
Example 4: Use the exponential equation m1 = m to write a logarithmic equation. If the base m is
greater than 0, then logm (m) = 1.
Property 3: loga (a)x = x because ax = ax
Example 5: Since 92=92, we may write the logarithmic equation with base 9 as log9 92 = 2.
Example 6: Since you know that 112=112, we may write the logarithmic equation with base 11 as
log11112 = 2.
INEQUALITIES
Any relation expressed using the symbols <, >, > or < is called an inequality.
An absolute inequality is an inequality which is always true. A conditional inequality is one
which is true only for certain values of the variable involved.
1. 4 > 3 is an absolute inequality
2. x > 3 is a conditional inequality
PROPERTIES OF INEQUALITIES
Let a, b, c, & d be real numbers. The following hold.
1. Trichotomy Property
a > b or a < b or a = b
2. a > b if a - b > 0
a < b if a b < 0
3.
a. If a> 0 and b> 0, then a + b> 0 and ab>0.
b. If a < 0 and b < 0, then a+b< 0 and ab> 0
4. Transitivity
If a < b and b < c then a < c.
5. Addition Property
If a < b and c < d, then a + c < b+ d
6. Multiplication Property
If a < b and c > 0, then ac < bc
If a < b and c < 0, then ac > bc
SOLVING INEQUALITIES
To solve an inequality means to find the value of the unknown that will make the inequality
true.
POLYNOMIAL FUNCTION
The function defined by the equation
f(x) = a0xn +a1xn-1 + a2xn-2 + . . .+ an-2x2 + an-1x + an

0
where n is a nonnegative integer and a 0, a1,

. . .,

an are constants, a0,

is a polynomial

function in x of degree n. The zeros or roots of f(x) are the numbers that will make f(x) = 0.
St. Louis Review Center-Inc-Davao Tel. no. (082) 224-2515

49

The Number of Positive and Negative Roots of a Polynomial Function


If f(x) is a polynomial function with real coefficients, then the following are true.
The number of positive real zeros of f(x) is either equal to the number of variations in sign in
f(x), or to that number diminished by a positive even integer.
The number of negative real zeros of f(x) is either equal to the number of variations in sign in f(x), or to that number diminished by a positive even integer.

LICENSURE EXAMINATION FOR TEACHERS (LET)


Refresher Course
Content Area: MATHEMATICS
Focus: ARITHMETIC, NUMBER THEORY AND BUSINESS MATH
St. Louis Review Center-Inc-Davao Tel. no. (082) 224-2515

50

Prepared by: Daisy de Borja-Marcelino


Competencies:
1. Simplifying expressions involving series of operations
2. Solve problems involving
a. GFC and LCMF
b. prime and composite
c. divisibility
d. inverse and partitive proportions
e. compound interest
f. congruence
g. linear Diophantine Equation
3. Apply Eulers function and theorems, or Fermats theorem in solving problem.

THE NUMBER SYSTEM

25 , 49 , negative numbers
Example: Some examples of imaginary numbers are:

, 3i, -7i.

Example: Simplify: 2 (3 + 2i) 5 (4 6i)


Solution: 2 (3 + 2i) 5 (4 6i) = ( 6 + 4i) (20 30i) = 6 + 4i 20 + 30i = -16 + 34i.
Rational numbers are numbers which can be expressed as quotient of two integers, or can be

2
3
expressed as fractions in simplest forms. Examples are 8, -3, 3.45, and
.
Irrational numbers are numbers which cannot be expressed as fractions in simplest forms.

3
3

Examples are
,4
, , e and
.
Set of Natural/Counting numbers: {1, 2, 3, 4, }. This set contains the numbers that we use in
counting; also called natural numbers.
Set of Whole Numbers: { 0 , 1, 2, 3, }. This set is the union of the number zero and the set of
counting numbers.
Set of Integers: { , -3, -2, -1, 0, 1, 2, 3, }. This set is the union of the set of counting numbers,
their negatives, and zero
II. THE COUNTING NUMBERS
A. Divisibility. An integer is divisible by a certain divisor (also an integer) if it can be divided
exactly by that divisor. That is, the remainder is zero after the division process is completed.
To illustrate, the integer 12 is divisible by 1, 2, 3, 4, 6, and 12.
To determine whether the integer is divisible by a certain integer or not, you may use the
following divisibility rules.
St. Louis Review Center-Inc-Davao Tel. no. (082) 224-2515

51

An integer is divisible by
a) 2 if it ends with 0, 2, 4, 6, or 8. (Examples: 134 or 12 or 12,330 or 4)
b) 3 if the sum of the digits is divisible by 3. (Examples: 132 or 18 or 12,330 or 45)
c) 4 if the last two digits form a number which is divisible by 4. (Examples: 13,412 or 12,332)
d) 5 if it ends with 0 or 5. (Examples: 135 or 10 or 12,330 or 495)
e) 6 if it ends with 0, 2, 4, 6, 8 and the sum of the digits is divisible by 3.(Examples: 134 or
12)
f) 7 if the difference obtained after subtracting twice the last digit from the number formed
by the remaining digits is divisible by 7. (Examples: 14 or 364)
g) 8 if the last three digits form a number which is divisible by 8. (Examples: 24160 or 5328)
h) 9 if the sum of the digits is divisible by 9. (Examples: 9, 432 or 18,504 or 270)
i) 10 if it ends with 0. (Examples: 120 or 7, 890 or 1, 230)
j) 11 if the difference between the sum of the digits on the even powers of 10 and the sum of
the digits on the odd powers of 10 is divisible by 11. (Examples: 2123 or 2816 or 94369 or
36465)
k) 12 if it is both divisible by 3 and 4. (Examples: 413,412 or 112,332)
l) 15 if it is both divisible by 3 and 5. (Examples: 150 or 350)
Remarks: Divisibility rules for two or more relatively prime numbers (GCF is 1) may be combined
to serve as a divisibility rule for their product.
Example: The rules for 3, 4, and 5 may be combined to serve as the rule for their product which is
60 since 3, 4, and 5 are relatively prime.
Exercises: Put a check mark on the space provided for, if the integer on the first column divides exactly the integer on the
top row.
456

36,720

800,112

456

2
3
4
5
6
7
8
9

36,720

800,112

10
11
12
14
24
32
45
77

Even numbers are whole numbers which can be divided exactly by two whole numbers.
Odd numbers are whole numbers which cannot be divided exactly by two whole numbers.
Example: If n3 is odd, which of the following is true?
I. n is odd
II. n2 is odd
III. n2 + 1 is odd
A) II only
B) I and II only

C) I only
D) I and III only

Example: If x is an odd integer and y is an even integer, which of the following is an odd integer?

x 2 +3y
A. 2x-y

C.

x +y- 1

x- 1

B.
D.
B. Factors and Multiples. In the number sentence 2 x 3 = 6, the numbers 2 and 3 are called
factors, while 6 is their product. Or we say, 2 and 3 are divisors of 6. Moreover, we say that 6 is a
multiple of 2 and 3.
Example: How many factors does 42 have?
A) 2
B) 4
c) 5
Answer:
(C). The factors of 42= 16 are {1, 2, 4, 8, 16}.
Example: What are the multiples of 6?
Answer: The multiples of 12 are {12, 24, 36, 48, }
a.
b.
c.
d.
e.

______
______
______
______
______

D) 16

Exercises Fill in the blanks with either 7 or 42.


is a factor of ______.
is divisible by ______.
is a divisor of ______.
is a multiple of ______.
divides _______.

C. Prime and Composite Numbers


Prime numbers are counting numbers that have exactly two factors in the set of counting
numbers: 1 and itself.
St. Louis Review Center-Inc-Davao Tel. no. (082) 224-2515

52

Composite numbers are counting numbers that have more than two factors in the set of counting
numbers.
The numbers 0 and 1 are special numbers. They are neither prime nor composite.
Example: What is the sum of prime numbers less than 15?
A) 4
B) 5
C) 6
D) 14
Answer: The number 2,3,5,7, 11 and 13 are prime number less than 15. Hence, the answer is C.

D. Prime Factorization. This is a process of expressing a number as product of prime factors.


Example:
Solution:

Express 24 as product of prime factors.


24= 2 x 2 x 2 x 3 = 23 x 3 or 3 x 23 .

Fundamental Theorem of Arithmetic


Every composite whole numbers can be expressed as the product of primes in exactly one
way (the order of the factors is disregarded).
E. The Greatest Common Factor (GCF)
The GCF of two or more numbers is the largest possible divisor of the given numbers.
Example: Determine the GCF of 12 and 42.
Solution:
24 = 2 x 2 x 3
42 = 2 x 3 x 7
GCF:
2x3=6
Example: What is the greatest integer that can divide the numbers 18, 24 and 36?
Solution:
18 = 3 x 3 x 2
24 = 3 x 2 x 2 x 2
36 = 3 x 3 x 2 x 2
GCF: 3 x 2 = 6
F. Least Common Multiple (LCM). The LCM of two or more numbers is the smallest possible
number that can be divided by the given numbers.
Example: Give the LCM of 20 and 30.
Solution:
20 = 2 x 2 x 5 = 22 x 5
30 = 2 x 3 x 5
LCM: 22 x 3 x 5 = 60.
Example: What is the smallest integer that can be divided by the numbers 24, 36 and 54?
Solution:
24 = 2 x 2 x 2 x 3 = 23 x 3
36 = 2 x 2 x 3 x 3 = 22 x 32
54 = 2 x 3 x 3 x 3 = 2 x 33
LCM: 23 x 33 = 216
G. Relatively Prime. Two numbers are relatively prime if their GCF is 1. The numbers themselves
may not be prime. The numbers 12 and 49 are relatively prime.
Example:

Which of the following pairs are relatively prime to each other?


A)15 and 36
B) 23 and 51
C) 231 and 27
D) 121 and 330

III. INTEGERS
Consecutive integers are two or more integers, written in sequence, in which each integer after
the first is 1 more than the preceding integer.
Examples:

1,2,3,4,5, 6
3, 4, 5, 6, 7, 8

4, 3, 2, 1, 0, 1, 2, 3

x, x+1, x+2, x+3, x+4, x+5

The absolute value of a number x, denoted by x , is the undirected distance between x and 0
on the number line.

St. Louis Review Center-Inc-Davao Tel. no. (082) 224-2515

53

It is also defined as

x
x

if x 0
if x < 0

Examples:
Evaluate each of the following.
a) 2 = 2

c) 0 = 0

b) 7 = 7

d) 15 = 15

A. Multiplication. The product of two integers with like signs is a positive while the product of
two integers with unlike signs is negative.
Example:
Example:

(-4) x 7 = (-28)
(-8) x (-5) = 40

or (-4) (7) = (-28)


or (-8) (-5) = 40

or (-4) 7 = (-28)
or (-8) (-5) = 40

B. Division. The quotient of two integers with like signs is a positive while the quotient of two
integers with unlike signs is negative.
Example:

(-72) (-8) = 9

Example:

(-123) 3 = - 41

C. Addition. The sum of two integers with like signs is the sum of their absolute values with the
common sign prefixed before it.
The sum of two integers with unlike signs is the difference of their absolute values with the
sign of the integer with the larger absolute value prefixed before the difference.
Example: (-3) + (-23) = (-26)
Example: (-34) + 12 = (-22)
D. Subtraction. Express subtraction statements as addition statements and follow the procedure
in addition. (That is, change the sign of the subtrahend to its opposite, and proceed to addition.)
Example: (-12) (-3) = (-12) + 3 = -9
Exercise: What number should
a) be added to (-12) to yield 26?
b) be subtracted from (-2) to yield 5?
c) be multiplied by (-4) to yield (-36)?
d) be divided by (-2) to yield 30?
E. P-E-MDAS. P-E-MDAS stands for Parenthesis-Exponent-Multiplication Division Addition
Subtraction.
When two or more operations are involved in a single expression, operations are performed
in the order of P-E-MDAS. That is, we perform first the operation inside the parenthesis (or any
grouping symbol), then followed by determining the power of the number which is raised to a
given exponent, then followed by multiplication/division, and lastly by the addition/subtraction.
Should there be multiplication and division only, perform the operation from left to right.
Should there be addition and subtraction only, perform the operation from left to right.
Example:

Simplify 20 + 100 ( 5 63 32 + 12)

Solution:

20 + 100 ( 5 63 32 + 12)
= 20 + 100 ( 5 63 9 + 12)
= 20 + 100 ( 5 7+ 12)
= 20 + 100 ( (2) + 12)
= 20 + 100 10
= 20 + 10
= 30.

St. Louis Review Center-Inc-Davao Tel. no. (082) 224-2515

54

1. Two bells ring at 5 P.M. For the rest of the day, one bell rings every half hour whereas the other
rings every 45 minutes. When is the first time, on that same day, that both bells ring at the same
time again?
a. 6:30 P.M.
b. 8:30 P.M.
c. 8:45 P.M.
d. 9:00 P.M.
2. Which is true?
a. The set of prime factors of 6 is {1,2,3}
numbers.
b. The product of irrational and rational is irrational.

c. All prime numbers are odd


d. 3.14 is a rational number.

3. Which of the two-digit numbers below when inserted in the blank will make 38__09 divisible by
3?
a. 98
b. 84
c. 34
d. 60
4. Which of the following number is divisible by 45?
a. 300,000,000,450 b.600,000,000,045 c. 100,200,600,090

d. 400,450,000,000

5. On its anniversary, a certain store offers a free sandwich for every 4 th customer and a free
softdrink for every 6th customer. After 75 customers, how many had received both free sandwich
and softdrink?
a. 30
b. 18
c. 12
d. 6
IV. FRACTIONS
Kinds of Fractions
As to relation between the numerator and the denominator
a. Proper the numerator is less than the denominator
b. Improper the numerator is equal to or greater than the denominator
As to relation of the denominators of two or more fractions

1 3 4
, ,
5 5 5
a. Similar the denominators are equal. Examples:

3 5 5
, ,
4 8 6
b. Dissimilar the denominators are not equal. Examples:
Other classes

3 6 12
, ,
4 8 14
a. Equivalent fractions having the same value. Examples:

3
5
2 , 5 .
4
8
b.
Rules
a.
b.
c.

Mixed composed of a whole number and a proper fraction . Examples:


involving Zero
Zero numerator and non-zero denominator the value is zero
Zero denominator no value, undefined
Zero value the numerator is zero

A. Multiplication of Fractions. Multiply numerator by numerator and denominator by


denominator to get the numerator and denominator respectively of the product

12 5 60

25 6 150
Example:

2
5
or

B. Division of Fractions. Multiply the supposed dividend by the reciprocal of the supposed
divisor.

36 6
36 10 360

25 10 25 6 150
Example:

12
5
=

2
or

2
5
.

1 2
2 3
4 5
Exercises: Evaluate the following.

a)

19
3
5
21 20

b)

5
1
3 1
15 6

1
7
2 1
3 18
c)
St. Louis Review Center-Inc-Davao Tel. no. (082) 224-2515

d)
55

D. Changing Dissimilar Fractions to Similar Fractions. Determine the LCM of the


denominators. Then with the said LCM as the denominator, express each fraction to its equivalent.

1
3
Example:

Express

1 4

3 4

3
4
,

5
6
,

4
12

Solution:

to similar fractions.

3 3

4 3
,

9
12

5 2

6 2

and

10
12
=

4 9
,
12 12
Therefore, the similar fractions are

10
.
12
and

E. Addition of fractions. Convert the fractions to similar fractions. Then add the numerators to
obtain the numerator of the sum and copy the denominator.

1
3

3
4

Example:

Evaluate

Solution:

The LCM is 12, so convert the addends to similar fractions with 24 as the
denominator.

1
3

3
4
+

5
6
+

5
6
+

4 9

12 12
=

10 23

12 12
+

11
.
12

or

F. Subtraction of Fractions. Convert the fractions to similar fractions. Then subtract the
numerators to obtain the numerator of the difference and copy the denominator.

7
12
Example: What number should be subtracted from

1
2
to obtain

Solution: Let the desired number be x. Then, the equation is given by

7
12

1
2
-x=

7
12
x=

1
2
-

7
6

12 12
=

1
12
=

G. Fraction as Part of a Whole

3
4
Example: What is
of 28?
Solution: Let the desired number be m. Then, the equation is given by

3
3
28
84
28

21
4
4
1
4
M =

Example: What part of 24 is 4?


Solution: Let the desired number be q. Then, the equation is given by
q 24 = 4

4
1
or
24
6

q=
.
H. Simplifying Fractions
A fraction is in simplest form if the numerator and the denominator are relatively prime
(their GCF is 1). Thus, to simplify fractions, multiply by the fraction whose numerator and
denominator are the reciprocal of the GCF of the numerator and the denominator of the given
fraction.

St. Louis Review Center-Inc-Davao Tel. no. (082) 224-2515

56

12
18
Example: The simplest form of

2
3
is

because

12 2 6 2

18 3 6 3

I. Ordering Fractions
Two fractions are equivalent if their cross products are equal. Otherwise, that fraction the
numerator of which was used to get the greater of the two cross products is the larger fraction.
Exercises
1. A 100-m wire is cut into two parts so that one part is of the other. How long is the shorter
piece of wire?
a. 120m
b. 80m
c. 25m
d.
20m
2. Luis left pan of a cake on the table. Dada ate of it. What fraction of cake was left?
a. 1/8
b. 3/8
c.
d.
n
26

21
39

3. If

and
a. 13

are equivalent fractions, what is the value of n?


b. 14
c. 20

5
8

1
5

d. 21

4. Mr. dela Cruz owned


of a business. He sold
of his share in the business at a cost of P1M.
What is the total cost of the business?
a. P 6M
b. P7M
c. P 8M
d. P 9M
5. Arrange the fractions 5/8, 4/5, 3/4 in increasing order.
a. 5/8, 4/5, 3/4
c. 3/4, 4/5, 5/8
b. 4/5, 3/4, 5/8
d. 4/5, 5/8, 3/4
6. Which of these fractions has the largest value?
a. 3/5
b. 11/16
c. 7/10
d. 5/8
7. Mark spent his monthly salary as follows: 3/5 for food and allowances, 1/3 for his childs
education and house rental. If his monthly salary is P15, 000, how much would he left at the end
of the month?
a. P 1,000
b. P2,000
c. P5,000
d. P 14,000
8. Chedy and Dada run for President for their organization. Chedy got 1/3 of the votes. If Dada got
300 votes, how many students voted for Chedy?
a. 900
b. 200
c. 150
d. 100
V. DECIMAL NUMBERS
A. The Decimal Numbers and the place value chart

100 000

10 000

1 000 100

10

0.1

0.01

Hundred Thousands

Ten Thousandths

Thousandths

Hundredths

Tenths

Ones

Tens

Hundreds

Thousands

Ten Thousand

Hundred Thousand

The place value chart

0.001 0.0001

0.00001

The number 0.8 is read as eight tenths and .214 as two hundred fourteen thousandths.
The number 0.8 is equal to .800.
The number 0.8 is greater than 0.214.
Exercise:
a) Arrange the following decimal numbers in ascending order:
0.5, 0.343, 0.142, 0.5254
b) In 2.3456, what digit is in the thousandths place?
B. Addition and Subtraction of Decimal Numbers. Addition of decimals is done by writing
them in a column so that their decimal points are aligned. Thus aligned, digits with the same place
values would be in the same column, and the addends (or the minuend and the subtrahend) are
added (or subtracted) as if they were whole numbers.
St. Louis Review Center-Inc-Davao Tel. no. (082) 224-2515

57

C. Multiplication of Decimal Numbers. To multiply decimals, multiply the numbers as if they


were whole and so place the decimal point in the result as to have as many decimal places in it as
there are in the factors combined.
D. Division of Decimal Numbers.
To divide a
i. decimal by a whole number, do as in dividing whole numbers but writing the decimal point
directly above that of the dividend.
ii. number by a decimal, multiply both dividend and divisor by that power of ten such that the
divisor becomes the least whole number, and then proceed as in (i) above.
VI. CONVERSION
A. Fraction to Decimal. Divide the numerator by the denominator.
Exercises: Convert the following to decimal:
a) 3/5 = ______
b) 5/6 = ______

c) 7/8 = ______

B. Decimal to fraction
a) Terminating multiply the number by a fraction (equal to one) whose numerator and
denominator is a multiple of 10 such that the numerator of the product is a whole number.
Example:

Convert 0.15 to fraction.

0.15

Solution:

15
3

100
20

100
100
=

b) Repeating decimal number

0.5
Example:

Convert

to fraction

0.5555555.

0.5
Solution:

Let n =

5.5555555.
10 n =

0.5555555.
n=
----------------------------------9n= 5

5
9
n=

5
9

0.5
Hence,

is equal to

Exercises
1. Jeepney fares are computed as follows: P7.50 for the four kilometers plus P0.50 for every
additional kilometer thereof. How much should Au pay for a ride that covers 10 kilometers?
a. P8.00
b. P9.50
c. P10.00
d. P10.50
2. Which of the following is 0.3 of ?
a.
b.
c.
d.
3. Which of the following is between 3 and 4?

15
2

a.
b. -3.5
c.
d.
4. Evaluate 14.8 + 3.95 + .003.
a. 5433
b. 753
c. 446
d. 18.753
5. Carmen bought 4 kilograms of rice at P31.45 per kilo and 6 kilograms of salt at P22.35 per kilo.
If she gave a P1000 bill to the cashier, how much change did she get?
a. P8.00
b. P9.50
c. P120.10
d. P740.10
St. Louis Review Center-Inc-Davao Tel. no. (082) 224-2515

58

6. Each capsule of a certain commercial vitamins contains 0.6 mg of calcium. In how many pieces
of capsules can 22.2 mg of calcium be distributed?
a. 8
b. 37
c. 50
d. 105
7. Which of the following is equal to 2.4545454545 ?

5
11

5
11

a.

44
45

b. 2

1
100

44
45

c.

1
1000

d. 2

1
25

8. The expression
+
+
is equal to _____________.
a. 0.0051
b. 0.006
c. 0.51
9. Which of these numbers is greater than ?
a. .04
b. (1/2)2
c. 1/8

d. 0.051
d. 1/0.04

VII. PERCENT
Per Cent literally meaning per hundred, it is one way of writing fractions in which the
denominator which is required to be 100 is written as %, and read as per cent.

3 75 1

4 100 4
Since 1 = 100% hence

= 75%

A. CONVERSION
Percent to Decimal Number. Divide the number by 100%. Note that 100% = 1.
Example: Convert the following to decimal:
a) 35%
c) 8.37%

1
4
b) 6
Solution:

1
4
%

d)

a) (35%) 100% = 0.35

b) (6 %) 100%
(6.25%) 100% = .0625
c) (8.37) 100% = .0837
d) (1/4%) 100%

(0.25%) 100% = .0025

Decimal Numbers to Percent. Multiply the decimal number by 100%. Note that 100% = 1.
Example:

Convert the following to percent.


a) 0.25
b) 0.143

Solution:

a) 0.25 x 100% = 25%


c) 2.03 x 100% = 203%

c) 2.03

d) .005

b) 0.143 x 100% = 14.3%


d) .005 x 100% = 0.5%

Exercises: Fill in the blanks so that the entries in each row are equal.
Fraction
Decimal
Percent
A
4/7
B
160%
C
0.95
D
6/11
E
%
B. Percentage. Percentage is a percent of a given number. The given number is called the base.
The percent is called the rate.

Formula: Percentage (P) = Base (B) x Rate (R)

Base

Percentage
Rate

Rate
and

Example:
Solution:

What is 50% of 140?


N
= 0.50 x 140 = 70.00 = 70

Example:

36 is 10% of what number?

Percentage
100%
Base

St. Louis Review Center-Inc-Davao Tel. no. (082) 224-2515

59

Solution:

36 = 0.10

Example:

22 is what percent of 88?

Solution:

22 = P

36
360
0.10

88

22
0.25 100% 25%
88

Example:
Mr. Ballaran receives a 10% increase in his salary. With the increase, he now
receives P22, 000. How much is his monthly salary before the increase?
Solution:
present salary = previous salary + increase
22, 000 = previous salary + (10% of previous salary)
22,000= previous salary (1 + .10)
22 000 1.1 = previous salary
20,000 = previous salary
C. Discount
The discussion on DISCOUNT is very similar with
Original Price/ Marked Price/ List price
Rate of Discount
Discount
Selling Price

the discussion on PERCENTAGE.


- as the Base
- as the Rate
- as the Percentage
- Original Price minus Discount

Example:
price.

A skirt with an original price of P250 is being sold at 40% discount. Find its selling

Solution:

S.P.

= Original price - Discount


=
250
(0.40 x 250)
=
250 - 100
=
150

Example:
An item has a selling price of P 210.00. If the selling price is 70% of the original
price, what is its original price?
Solution:
Selling price is 70% of the original price
210 = 0.70 O.P.

Therefore, the original price is P300.


Example:
discount.
Solution:

O.P. = 210 0.70 = 300.

A shirt is being sold at P 199.95. If its original price is P 430, find the rate of
Discount = O.P. S.P.
= 430 199.95 = 230.05

230.05
100% 53.5%
430
Rate of Discount =
D. Simple Interest
Interest (I) is the amount paid for the use of money or the money earned for depositing
the money.
Principal (P) is the money that is borrowed or deposited.
Time (t) is the number of days/months/years for which the money is being

I = Prt,

borrowed/deposited and interest is calculated.

P=

I
rt

t=

I
Pr

r=

Example:
Solution:

Give the simple interest of P10,000 for three years at 5.5% per year.
I=Prt
I = (10 000) (.055) (3)
I = P1 100.

Example:
months is

Determine the amount of the principal if the interest at 10% per annum after 8
P3,600.

St. Louis Review Center-Inc-Davao Tel. no. (082) 224-2515

60

P
Solution:

I
rt

I=Prt

3600
(0.10)(8 / 12)
=

= 45,000

Compound Interest ( Final Amount = P[ 1 + r ]n )

Compound interest is different from simple interest because after the first interest calculation, the interest is added
to the principal, so interest is earned on previous interest in addition to the principal. Compound Interest rates may be
given as annual (1 time a year), semiannual (2 times a year), quarterly (4 times a year), monthly (12 times a year), and daily
(365 times a year).
Example: If P500is invested at 8% compounded semiannually, what will the final amount be after three years?
Final Amount

= P[ 1 + r ]n = 500[ 1 + (8% / 2)]3 * 2 = 500[ 1 + 0.04 ]6


= 500[1.27]
= 635

Exercises
1. John bought a jacket for Php 850.00. If he was given a discount of 15%, what was the original
price?
a. P8,500.00
b. P1,000.00
c. P900.00
d.P765.00
2. In a basket, there are 15 santol, 12 balimbing, and 3 durian. What percent of the fruits are
durian?
a. 10%
b. 12.5%
c. 12%
d. 15%
3. A certain mobile phone model was sold for P4,000 in 2000. Two years later, the same mobile
phone model sold for P2,800. What was the percent decrease of the price?
a. 15%
b. 30%
c. 20%
d.
35%
4. If is 50% of a larger figure, which of the following is the larger figure?
a.
b.
c.
d.
5. A senior class of 50 girls and 70 boys sponsored a dance. If 40% of the girls and 50 % of the
boys attended the dance, approximately what percent attended?
a 44
b. 46
c. 42
d. 40

1
2 %
2
6. Which of the following is equal to

5
2
a. 2.5
b. 0.25
c.
d. 0.025
7. Sarahs earning P 9,200 a month will receive a 15% increase next month. How much
will her new salary be?
a. P 10,500
b. P 10,530
c. P 10,580
d. P 10,560
8. How much is 37% of 80% of 24?
a. 7.1
b. 1.92
c. 19.2
d. 71
9. According to the latest survey, 60% of the cancer patients were smokers. If there were 180
smoking cancer patients, how many cancer patients are there in all?
a. 90
b. 108
c. 240
d. 300
10. Which of the following is 70% of 50?
a. 7
b. 17.5
c. 35
d. 71
11. Twenty four is 12% of what number?
a. 40
b. 150
c. 200
d. 400
12. Thirty six is what percent of 90?
a. 32.4%
b. 40%
c. 45%
d. 76%
13. In a mathematics test of 40 items, Mavic got 90%. How many items did Mavic get?
a. 7
b. 28
c. 36
d. 360
14. Mr. Mabini receives a 10% increase in his salary. With the increase, he now receives P13,200.
How much is his monthly salary before the increase?
a. P12 000
b. P 13, 500
c. 14, 100
d. P14, 520
15. According to the latest survey, 60% of the cancer patients were smokers. If there were 180
smoking cancer patients, how many cancer patients are there in all?
a. 70
b. 150
c. 300
d. 360

VIII. RATIO AND PROPORTION

A ratio is a comparison of two or more quantities.

A proportion is a number sentence stating the equivalence of two ratios.


St. Louis Review Center-Inc-Davao Tel. no. (082) 224-2515

61

Note that in ratio, we are comparing quantities of the same units and that the ratio is
expressed in terms of integers.
Examples:
a) The ratio of 12 days to 3 weeks is 12:21 or 4:7.
b) The ratio of 3 meters to 180 cm is 300:180 or 5:3.
c) The ratio of 2 hours to 25 minutes is 120:25 or 24:5.
d) The ratio of 1 to 4 is 1:3.

A. Direct Proportion. As one quantity increases, the other increases also.


Example:
Solution:
Thus,

Find the value of x if 15:20 = 14 : x.


Equate the product of the means and the product of the extremes. Then solve for x.
(15) (n) = (20) (14)

2014

18

18.6

15
=

or

2
3
.

Example:

A car travels at an average rate of 260 km in 5 hours. How far can it go in 8 hours,
if traveling at the same rate?

Solution:

260 : 5 = x : 8

(5) x = (260) (8)

260 8

= 416.
Example: If the ratio of teachers to students in a school is 1 to 18 and there are 360 students, how
many teachers are there?
Solution: Let x be the number of teachers,

1
x

18 360
or 1 : 18 = x : 360
(18)x = (360)1
x = 360/18 = 20 teachers
B. Inverse Proportion. As one quantity increases, the other decreases.
Example:
If the food is sufficient to feed 10 flood victims in 15 days, how many days would it
last for 8 flood victims?
Solution:

Equate the product of the terms in the first condition to the product of the terms in
the second condition. Thus, we have:
(10 victims)(15 days) = (x) (8 victims)

1015
8
= 18.75 days

C. Partitive Proportion. One quantity is being partitioned into different proportions.


Example:

A wood 120 m long is cut in the ratio 2:3:5. Determine the measure of each part.

2 120
24

=
= 24
10 1
1
Solution:

3 120
36

=
= 36
10 1
1
m

St. Louis Review Center-Inc-Davao Tel. no. (082) 224-2515

62

5 120
60

=
= 60
10 1
1
m
Example: A wire is cut into three equal parts. The resulting segments are then cut into 4, 6, and 8
equal parts respectively. If each of the resulting segments has an integer length, what is the
minimum length of the wire?
A) 24

B) 48

C) 72

D) 96

Solution: Each third of the wire is cut into 4,6 and 8 parts respectively, and all the resulting
segments have integer lengths. This means that each third of the wire has a length that is evenly
divisible by 4, 6, and 8. The smallest positive integer that is divisible by 4, 6, and 8 is 24, so each
third of the wire has a minimum length of 24. So, the minimum length of the whole wire is three
times 24, or 72.

Exercises
1. A 300 m ribbon is cut into four pieces in the ratio 1:2:3:4. Give the length of the shortest
piece.2. If there are 18 boys and 45 girls in the gym, what is the ratio of the girls to the boys?
a. 2:5
b. 2:3
c. 5:2
d. 3:7
2. What one number can replace x in 2: x = x: 32?
a. 2
b. 6
c. 4
d. 8
3. If 5 men can do a job in 12 days, how long will it take 10 men to complete this task, assuming
that they work at the same rate?
a. 20 days
b. 6 days
c. 2 days
d. 0.06 day
4. If 3 kg of oranges cost as much as 5 kg of chicos, how many kg of oranges would cost as much
as 60 kg of chicos?
A. 100
B. 36
C. 7.5
D. 4
5. If 2/5 mm in a map represents 120 km, how many km will be represented by 2 mm?
A. 600 km
B. 300 km
C. 96 km
D. 24 km
6. In a Mathematics Club, the ratio of boys to girls is 3:5. If there are 240 members, how many are
girls?
A. 90
B. 144
C. 150
D. 450
7. A photographer wishes to enlarge a picture 18 cm long and 12 cm wide so that it will be
36 cm wide. How long will the enlarged picture be?
A. 54 cm
B. 72 cm
C. 24 cm
D. 6 cm
8. If 8 secretaries can type 800 pages in 5 hours, how long would it take for 12 secretaries to
type 800 pages at the same rate?
A. 7 1/2 hours
B. 3 1/3 hours
C. 10 hours
D. 2 1/2 hours
THE THEORY OF CONGRUENCES

If a and b are integers, m a positive integer and m (a b), we say that a is


congruent to b modulo m. In symbols, we write this as a b (mod m). CONGRUENCE
was invented by Karl Friedrich Gauss at the beginning of the 19 th century and is a
convenient statement about divisibility.
The following

are equivalent and may be used interchangeably.


a b (mod m).
m (a b) or (a b) is divisible by m.
a = b + mk, k Z.

Theorem: If a and b are integers and m a positive integer then a b (mod m) if and only if a and b
leave the same remainder upon division by m.

{ a , a ,..., a }
1

Let m be a positive integer. A collection of m integers


is called a complete residue
system modulo m if every integer b modulo m is congruent to exactly one of the elements in the
collection.

Properties of Congruence
Congruence is an equivalence relation in the set of integers; that is, congruence is reflexive,
symmetric and transitive with respect to integers.
In the following, let a, b, c, and d be integers and m a positive integer.
St. Louis Review Center-Inc-Davao Tel. no. (082) 224-2515

63

If a b (mod m) then
a+c b+c (mod m).
ac bc (mod m).
ar br (mod m) where r is a positive integer.
If a b (mod m) and c d (mod m), then
a + c (b + d) (mod m).

ac (bd) (mod m).

If ac bc (mod m), then

a b mod
d

where d = (c, m).

The following are some applications of congruence.


a) Finding the units digit (or hundreds digits) of a very large number written in
exponential form; and
b) Finding the remainder when a very large number is divided by another number.
Two of the most prolific mathematicians in Number Theory are Pierre de Fermat and
Leonhard Euler.
FERMATS AND EULERS THEOREMS
Theorem 5. (Fermats Little Theorem) Let p be a prime number and a Z . If p does not divide a,
then
ap 1 1 (mod p) .
Theorem 6. (Fermats Second Theorem). Let p be a prime number and a Z . If p and a are
relatively prime, then
ap a (mod p) .

(m)
DEFINITION OF
Let m be a positive integer greater than 1. The number of positive integers less than and

relatively prime to m is the value of Eulers totient or

(m)

function at m and is denoted by

( p)
Remarks: If p is prime, then

= p 1.

Theorem 7. Eulers Theorem: If n is a positive integer and the greatest common divisor of a and
n is 1, then

a n 1 mod n

.
LINEAR DIOPHANTINE EQUATIONS
An equation in one or more unknowns having integral solutions is called a Diophantine
equation, in honor of Diophantus of Alexandria.
Theorem 8. Given two integers a and b where (a , b) = d. The linear Diophantine equation
ax + by = c has an integral solution if and only if d c.
Theorem 9. If the equation ax + by = c has a solution x = x0 , y = y0,
can be expressed in the form

x x0

b
t
d
,

y y0

then any other solutions

t Z

and

a
t
d

,
t Z.
Example: To determine the integral solution of 24x + 138y = 18, we note that since (24,138) = 6
and 6 18. Then we know that it has solution. We now have the following.
138 = 5 (24) + 18
24 = 1 (18) + 6
18 = 6 (3).
Observe that,
St. Louis Review Center-Inc-Davao Tel. no. (082) 224-2515

64

= 24 1(18)
= 24 [138 - 5 (24)]
= (-1)(138) + 6(24)

Moreover,
18 = 3(6)
= 3[(-1)(138) + 6(24)]
=(-3)(138) +(18)(24)
Thus,

y0 = -3

and

x0 = 18

Hence, the solution of the equation is of the form y = -3+23t and x = 18 4t where t is an
integer.
There are problems which can be solved using linear Diophantine equations as working
equations.
The following steps may be used in solving word problems which involve linear Diophantine
equations in two unknowns/variables:
Step 1. Represent the unknown values using any two variables.
Step 2. Form the equation using the condition given in the problem.
Step 3. Solve the resulting linear Diophantine equation.
Step 4. Determine the solution/s to the problem using the results in step 3.

1 mod p
Theorem: (Wilsons Theorem) If p is a prime, then (p-1)!

Exercises
1. Which of the following is true?

54 5 mod 25

34 3 mod 5

A.

B.

15 9 mod 6

308 0 mod 11
C.

3 4 mod 15
D.

2. Mavic argues that


. Is she correct? Why?
A. Yes, because 6 divides 15 - 9.
C. No, because 6 does not divide 15 + 9.

15 9 61

B. Yes, because 6 divides 15 + 9.


D. No, because
3. Which of the following is congruent to 11 modulo 13?
A. -7
B. -5
C. -2

n 6 1 0 mod 7

.
D. 4

4. Which of the following must be the value of n if


?
A. x is divisible by 7
C. x is relatively prime with 7
B. x is prime
D. x is any integer greater than 7

5105
5. What is the remainder when
A. 1
B. 2

is divided by 3?
C. 3

D. 5

6. What is the units digit of


?
A. 1
B. 3

C. 5

D. 9

7. What is
A. 1

C.6

D. 11

100

12

?
B.4

SET A Exercises. Choose the letter of the best answer.


1. What is the sum of the first four prime numbers?
a. 11
b. 26
c. 17
d. 28
2. Which of the following is NOT true about the sum of two consecutive positive odd integers?
a. it is even
b. it is only divisible by 12
c. it is divisible by 4
d. it is always divisible by 1
3. In a sequence of starts and stops, an elevator travels from the first floor to the fourth floor and
then to the second floor. From there, the elevator travels to the third floor and then to the first
floor. If the floors are 3 meters apart, how far has the elevator traveled?
St. Louis Review Center-Inc-Davao Tel. no. (082) 224-2515

65

a. 21 m
b. 24 m
c. 28 m
d. 32
m
4. An orange light blinks every 4 seconds. A blue one blinks every 5 seconds while a red one
blinks every 6 seconds. How many times will they blink together in two hours?
a. once
b. 2 times
c. 10 times
d. 60 times
5. If one prime factor of 84 is 3, what are the other prime factors?

a. 2 and 3

b. 2 and 7

c. 3 and 5

d. 4 and 7

6. A television show reports the following temperature for 5 cities:


Beijing
London
Chicago
Philippines
Moscow
2 0C
-6 0C
0 0C
300C
-9 0C
Which city is the coldest?
a. Beijing
b. Chicago
c. London
d. Moscow
7. If the sum of a certain number and 7 is divided by 4, the quotient is 3. What is the number?
a. 5
b. 12
c. 15
d. 18
8. Which of the following numbers has the largest value?
a. 22
b. 10
c. 75
d.3
9. Which of the following numbers has the least value?
a. 22
b. 10
c. 75
d.3
10. What is the difference in the elevation between the top of a mountain 51 meters above sea
level and a location 28 meters below sea level.
a. 23 m
b. 33 m
c. 79 m
d. 89 m
11. A pack of P50-bills is numbered from RV628 to RV663. What is the total value of the pack of
bills, in pesos?
a. 35
b. 36
c. 1750
d. 1800
12. Simplify: [ 5 81 32 5 3 + 2] (42 23)
a. 15
b. 4
c. -6
d. 15 1/4

5
13. If each container contains
b. 68 kg
kg

2
3
kg of flour, how many kg of flour are there in 12 container?
b. 70kg
c. 72 kg
d. 80

14. Eighteen is 2/3 of what number?


a. 6
b. 12
c. 6
d. 27
15. What part of an hour has passed from 2:48 am to 3:20 am?
a. 7/8
b. 1/3
c. 8/15
d. 8/25
16. Clarita spent one-sixth of her money in one store. In the next store, she spent three times as
much as she spent in the first store. If she had 80 pesos left, how much money did she have from
the start?
a. 240 pesos
b. 252 pesos
c. 300 pesos
d. 360 pesos
17. Philip has obtained the following grades: 1.4, 1.7, 1.8 and 2.5. What must be his fifth grade so
that his average is 1.7?
a. 2.1
b. 1.9
c. 1.5
d. 1.1
18. Out of the 20 numbers, 6 were 2.5s, 4 were 3.25s and the rest were 2.2s. Give the
arithmetic mean of the numbers.
a. 2.5
b. 2.65
c. 10
d. 22
19. Ron bought X number of notebooks at P23.00 each, Y pad papers at 18.45 each, and Z
ballpens at P8.25 each. If he gave an amount of P1000 to the cashier, how much change did he
receive?
a. P 434.25
c. 1000 [(23.00)(X) + (18.45) (Y) + (8.25) (Z)]
b. P 334.25
d. none of these
20. A bag has a selling price of P60.00. If the selling price is 75% of the original price, what is its original price?
a. P80
b. P120
c. P200
d. P280
21. Mr. de Borja, a store owner, advertises a polo-shirt originally sold for P200 for P170 only. What
rate of discount is he giving?
a. P 30
b. P15
c. 30%
d. 15%
22. Ja bought an article for P400 and sold it for P500. What rate of profit did she enjoy in that
deal?
a. P100
b. 100%
c. 25 %
d. 20%
23. The price of an item is increased by 70% and then offered at 40% discount. What happened
to the original price?
a. There is an increase of 30%.
c. There is an increase of 2 %.
b. There is an increase of 28%.
d. There is a decrease of 32%.
24. How much should Allan invest so that his money earns P2,250 deposited at 6% for 9 months?
a. P 50,000
b. 37,500
c. P 135
d. P 101.25
25. Dan sells a real estate. He receives a monthly salary of P10,000 plus a commission of 1/5 %
of his net sales for that month. Find his gross pay for a month during which his net sale is one
million pesos.
a. P 2,000
b. P 12,000
c. P 200,000
d. P 210,000
26. There are 20 million Filipinos who are qualified voters. If 25% of the population are qualified
voters, how many are not qualified voters?
a. 80 million
b. 60 million
c. 15 million
d. 5 million
27. Three cavans of rice for a family of six members last for 5 weeks. At this rate, how many
weeks will 4 cavans of rice last a family of 8 members?
St. Louis Review Center-Inc-Davao Tel. no. (082) 224-2515

66

A. 4
B. 5
C. 5 1/3
D. 6
28. If the assembly, ratio of boys to girls is 1:4. What percent of the assembly are the boys?
A. 10%
B. 20%
C. 25 %
D. 80%

1130
29. What is the remainder when
A. 1
B. 2

is divided by 31?
C. 10

D. 101

2061

30. Which of the following is the remainder when


is divided by 7?
A. 1
B. 2
C. 3
D. 6
31. If y is the remainder when 47 is divided by 6, what is the remainder when 19 is divided by y?
A. 1
B. 2
C. 4
D. 5

3 z 1 mod 13

32. Which of the following is a value of z such that the congruence


is NOT true?
A. 9
B. 22
C. 30
D. 48
33. What is the remainder when 18! + 2 is divided by 19?
A. 0
B. 1
C.3
D. 97
34. A certain number of sixes and nines are added to give a sum of 126. If the numbers of sixes
and nines are interchanged, the new sum is 114. How many of each were there after the switch?
A. Ten sixes and 6 nines
B. Four sixes and Twelve nines
C. Seven sixes and nine nines
D. Six sixes and Ten nines

SET B Exercises. Choose the letter of the best answer.


1. How many prime numbers are less than 37?
a. 9
b. 10
c. 11
d. 12
2. In a series of card games. Marlon starts out with P200 and wins a total of P450. If he later loses
P350, wins P60 and loses P150, how much cash does Marlon have?
a. 0
b. P150
c. P210
d. P300
3. Your score in a game is -6. How many points must you earn to get a score of 10?
a. -6
b 15
c. 16
D. 22

4. Arrange the fractions 5/12, 3/7, 2/5 in decreasing order.


a. 2/5, 5/12, 3/7
c. 3/7, 5/12, 2/5
b. 5/12, 2/5, 3/7
d. 2/5, 3/7, 5/ 12

11

1
5

5. Edwin, Doms and Lon weigh 45 kg. If Edwin and Doms weigh
respectively, what is the weight of Lon in kilograms?

20
a.

1
20

24

19
20

b.

42

9
20

c.

13

3
4

kg and

kg,

19
20

69
d.

3
4
4
6. Which of the following should be multiplied to
so that the product is 57?
a. 6
b. 12
c. 6
d. 75

15 1 mod m

7. Which of the following is a value of m if


A. 2
B. 3

C. 23

D. 32

1
3
3
8. Alex works on his assignment
hours a day, what part of the day does he spend doing his
assignment?
a. 1/8
b. 1/7
c. 5/36
d.
15/28
9. What value of p will satisfy the equation 0.2 (2p + 1470) = p?
a. 294
b. 490
c. 560
d. 1470
10. A blouse originally priced at P600 is being sold at a discount of 30%. How much would you
pay if you buy that blouse?
a. P 30
b. P180
c. P 420
d. P 570
11. A pair of slippers with a selling price of P120 is sold at 40% discount. What is its original price?
a. P 48
b. P72
c. P 200
d. P 300
12. An item is offered at 20% discount. Later, it is offered at 30% discount. If the new selling price
is P112, what is the first original price?
a. P162
b. P200
c. P224
d. P1866.67
13. To have a 25% profit, the vendor should sell the item at P80.00. How much is his profit?
a. P20.00
b. P60.00
c. P16.00
d. P64.00

14
14. Minda deposited P50,000 in a bank that pays a simple annual interest of
money will she have in the bank after five years.
St. Louis Review Center-Inc-Davao Tel. no. (082) 224-2515

1
5
%. How much
67

a. P 85,500
b. P35,500
c. P36,250
d. P86,250
15. The ratio of cows and carabaos in the field is 4:9. If there are 468 cows and carabaos in the
field, how many are carabaos?
A. 52
B. 117
C. 144
D. 324
16. In the class, the ratio of boys to girls is 6:5. If there are 90 girls, how many persons are in the
class?
A. 75
B. 108
C. 165
D. 198
17. Edwin painting a wall at 9:00 a.m. and was able to finish painting 3/5 of it at 10:30 a.m.
Continuing at this rate, at what time will he finish?
A. 10:45 a.m.
B. 11:30 a.m.
C. 11:45 a.m. D. 12:15 a.m.

4 200
18. What is the remainder when
is divided by 15?
A. -1
B. 1
C. 2
19. If x is any positive integer, then 23x + 1 is _______ divisible by 8.
A. Always
B. Never
C. Sometimes

D. 4
D. Equivalently

29

20. What is
A. 0

?
B. 1

C. 2

2 1

D. 28

50

21. Which of the following is the remainder when


A. 0
B. 2
C. 4

is divided by 7?
D. 5

x 1 mod 4

22. Which of the following is equivalent to the pair of congruence

x 1 mod 12

A.

x 2 mod 12

B.

x 3 mod 12

a 5 a mod 10

a5
A. a divides

x 5 mod 12

C.
D.
23. Which of the following is NOT true if a is a positive integer?

x 2 mod 3
and

C.

a 1

B.
divides
.
D. a and
have the same units digit
24. Which of the following gives a remainder of 2 when divided by 5 and a remainder of
12 when divided by 13?
A. 22

B.38

C. 77

D. 92

2!4!6!... 2008!2010!

25. What is the remainder upon dividing the sum


A. 1
B. 2
C. 3
26. What is the least residue if 17109 is a multiple of 6?
A. 1
B. 2
C. 3
27. Which of the following has an integral solution?

21x 13 mod 77

A.

3 x 5 mod 36

C.
28. How many integral solutions does
A. 2
B. 3

by 5?
D. 4
D. 5

3 x 4 mod 81

B.

3 x 4 mod 5

D.

18 x 21 mod 24
have?
C. 6

D. 10

29. Which of the following has a solution if the variables are positive integers?
A. 5x + 30y = 18
B. 8x + 10y = 15

101x 37 y 3819

C. 22x + 4y = 28
D.
30. When 16! is divided by 17, the remainder is ______.
A. 0
B. 1
C. 17
D. 18
31. A Johns transcript shows x number of 3-unit courses and y number of 5-unit courses for a total
of 64 units. Which of the following may appear in the transcript?
A. 2 xs and 18 ys
C. 11 xs and 3 ys

B. 13 xs and 5 ys
D. 9 xs and 8 ys

x 2 1 mod 8

32. Which of the following is a value of x if


?
A. 2
B. 5
C. 6
D. 10
33. When students in a certain college are grouped by 2s, 3s, 4s, 5s or 6s at a time, there
remain, 1,2,3,4, or 5 students respectively. When the students are grouped by 7s, no is student
left. What is the smallest possible number of students in the school?
St. Louis Review Center-Inc-Davao Tel. no. (082) 224-2515

68

A. 227

B.1,534

C. 1,379

D. 2,778

LICENSURE EXAMINATION FOR TEACHERS (LET)


Refresher Course
Content Area: MATHEMATICS
Focus: Plane and Solid Geometry
Prepared by: Daisy de Borja-Marcelino
Basic Ideas
The undefined terms, point, line, and plane are geometric ideas and they are visually
represented by a tiny dot, a thin wire, and a smooth flat surface, respectively. Points are labeled
by means of capital letters, lines by naming any two of its points, and planes by naming at least
three of its points. The subsets of a line are ray, segment, and the line itself. Space is the set of all
points.
St. Louis Review Center-Inc-Davao Tel. no. (082) 224-2515

69

A
..

B
.

A
.

B.

Line

A.

Ray

AB

AB

B.
Line segment

AB

Some postulates on points, lines and planes:


The Distance Postulate. To every pair of different points there corresponds a unique positive number.
The Ruler Postulate. The point of a line can be placed in correspondence with the real numbers in such
a way that
(1) to every point of the line there corresponds exactly one real number;
(2) to every real number there corresponds exactly one point of the line; and
(3) the distance between any two points is the absolute value of the difference of the
corresponding numbers.
The Ruler Placement Postulate. Given two points P and Q of a line, the coordinate system can be chosen in
such a way that the coordinate of P is zero and the coordinate of Q is positive.
The Line Postulate. For every two points there is exactly one line that contains both points.
The Plane Postulate. Any three points lie in at least one plane, and any three noncollinear points lie in
exactly one plane.
The Plane Separation Postulate. Given a line and a plane containing it. The points of the plane that
do not lie on the line form two sets such that
(1) each of the sets is convex, and

PQ
(2) if P is in one of the sets and Q is in the other, then the segment

intersects the line.

The Space Separation Postulate. The points of space that do not lie in a given plane form two sets,
such that
(1) each of the sets is convex, and

PQ
(2) if P is in one of the sets and Q is in the other, then the segment
plane.

intersects the

The following are also true.


(a) Every plane contains at least three noncollinear points.
(b) Space contains at least four noncoplanar points.
(c) If two points of a line lie in a plane, then the line lies in the same plane.
(d) If two different planes intersect then, their intersection is a line.
Some theorems on points, lines and planes:

If A,B, and C are three different points of the same line, then exactly one of them is between the
other two.

The Point-Plotting Theorem. Let

AB
AB

be a ray, and let X be a positive number. Then there is

AP x

exactly one point P of


such that
.
Every segment has exactly one mid-point.
If two different lines intersect, their intersection contains only one point.
If a line intersects a plane not containing it, then the intersection contains only one point.
Given a line and a point not on the line, there is exactly one plane containing both.
Given two intersecting lines, there is exactly one plane containing both.
The First Minimum Theorem. The shortest segment joining a point to a line is the perpendicular
segment.
In a given plane, through a given point of a given line, there is one and only one line
perpendicular to the given line.
The Perpendicular Bisector Theorem. The perpendicular bisector of a segment, in a plane, is
the set of all points of the plane that are equidistant from the end points of the segment.
Through a given external point there is at least one line perpendicular to a given line.
Through a given external point there is at most one line perpendicular to a given line.
If M is between A and C on a line L, then M and A are on the same side of any other line that
contains C.
St. Louis Review Center-Inc-Davao Tel. no. (082) 224-2515

70

If B and C are equidistant from P and Q, then every point between B and C is equidistant from p
and Q.
If a line is perpendicular to each of two intersecting lines at their point of intersection, then it is
perpendicular to the plane that contains them.
Through a given point of a given line there passes a plane perpendicular to the given line.
If a line and a plane are perpendicular, then the plane contains every line perpendicular to the
given line at its point of intersection with the given plan.
Through a given point of a given line there is only one plane perpendicular to the line.
The Perpendicular Bisecting Plane Theorem. The perpendicular bisecting plane of a segment is
the set of all points equidistant from the end points of the segment.
Two lines perpendicular to the same plane are coplanar.
Through a given point there passes one and only one plane perpendicular to a given line.
Through a given point there passes one and only one line perpendicular to a given plane.
The Second Minimum Theorem. The shortest segment to a plane from an external point is the
perpendicular segment.
Two parallel lines lie in exactly one plane.
In a plane two lines are parallel if they are both perpendicular to the same line.
Let L be a line and let P be a point on L. Then there is at least one line through P, parallel to L.

Angle. It is a union of two non-collinear rays that have a common endpoint.


An acute angle measures more than 0o but less than 90o.
A right angle measures exactly 90o.
An obtuse angle measures more than 90o but less than 180o.
Two angles are complementary if the sum of their measures is 90o.
Two angles are supplementary if the sum of their measures is 180o.
Two angles are adjacent if they have a common vertex, common side but no common
interior points.
Two angles form a linear pair if they are both adjacent and supplementary.
Some postulates on angles:

The Angle Measurement Postulate. To every angle there corresponds a real number
between 0 and 180.

AB
The Angle Construction Postulate. Let

be a ray on the edge of the half-plane H. for

AP
every number r between 0 and 180 there is exactly one ray

mPAB r

, with P in H, such that

The

BAC
Angle

Addition

mBAC mBAD mDAC

Postulate.

If

is

in

the

interior

of

then

.
The Supplement Postulate. If two angles form a linear pair, then they are supplementary.

The transversal line is a line intersecting two or more coplanar lines at different points
1
3
5
7

2
4

Interior angles: 3, 4, 5, 6
Exterior angles: 1, 2, 7, 8
Corresponding angles: 1 and 5, 2 and 6
3 and 7, 4 and 8

Alternate interior angles: 3 and 6, 4 and 5


Alternate exterior angles: 1 and 8, 2 and 7
Same-side interior angles: 3 and 5, 4 and 6
Some theorems on angles and transversal lines:

Given two lines cut by transversal. If a pair of alternate interior angles are congruent, then the
lines are parallel.
Given two lines cut by transversal. If a pair of corresponding angles are congruent, then a pair
of alternate interior angles are congruent.
Given two lines cut by transversal. If a pair of corresponding angles are congruent, then the
lines are parallel.
St. Louis Review Center-Inc-Davao Tel. no. (082) 224-2515

71

Given two lines cut by transversal. If a pair of interior angles on the same side of the
transversal are supplementary, the lines are parallel.
If two parallel lines cut by transversal, each pair of corresponding angles are congruent.
In a plane, if a line intersects one of two parallel lines in only one point, then it intersects the
other.
Every right angle has measure 90, and every angle with measure 90 is a right angle.
If two angles are complementary, then both are acute.
Any two right angles are congruent.
If two angles are both congruent and supplementary, then each is a right angle.
The Supplement Theorem. Supplements of congruent angles are congruent.
The Complement Theorem. Complements of congruent angles are congruent.
The Vertical Angle Theorem. Vertical angles are congruent.
If two intersecting lines form one right angle, then they form four right angles.
The Angle Bisector Theorem. Every angle has exactly one bisector.
The Isosceles Triangle Theorem. If two sides of a triangle are congruent, then the angles
opposite these sides are congruent.
If two angles of a triangle are congruent, then the sides opposite them are congruent.

BAC

BC

If M is between B and C, and A is any point not on


, then M is in the interior of
.
The Exterior Angle Theorem. An exterior angle of a triangle is greater than each of its remote
interior angles.

Polygon. It is a closed plane figure with three or more sides, consists of segments (sides) that
meet only at their endpoints (vertices) such that every side contains only two vertices, and every
vertex is on exactly two sides.
Name of Polygon
Triangle
Quadrilateral
Pentagon
Hexagon
Heptagon
Octagon

Number
Sides
3
4
5
6
7
8

of

Exact

Name of Polygon
nonagon
decagon
undecagon
dodecagon
pentadecagon

Number
sides
9
10
11
12
15

Notes:
of
the
always
of
the
legs x, y
y2 = z2

is

of

Exact

a) In a triangle, the sum


lengths of two sides is
greater than the length
third side.
b) In a right triangle with
and hypotenuse z: x2 +
(Pythagorean triple)
c) The sum of the interior
angles of an n-gon is (n2) 180o. The sum of the
interior angles of a
triangle is 180o while the
sum of the interior
angles of a quadrilateral
360o.

A polygon is equilateral if the measures of all its sides are equal.

A polygon is equiangular if the measures of all its angles are equal.

A polygon is regular if all its sides and angles are correspondingly congruent.

Some postulates on polygons


The SAS Postulate. Every SAS correspondence is a congruence.
The ASA Postulate. Every ASA correspondence is a congruence.
The SSS Postulate. Every SSS correspondence is a congruence.
The Parallel Postulate. Through a given external point there is only one parallel to a given
line.

The Area Postulate. To every polygonal region there corresponds a unique positive real
number.

The Congruence Postulate. If two triangles are congruent, then the triangular regions
determined by them have the same area.

St. Louis Review Center-Inc-Davao Tel. no. (082) 224-2515

72

The Area Addition Postulate. If two polygonal regions intersect only in edges and vertices
(or do not intersect at all), then the area of their union is the sum of their areas.
The Unit Postulate. The area of a square region is the square of the length of its edge.
The Unit Postulate. The volume of a rectangular parallelepiped is the product of the
altitude and the area of the base.
The Cavalieris Principle. Given two solids and a plane. Suppose that every plane parallel
to the given plane, intersecting one of the two solids, also intersects the other, and gives cross
sections with the same area. Then the two solids have the same volume.
The SAA Theorem. Every SAA correspondence is a congruence.
The Hypotenuse-Leg Theorem. Given a correspondence between two right triangles, if the
hypotenuse and one leg of one of the triangles are congruent to the corresponding parts of the
second triangle, then the correspondence is a congruence.
If two sides of a triangle are not congruent, then the angles opposite them are not
congruent, and the larger angle is opposite the longer side.
If two angles of a triangle are not congruent, then the sides opposite them are not
congruent, and the longer side is opposite the larger angle.
The Triangle Inequality. The sum of the length of any two sides of a triangle is greater than
the length of the third side.
The Hinge Theorem. If two sides of one triangle are congruent, respectively, to two sides
of a second triangle, and the included angle of the first triangle is larger than the included angle
of the second, then the third side of the first triangle is larger than the included angle of the
second.
The Converse Hinge Theorem. If two sides of one triangle are congruent respectively to
two sides of a second triangle, and the third side of the first triangle is longer than the third side
o the second, then the included angle of the first triangle is larger than the included angle of the
second.
For every triangle, the sum of the measures of the angles is 180.
Each diagonal separates a parallelogram into two congruent triangles.
In a parallelogram, any two opposite sides are congruent.
In a parallelogram, any two opposite angles are congruent.
In a parallelogram, any two consecutive angles are supplementary.
The diagonals of a parallelogram bisect each other.

Exercises
1. Which of the following has a definite length?
A) ray
B) line
C) angle
2. A plane is determined by ______________.
i. a line and a point.
iii. any three points.
A) ii only
B) iv only
3. In angle LON, what is the vertex?
A) L
B) O

D) line segment

ii. two intersecting lines.


iv. a line and a point not on it.
C) ii and iii

D) ii and iv

C) N

D) cannot be determined

4. These shapes are arranged in a pattern.


Which of these shapes are arranged in the same pattern?
A)

B)

C)

D)

5. Which of these triangles cannot be isosceles?


A) acute
B) right

C) obtuse

D) scalene

6. Two angles that are complementary _____________________.


i. are each acute.
ii. are congruent.
iii. have equal measures.
iv. have measures that add up to 900.
St. Louis Review Center-Inc-Davao Tel. no. (082) 224-2515

73

A) ii only
B) iv only
C) ii and iii
D) i and iv
7. Refer to the figure on the right. Given: m 1 = 57 and m 3 = 80. What is m 5?
A) 43
B) 63
C) 137
D) 180
8. A quadrilateral MUST be a parallelogram if it has ________________________.
A) one pair of adjacent sides equal
B) one pair of parallel sides
C) two pairs of parallel sides
C) two adjacent angles equal
9. NCTE is a parallelogram. If m N = 67, then m T = _____.
A) 113
B) 90
C) 67
D) 23
10. Which two properties of the rhombus do not hold true for other parallelograms?
A) Diagonals are perpendicular and congruent.
B) Diagonals are congruent and bisect each other.
C) Diagonals are perpendicular and bisect each other.
D) Diagonals are perpendicular and each bisects the angles of the parallelogram.
11. The supplement of an angle is three times its measure. What is the measure of its
supplement?
A) 22.5
B) 45
C) 67.5
D) 135

12. Refer to the figure on the right. If lines r and s are parallel,
which of the following pairs of angles are congruent?
A) 4 and 5

B) 4 and 3

C) 1 and 8

D) 3 and 5

13. The angles of a triangle are in the ratio 2: 3: 5. What is the largest angle?
A) 36
B) 54
C) 90
D) 99
14. An angle measures 65.5. What is the measure of its supplement?
A) 24.5
B) 25. 5
C) 114.5
D) 124.5
15. Which refers to a pair of lines that intersect and form 4 right angles?
A) parallel lines
B) perpendicular lines
C) intersecting lines D) skew lines
16. Which of the following quadrilaterals best describes a square?
A) Its diagonals are perpendicular to each other.
B) It is an equiangular rhombus.
C) Its diagonals are congruent.
D) It has four right angles.
17. If ABC is an isosceles triangle with a right angle at B, then
A)

BC

is the hypotenuse.

B) AC = BC

C) angle BCA measures 45.


D) AB = AC
18. What is the measure of each interior angle of a regular pentagon?
A) m
108
B) 140
C) 180

D) 540

19. What is the sum of the measures of the interior angles of a regular heptagon?
A) 1260
B) 1080
C) 900
D) 112.5

20. In the adjoining figure, if m = 63 and h = 134, then a must be equal to _____.
A) 46
B) 71
C) 109
D) 117
21. Which of the following statements is true about parallel lines?
A) They form a right angle.
B) They do not intersect at all.
C) They are skew.
D) None of these.
22. In the adjoining figure, the quadrilateral is a parallelogram. What is the value of x?
A) 150
B) 75
C) 60
D) 50
23. Which of the following can be lengths of the sides of a triangle?
A) {3, 4, 9}
B) {5, 5, 10}
C) {-12, 6, 8}
D) {3, 18, 20}
24. The measures of the exterior angles of a triangle are in the ratio 2:3:4. What is the measure of
the widest angle?
A) 40
B) 80
C) 160
D) 170
G. Perimeter. The distance around a polygon is called a perimeter. To obtain this, we just add the
length of all the sides of the polygon. In the case of equilateral polygons, we just multiply the
length of one side to the number of sides.
Example:
Solution:

Determine the perimeter of a triangle with sides 7, 12, 18.


P = 7 + 12 + 18 = 37 units.

Exercises:

Complete the table below.

St. Louis Review Center-Inc-Davao Tel. no. (082) 224-2515

74

Regular Polygon
a) square
b)
c) nonagon
d)
e) heptagon

Length of a Side
13 cm
7 cm

Perimeter
56
54
45
28

9 cm

cm
cm
cm
cm

H. Areas of Triangles and Quadrilaterals


a) triangle = (1/2) (base) (height)
b) rectangle = (length) (width) or (base) height)
c) square = (side)(side) or (1/2)(d1)(d2)
d) parallelogram = (base) height)
e) rhombus = (1/2)(d1)(d2)
f)

trapezoid = (1/2) (upper base + lower base)(height)

Example:
Solution:
Example:
base.

Give the area of a square whose perimeter is 100 m?


Since the perimeter of the square is 100 cm, then each side measures 25 m. Hence,
its area is (25 m)2 = 625 m2.
Determine the area of a triangle whose base is 10 mm and its altitude is thrice its

1
bh
2

1
(10) [(3) (10)]
2

150
mm2 .

Solution:

A=

Example:
width by 3.
Solution:

Give the area of a rectangle whose width is 5 cm and its length exceeds 4 times the
A = lw

= [4w + 3] (w)
= [(4)(5) + 3] (5)
= (23) (5) = 115 sq. cm.

Example:
Determine the area of the trapezoid whose bases are 6 cm and 10 m, while the
altitude is 7 m.
A = (1/2) (b1 + b2) (height) = (1/2) (6 + 10) (7) = 56 m2

Solution:

I. Circle. This is a set of points in a plane, equidistant from a fixed point. The fixed point is
called the center, and the fixed distance is the length of the radius.

Circumference is the distance around the circle. Formula: 2 r


Area is the interior region bounded by the circle. Formula: r2
Terms related to a circle:
Center the fixed point
Radius segment (or distance) drawn from the center to any point on the circle
Chord segment whose endpoints are any two points on the circle
Diameter a chord that passes through the center (the longest chord)
Secant a line/ray/segment that cuts the circle in two points
Tangent a line that touches the circle in only one point

Note: The diameter is twice as long as the radius.


Exercises: In the figure on the right, identify the following:
center, radius, chord, diameter, secant, tangent.
Example:
Solution:

Determine the circumference of a circle with radius 10 mm.


C = 2 r = 2 (3.1416) (10 cm) = 62.832 mm

St. Louis Review Center-Inc-Davao Tel. no. (082) 224-2515

75

Example:
Solution:

Give the area of the circle if its circumference is 18 m.


Since the circumference is 18 cm, then its radius is 9 m. Hence,
A = r2 = (3.1416) (9m)2 = 254.4696 or 254.47 m2

Exercises
1. Complete the table below.
Radius
Diameter
1. 2 cm
2.
6 dm
3.
4.
5. 9 m

Circumference (in )

Area (in )

50 m
36 km2

2. A diameter of a circle is also a ________.


A) radius
B) secant
C) chord
D) tangent
3. Which of the following sets of numbers can be sides of a right triangle?

6 , 6 ,2 3

3, 6 ,3 2

A. 4, 5, 6
B.
C.
D. 2, 2, 3
4. The length and width of a rectangle are (3x 2) and (2x + 1). What is its perimeter?
A. 5x-1
B. 10 x - 2
C. 6x2 x -2
D. 6x2 x +2
5. The area of a square is 32x. Which of the following could be the value of x?
A. 2
B. 6
C. 3
D. 4
6. If the area of one circle is twice of another circle, what is the ratio of the area in percent of the
smaller to larger circle?
A. 70%
B. 25%
C. 75%
D. 50%
J. Volume of Solids. It describes how much space a three dimensional figure occupies.
= (side)3

a) cube

b) rectangular prism = (length)(width)(height) or (area of the base)(height)


c) pyramid

= (1/3) (area of the base) (height)

d) sphere

= (4/3)( )(radius)3

e) cylinder

= (radius)2 (height)

f)

= (1/3) (radius)2 (height)

cone

Example:
Solution:

What is the volume of a cube whose edge is 4 mm?


V = (side)3
= (4 mm)3
= 64 mm3

Example:

The length of a rectangular box is 20 cm. Its width exceeds 1/4 of the length by 5
cm, while the height is 7 cm less than 1/2 of the length. What is its volume?

Solution:

Volume = (length)(width)(height)

20cm 5cm
4

= (20 cm)
= (20

20cm 7cm
2

cm) (10 cm) (3 cm) = 600 cm3

Example:

The area of the base of a pyramid is 48 cm2 while the height is 6 cm. What is its
volume?

Solution:

V = (1/3) (area of the base) (height)


= (1/3) (48 cm2) (6 cm ) = 96 cu. cm.

Example:
Solution:

Determine the volume (in ) of a sphere whose diameter is 12 m.


V = (4/3)( )(radius)3 and since diameter is 12 cm, then the radius is 6 m.
= (4/3)( )(6 cm)3 = 288 m3

Example:

A cone has a base diameter of 32 cm and its height is 3 cm less than 3/4 of the
radius. What is its volume?

Solution:

V = (1/3) (radius)2 (height)

1

3
=

(16 cm)2

16cm 3cm
4

St. Louis Review Center-Inc-Davao Tel. no. (082) 224-2515

76

= 768 cm3
K. Lateral and Total Surface Areas of Solids.
Solids
a) cube
b)
rectangular
prism
c) pyramid

Lateral area
4 s2
(perimeter of the base)x height

Total surface area


6s2
2B + ph

1/2 (perimeter of the base) slant height

d) sphere
e) cylinder
f) cone

None
(Circumference of the base) (slant height)
1/2 (Circumference of the base)
(slant
height)

B + 1/2
p (slant
height)
4r2
2r2 + 2r (slant height)
r2 + r (slant height)

Exercise
1. Determine the volume, lateral and surface area of the following: (All units are in cm.) Express
your answers in , if possible.

edge
10cm,

base edge = 10 m
width = 3cm,
height = 4cm

radius = 5

radius = 5
height = 12

Volume (in cm3)

Lateral Area (in cm2)

= 2
length

slant height = 13m


altitude = 12m

radius = 5
slant height = 5
altitude = 4
Total Surface Area (in cm2)

a) cube
b)
rectangular
prism
c) square pyramid
d) sphere
e) cylinder
f) cone
PRACTICE EXERCISES
1. What will happen to the area of a circle if its radius is doubled?
a. The area will be doubled.
c. The area will be quadrupled.
b. The area will remain the same.
d. The area will be reduced to half.

2. What is the reason for your answer in item number 1?


a. The area of the circle varies directly to the radius.
b. The area of the circle varies inversely to the radius.
c. The area of the circle varies directly to the square of its radius.
d. The area of the circle varies inversely to the square of its radius.
3. If a side of square measures 5mm, which of the following is its perimeter?
a. 20mm
b. 25mm
c. 20 mm2
d. 25mm2
2
4. If a square has an area of 144m , which of the following is the measure of its side?
a. 12mm
b. 12cm
c. 1,200 cm
d. 1.2m
St. Louis Review Center-Inc-Davao Tel. no. (082) 224-2515

77

mm
=

5. If the base of a triangle measures 20 centimeters and its height measures 1 inch, which of
the following is its area?
a. 20 cm
b. 25.4 cm
c. 20 cm2
d. 25.4 cm2
6. A string 1mm long is used to form a shape that would give a maximum area. What shape is
it?
a. square
b. rectangle
c. circle
d. parallelogram
7. If the length of the side of a square is reduced to half, what will happen to its perimeter?
a. It will be tripled.
c. It will be doubled.
b. It will remain the same.
d. It will be reduced to half.
8. The area of a new circle is nine times its original area. What made this happen?
a. The radius was tripled.
c. The value of was squared.
b. The radius was multiplied by 9.
d. The radius was reduced to half.
9. A regular polygon is inscribed in a circle. Suppose we increase continuously the number of
its sides, what do you think will happen?
a. The regular polygon with infinitely many sides will never exist.
b. It is impossible for a regular polygon with infinitely many sides to be inscribed in a
circle.
c. The perimeter of the regular polygon will be greater than the circumference of the
circle.
d. The perimeter of the regular polygon will be approximately equal to the
circumference of the circle.
10. Which of the following is the most reasonable weight of an adult woman?
a. 12 lbs
b. 120 lbs
c. 1,200 lbs
d. 12,000 lbs
11. Which of the following is the nearest approximation of 10 kilometers?
a. 5 miles
b. 10miles
c. 16 miles
d. 20 miles
12. Consider a rectangle with a fixed perimeter. Assume that we allow continuous variations of
either its width of length. Which of the following is NOT possible?
a. The area of the new rectangle will be greater than the original area.
b. The area of the new rectangle will be smaller than the original area.
c. The area of the original rectangle will be equal to the area of all new rectangles that
can be formed.
d. The area of the original rectangle will be equal to the area of some new rectangles
that can be formed.
13. What is the reason for your answer in item number 12?
a. Rectangles possibly have the same perimeter but of different area.
b. Rectangles with the same perimeter always have the same area.
c. If rectangles have larger perimeter, then their area is wide.
d. If rectangles have small perimeter, then their area is small.
14. A largest circle is to be made out of a square having sides of 20 cm. Approximate the
materials that will be wasted.
a. 400.00 sq. cm
b. 314.00 sq. cm
c. 86.00 sq. cm
d. 22.80 sq.
cm
15. A circle with radius 12 cm and a rectangle with width 16 cm have equal areas. Which of the
following is the approximation of the length of the rectangle?
a. 4.19 cm
b. 9.00 cm
c. 4.71 cm
d. 28.26 cm
16. 2. One hectare is equivalent to 10,000 square meters. How many hectares are in a
rectangular field which is 750 m wide and 800 m long?
A. 120
B. 56
C. 60
D. 40
17. An aquarium is 40 cm high. Its length is twice its height, while its width is half its height.
How much water is needed to fill in the aquarium in cu. cm.?
A. 64,000
B. 32,000
C. 16,000
D. 8,000
18. What is the volume of a cube if one face has a perimeter of 36 cm?
A. 729 cu. cm.B. 216 cu. cm.
C. 46,656 cu. cm
D. 5,832 cu. cm
19. How much water can be filled with a cone whose diameter is 24 cm and whose height
exceeds 2/3 of the radius by 2 cm?
A. 480 cu. cm.
B. 540 cu. cm.
C. 660 cu. cm.
D.
900 cu. cm.

LICENSURE EXAMINATION FOR TEACHERS (LET)

St. Louis Review Center-Inc-Davao Tel. no. (082) 224-2515

78

Refresher Course
Content Area: MATHEMATICS
Focus: Algebra
Prepared by: Daisy de Borja-Marcelino
LET Competencies:

Solve for the roots of a given quadratic equation


Solve problems on quadratic equations
Determine an equation given a set of roots which are imaginary/complex numbers
Perform operations involving exponential and logarithmic functions
Solve for the solution set of a given inequality
Determine the rth term of the expansion (a + b) n
Solve problems involving variations
Determine the number of positive and negative roots of a given polynomial

Equations
An equation that contains at least one variable is called an open sentence. Equations

b&c
above are examples of open sentences. In equation b, only -1 makes the sentence true or
satisfies the equation. However, more than one number might satisfy an equation. For example,

x2 4 0
+2 and -2 satisfy the equation
. Any number that satisfies an equation is called a
solution or root to the equation. The set of numbers from which you can select replacements for
the variable is called the replacement set. The set of all solutions to an equation is called the
solution set to the equation. To solve an equation means to find all of its solutions.
QUADRATIC EQUATION
An equation of the form ax2 + bx + c = 0 where a
quadratic equation.

0, a ,b, and c are constants, is a

ROOTS OF QUADRATIC EQUATIONS


To solve a quadratic equation means to find the value of x (unknown) that will satisfy the
given equation. The values of x that will make the equation true are called the roots or solution of
the quadratic equation.
Methods of Finding the Roots of a Quadratic Equation
1. Factoring (Use this method if the quadratic equation is factorable)
Example: Determine the roots of x2 -8x - 15 = 0
Solution: Factoring the left side of the equation,
(x - 5) (x 3) = 0
Equating each factor to zero,
(x - 5) = 0
(x 3) = 0
x=5
x=3
2. Quadratic Formula

b b 2 4ac
x
2a
The quadratic formula is
BINOMIAL FORMULA
To obtain the terms of the binomial expansion (a + b)n, we use the binomial formula:

a na
n

n 1

n (n 1)a n 2 b 2 n (n 1)( n 2)a n 3 b 3


b

... nab n 1 b n
2!
3!

(a + b)n =
THE rth TERM OF THE EXPANSION (a + b)n

St. Louis Review Center-Inc-Davao Tel. no. (082) 224-2515

79

n (n 1)( n 2)...( n r 2)a n r 1 b r 1


(r 1)!
rth term =

EXPONENTIAL FUNCTIONS

The exponential function f with base b is denoted by f(x) =


bx, where

Properties of f(x) = bx

b > 0 , b

1, and x is any real number.

f has the set of real numbers as its domain.

f has the set of positive real numbers as its range.

f has a graph with a y-intercept of (0,1).

f is a one-to-one function.

f has a graph asymptotic to the x-axis.

f is an increasing function if b>1 and f is a decreasing function if 0<b<1.


An exponential function has a constant base and a variable exponent.

The Natural Exponential Function

f ( x) e x
For all real numbers x, the function defined by
is called the natural exponential
function. Note that e is an irrational number and its accurate value to eight places is 2.71828183.

EXPONENTIAL EQUATIONS
An equation where the unknown quantity appears in an exponent is called an exponential
equation.
SOLVING EXPONENTIAL EQUATIONS
To solve an exponential equation is to find the value of the unknown quantity in the given
equation.
LOGARITHMIC FUNCTIONS

For x > 0, b > 0, and b


Note: only if ay = x.

1, we have y = log a(x) if and

1. If the base of the logarithm is not indicated it is understood that the base is 10.
2. If the base of the logarithm is the number e, then it is called a natural logarithm and it is
written as f(x) = ln x.

f x log b x

Properties of

f has the set of positive real numbers as its domain.

f has the set of real numbers as its range.

f has a graph with a x-intercept of (1,0).

f is a one-to-one function.

f has a graph asymptotic to the y-axis.

f is an increasing function if b>1 and f is a decreasing function if 0<b<1.


St. Louis Review Center-Inc-Davao Tel. no. (082) 224-2515

80

Remarks

Logarithmic functions are the inverse of exponential functions.

We can use the rules of exponents with logarithms.

The two most common logarithms are called common logarithms and natural logarithms.
Common logarithms have a base of 10, and natural logarithms have a base of e.

Equation in exponential form can be rewritten in logarithmic form, and vice versa.

y log b x
The exponential form of

by x
is

Example1: The exponential equation 72 = 49 may be written in terms of a logarithmic equation as


log7 (49) = 2.

1
93
Example 2: The exponential equation 9-3 =

1
729
or

may be written in terms of a logarithmic

729

equation as log9

= -3

Basic Properties of Logarithms


Property 1 : loga (1) = 0 because a0 = 1.
Example 1: In the equation 220 = 1, the base is 22 and the exponent is 0. Remember that a
logarithm is an exponent, and the corresponding logarithmic equation is log 22 (1) = 0, where the 0
is the exponent.

2

5
Example 2: In the equation

2
5
= 1, the base is

and the exponent is 0. Remember that a

log 2
5

logarithm is an exponent, and the corresponding logarithmic equation is

1 = 0.

Property 2: loga (a) = 1 because a1 = a


Example 3: In the equation 71 = 7, the base is 7, the exponent is 1, and the answer is 7. Since a
logarithm is an exponent, and the corresponding logarithmic equation is log 7 7 = 1
Example 4: Use the exponential equation m1 = m to write a logarithmic equation. If the base m is
greater than 0, then logm (m) = 1.
Property 3: loga (a)x = x because ax = ax
Example 5: Since 92=92, we may write the logarithmic equation with base 9 as log9 92 = 2.
Example 6: Since you know that 112=112, we may write the logarithmic equation with base 11 as
log11112 = 2.
INEQUALITIES
Any relation expressed using the symbols <, >, > or < is called an inequality.
An absolute inequality is an inequality which is always true. A conditional inequality is one
which is true only for certain values of the variable involved.
1. 4 > 3 is an absolute inequality
2. x > 3 is a conditional inequality
PROPERTIES OF INEQUALITIES
Let a, b, c, & d be real numbers. The following hold.
St. Louis Review Center-Inc-Davao Tel. no. (082) 224-2515

81

1. Trichotomy Property
a > b or a < b or a = b
2. a > b if a - b > 0
a < b if a b < 0
3.
a. If a> 0 and b> 0, then a + b> 0 and ab>0.
b. If a < 0 and b < 0, then a+b< 0 and ab> 0
4. Transitivity
If a < b and b < c then a < c.
5. Addition Property
If a < b and c < d, then a + c < b+ d
6. Multiplication Property
If a < b and c > 0, then ac < bc
If a < b and c < 0, then ac > bc
SOLVING INEQUALITIES
To solve an inequality means to find the value of the unknown that will make the inequality
true.
POLYNOMIAL FUNCTION
The function defined by the equation
f(x) = a0xn +a1xn-1 + a2xn-2 + . . .+ an-2x2 + an-1x + an

0
where n is a nonnegative integer and a 0, a1,

. . .,

an are constants, a0,

is a polynomial

function in x of degree n. The zeros or roots of f(x) are the numbers that will make f(x) = 0.
The Number of Positive and Negative Roots of a Polynomial Function
If f(x) is a polynomial function with real coefficients, then the following are true.
The number of positive real zeros of f(x) is either equal to the number of variations in sign in
f(x), or to that number diminished by a positive even integer.
The number of negative real zeros of f(x) is either equal to the number of variations in sign in f(x), or to that number diminished by a positive even integer.
LICENSURE EXAMINATION FOR TEACHERS (LET)
Refresher Course
Majorship: MATHEMATICS
Prepared by: Daisy de Borja-Marcelino
FOCUS: Mathematical Investigation and Problem Solving
LET COMPETENCIES:
1. Cite differences between problem solving and mathematical investigations.
2. State patterns observed as conjectures.
3. Solve non-routine problems.
CONTENT UPDATE
I. Problem Solving
Problem solving is defined as a set of actions to be done to perform the task or to solve the
problem. It is a process of applying acquired knowledge to a new or unfamiliar situation.
I. Polyas Problem-Solving Principles
1. Understand the problem
Understanding the given problem is a very important principle in order to solve it correctly.
The following questions may be helpful in the analysis of a given problem:
Do you understand all the words used in stating the problem?
St. Louis Review Center-Inc-Davao Tel. no. (082) 224-2515

82

What are you asked to find or show?


Can you restate the problem in your own words?
Could you work out some numerical examples that would help make the problem clear?
Could you think of a picture or diagram that might help you understand the problem?
Is there enough information to enable you to find a solution?
Is there extraneous information?
What do you really need to know to find a solution?
Is there yet another way to state the problem?
What does key word really mean?

2. Devise a plan
The following strategies may be of great help as you learn the art of problem solving.
guess and check
make a table
make an orderly list
use a variable
draw a diagram
work backward
look for a pattern
eliminate possibilities

3. Carry out the plan


To carry out the plan you devised earlier be careful and be patient to make it work. If it
doesnt work after several trials, then discard it and try a new strategy.
4. Look back
Looking back is an important step in developing problem-solving skills. Once you have
solved the problem make it a habit to go over your solution and polish it.
II. Mathematical Investigation
A mathematical investigation of a problem or of an open-ended situation is a sustained
exploration of
the problem or situation.
Stages in a Mathematical Investigation
1. Getting Started
- Attaining familiarity with the situation to be investigated.
- Producing instances, maybe starting from the simplest or whatever is interesting.
- Deciding on what is worth pursuing.
2. Exploring Systematically
- Systematic listing/ drawing;
- Organizing relationships in tables or graphs; and
- Looking for a pattern or relationship.
3. Making Conjecture
- Making general statements about patterns or relationships observed in the cases
considered.
A conjecture is a generalization obtained inductively, which has not been validated or proven
true.
4. Testing/ Verifying Conjectures
- Checking consistency of conjectures using existing cases;
- Predicting results for untried cases for which data are available.
5. Explaining/ Justifying Conjectures - Explaining why the conjectures made will work for new or all
cases
6. Reorganizing
- Simplifying/ generalizing the approach
- Seeing the connection among the conjectures
7. Elaborating - Extending the investigation by considering other aspects of the investigation
8. Summarizing - Involves an account or summary, written or oral, of what has been obtained in
stages 2 7, with some reference on the experiences in stage 1.

MAJORSHIP: MODERN GEOMETRY


St. Louis Review Center-Inc-Davao Tel. no. (082) 224-2515

83

LET COMPETENCIES:
1. Give characteristics of non-Euclidean geometry which are not found in Plane Euclidean
Geometry.
2. Define and illustrate concepts in linear algebra
3. Apply properties of matrices in performing matrix operations
4. Evaluate determinants
5. Perform modular clock arithmetic
MODERN GEOMETRY
Non-Euclidean geometry
Non-Euclidean Geometry is any geometry that is different from Euclidean geometry. The two most
common non-Euclidean geometries are elliptic geometry and hyperbolic geometry.
A. Hyperbolic Geometry
Hyperbolic geometry is known as saddle geometry or Lobachevskian geometry. It differs in many
ways from Euclidean geometry, often leading to quite counter-intuitive results. Some of the
remarkable consequences of this geometry's unique fifth postulate include: Moreover, in this field,
more than one distinct line through a particular point will not intersect another given line..
1. The sum of the three interior angles of a triangle is strictly less than 180. Moreover, the angle
sums of two distinct triangles are not necessarily the same.
2. Two triangles with the same interior angles have the same area.
B. Elliptic Geometry
In Elliptic geometry there are no lines that will not intersect,
Some theorems in Elliptic Geometry
1. The angle sum of any triangle is more than 180o.

2. Given two lines perpendicular to line CG. By the parallel postulate for elliptic geometry,
these two lines meet at a point A. Then every line through A is perpendicular to line CG.
C. Projective Geometry
Projective geometry is the most general and least restrictive in the hierarchy of fundamental
geometries. It is an intrinsically non-metric geometry, whose facts are independent of any metric
structure. Under the projective transformations, the incidence structure and the cross-ratio are
preserved. In particular, it formalizes one of the central principles of perspective art: that parallel
lines meet at a point called an ideal point. Consequently, the five initial axioms in Euclidean
Geometry resulted to the following axioms.
1. Any two distinct points determine one and only one line.
2. Any two distinct coplanar lines intersect in one and only one point.
3. Any line not in a given plane intersects the plane in one and only one point.
4. Any two distinct planes intersect in one and only one line.
5. Any three noncollinear points, also any line and a point not on the line, determine one and
only one plane.
MATRICES AND MATRIX OPERATIONS
Definition
A matrix is defined as a rectangular array of elements. The entries, also called elements, may
be real, complex or functions. If the arrangement has m rows and n columns, then the matrix is of
order m x n (read as m by n). A matrix is enclosed by a pair of parameters such as ( ) or [ ]. It is
denoted by a capital letter.
St. Louis Review Center-Inc-Davao Tel. no. (082) 224-2515

84

a11
a
A 21

am1

a12
a22

am 2

a1n
a2 n
aij

amn
A = [ a ij ]

TYPES OF MATRICES

1. The ROW MATRIX: This matrix has only one row.

Example: [1 6 4 3] This is a 1 X 4 row matrix


2. The COLUMN MATRIX: This matrix has only one column.

10
2

8
Example:

This is 4 x 1 column matrix

3. The RECTANGULAR MATRIX: This has two or more rows with two or more columns.
Example

2 3 5 2
0
8 1 4

This is a 2 X 4 matrix, because it contains two rows and four

columns

0
8 11 22
7
6
95 10

0
45 49
68
This is a 3 X 4 matrix
4. The SQUARE MATRIX: This is a special case of a Rectangular Matrix; here the
number of rows is equal to the number of columns.

a b
d e

g h
Example:

1
5

c
f
i

3
1
1
5

4
1
1

2
3
1
3

B=

Here A and B are square matrices of order 3 and 4 respectively


5. The DIAGONAL MATRIX: This is a square matrix where all its non-diagonal elements are 0
Example:

8 0 0
0 1 0

0 0 8

92 0
0 1

(a)

5
0

(b)

0
6
0
0

0 0
0 0
3 0

0 10

(c)

These are diagonal matrices of order 2, 3 and 4 respectively.


6. The SCALAR MATRIX: This is a diagonal matrix where all the elements on its leading
diagonal to bottom right are of equal value.
Example:

St. Louis Review Center-Inc-Davao Tel. no. (082) 224-2515

85

6
0

3 0 0
0 3 0

0 0 3

2 0
0 2

0
6
0
0

0
0
6
0

0
0
0

(a)
(b)
(c)
7. The IDENTITY MATRIX: This is a scalar matrix where the elements on its leading diagonal
(the diagonal running from top left to bottom right) are 1 and the rest are of value 0
Example:

1
0

0
1
0
0

0
0
1
0

0
0
0

1
This is an identity matrix I4 of order 4.

PROPERTIES OF MATRIX ADDITION


Theorem Let A, B, C, and D be matrices of the same size, m x n.
1. A + B = B + A

(Commutativity)

2. A + (B + C) = (A + B ) + C

(Associativity)

3. There is a unique m x n matrix O such that A+ 0 = A for any m x n matrix A. The


matrix O is called the m x n additive identity or zero matrix.
4. To each m x n matrix A, there is a unique m x n matrix D such that
A+D=0
We write D as (- A), so that A + D = 0 can be written as A + (-A) = 0. The matrix (-A) is called the
additive inverse or negative of A.
PROPERTIES OF MATRIX MULTIPLICATION
Theorem Let A, B, and C be matrices of the appropriate sizes.
1. A(BC) = (AB)C

(Associativity)

2. A(B + C) = AB + AC

(Right Distributivity)

3. (A + B)C = AC + BC

(Left Distributivity)

PROPERTIES OF MULTIPLICATION BY A SCALAR


Theorem Let A and B be matrices of the appropriate sizes, and let r and s be scalars.
1. r(sA) = (rs)A (Associativity)
2. (r + s)A = rA + sA (Distributivity I)
3. r(A + B) = rA + rB (Distributivity II)
4. A(rB) = r(AB) = (rA)B

St. Louis Review Center-Inc-Davao Tel. no. (082) 224-2515

86

Definition

Let A = [ai j ] be an n x n matrix. We define the determinant of A (written

A
det(A) or

) by

()a

A
det A =

1 j1

a 2 j ...a n j
2

where the summation ranges over all permutations j 1 j2 jn of the set S =


{1,2,n}.The sign is taken as + or according to whether the permutation
j1 j2
jn is even or odd.

Second-order Determinant

a1
a
2

b1
b2

If A is the square matrix of order two

a1
a2

either det A or

b1
b2

a1
a2

,is defined by

then the determinant of A, denoted by

b1
b2

= a1b2 a2b1

Example:
Compute the determinant:

PROPERTIES OF DETERMINANT
1. The determinant of a matrix and its transpose are equal, that is, det(AT ) = det(A).
2. If two rows (columns) of A are equal, then det (A) = 0
3. If a row (column) of A consists entirely of zeros, then det (A) = 0

4. The determinant of a diagonal matrix is the product of the entries on its main diagonal.
5. If matrix A = [ai j] is upper (lower) triangular, then det (A) =a11a22ann; that is
The determinant of a triangular matrix is the product of the elements on the main diagonal.
6. If a multiple of one row of A is added to another row to produce a matrix B,
then det B = det A.

St. Louis Review Center-Inc-Davao Tel. no. (082) 224-2515

87

7. If two rows of A are interchanged to produce B, then det B = - det A.

8. If one row of A is multiplied by k to produce B, then det B = k det A.

9. The determinant of a product of two matrices is the product of their determinants; That is,
det(A B) = det(A) det(B)
MODULAR CLOCK ARTITHMETIC
Clock (or modular) arithmetic is arithmetic you do on a clock instead of a number line. On a 12hour clock, there are only twelve numbers in the whole number system. However, every number
has lots of different names. For example, the number before 1 is 0, so 12 = 0 on a 12-hour clock

In clock arithmetic, you can add, subtract, and multiply, you can divide by some numbers.
Addition and Subtraction
Addition and subtraction work the same as on number line. For example, to add 9 and 7, start at
0, count 9 along the line, and then count 7 more. You are at 16.If you count on a 12-hour clock,
you will be at 4.
To add negative numbers, use the minus (-) sign to change direction. To subtract on a clock, first
find standard (positive) names for the two numbers, count clockwise for the first one, and count
counter clockwise for the second.

In ordinary arithmetic, the additive inverse of 4 is -4. In mod 12 arithmetic, the additive
inverse of 4 is 8. In either system, the sum of a number and its additive inverse is zero.

Rows corresponding to additive inverses are opposites of one another, save for the
number 0.

Addition Mod 12 ( 12- hour clock)

St. Louis Review Center, Inc.


3RD Flr. La Suerte Bldg., Rizal
Ave., San Pablo City
Tel. no. (049) 562-2239/0929-688-4348

POST TEST SOCIAL SCIENCES

St. Louis Review Center-Inc-Davao Tel. no. (082) 224-2515

L.E.T APRIL 2010


88

What branch of philosophy is closely


related to education because it is
concerned with human knowledge in
general and the criteria of truth?
a. Metaphysics
c. Epistemology
b. Logic
d. Ethics
2 This is the philosophical study of the
universe and everything in it
a. Cosmology
c. Metaphysics
b. Ethics
d. Axiology
3 What philosophy posits that a social
teacher is a revered person, central to the
educative process and therefore must be
excellent mentally and morally in
personally conduct and conviction?
a. Realism
c. Essentialism
b. Idealism
d. Progressivism
4 What would the aim of a realist education
likely be?
a. Contribute to the development of
mind and self
b. Provide students with the knowledge
he will need to survive in the natural
world
c. Give students insights into their
traditions and adapting students to
society
d. Meet the need of a growing child
rather than emphasize the subject
matter
5 This philosophy emphasizes the teaching
of controversial issues to prepare
students for a progressive structuring of
the social order
a. Progressivism
c.
Reconstructionism
b. Social traditionalism
d. Social
Experimentalism
6 The philosophy whose criterion of truth is
seeing is believing
a. Essentialism
c. Progressivism
b. Native realism
d.
Existentialism
7 This
is
the
systematic
consistent
explanation of all the facts of experience.
Its technical term is reason which is
considered as the best criterion of truth
a. Pragmatism
c.
Correspondence
b. Consistency
d.
Coherence
8 This philosophy contends that truth is
universal an unchanging and therefore a
good education is also universal and
constant
a. Perennialism
c. Existentialism
b. Progressivism
d. Essentialism
9 Which philosophy has for its curriculum a
body intellectual subject matter that are
conceptual on subject, essential for the
realization
of
mental
and
moral
development?
a. Realism
c. Existentialism
b. Idealism
d. Pragmatism
10 A branch of Philosophy that studies the
morality of human action
a. Epistemology
c. Ethics
b. Metaphysics
d. Cosmology
11 The area of Philosophy that specifically
deals with the problem of human values
a. criteriology
c. Theodicy
b. Epistemology d. Axiology
12 The aim of this philosophy is to reduce
statements about education to empirical
terms

13

14

15

16

17

18

19

20

21

22

a. Reconstructionism
c.Philosophical
analysis
b. Existentialism
d.
Social
traditionalism
The educational theory, in response to
progressivism, which concerns with the
revival of effort in the direction of
teaching the fundamental tools of
learning as the most indispensable type
of education
a. Essentialism c. Philosophical analysis
b. Perennialism d. Social traditionalism
This philosophy believes that the schools
should originate policies and progress
which would bring about reform of the
social order
a. reconstructionism
c.
Existentialism
b. Progressivism
d.
Perennialism
This is the logical conclusion if we are to
apply the philosophy of martin Buber to
education
a. Education will be characterize by
imposition and rules
b. Education will be characterize by
mutual unfolding of persons
c. Education will be characterize by
pretensions
d. Education will be characterize by self
motivation
Man as an embodied spirit means
a. Man is a body and soul
b. Man is a rational animal
c. Man is a totally, unique core or center,
source wellspring of initiative and
meaning
d. Man is subjective
Man as an embodied sprit implies that
education should
a. Develop the intellect / mind
b. Develop mans individuality
c. Develop
essential
physical
and
intellectual skills
d. Develop the total man
human beings are living in a human world
implies that
a. Education must help man give
meaning to his world of others
b. Education must develop skills to
survive in this world
c. Education must be relevant to this
world
d. Education must be in the context of
societal needs
This philosophy contends that the
intellectual faculty of man is higher that
the physical faculties
a. Idealism
c.
Existentialism
b. Realism
d.
Logical
Positivism
The philosophical study of the essence
and nature of man
a. Rational psychology
b. Epistemology
c. Philosophy of human person
d. Social Philosophy
The branch of philosophy that studies the
morality of human acts
a. Aesthetics
c. Ethics
b. Cosmology
d. Theodicy
This is the philosophical study of human
knowledge and the criteria of truth
a. Axiology
c. Logic
b. Epistemology
d. Metaphysics

St. Louis Review Center-Inc-Davao Tel. no. (082) 224-2515

89

23 This contemporary philosophical theme in


education contends that education must
involve in its processes the total human
being-his subjectivity, the rational, the
material and the emotional aspect for
man is a unity and not a duality
a. Genuine dialogue
b. Man as a being in the world
c. Man as an embodied spirit
d. The socials and the inter human being
24 This philosophy proclaims the spiritual
nature and the universe. It holds that the
good, the true and the beautiful are
permanently part of the structure of an
unchanging universe
a. Existentialism
c. Materialism
b. Idealism
d. Realism
25 This philosophy posits the know ability of
the world and everything in its as they
are in themselves and their existence
independent of human mind
a. Existentialism
c. Materialism
b. Idealism
d. Realism
26 The central aim of education should be
to develop the power of thought this
would likely be the aim of what
educational practitioner?
a. Existentialism
c. Progressivist
b. Perennialist
d.
Reconstructivist
27 This educational theory is based on the
philosophy grounded on experience and
the interaction of the person with his
environment. It posits that education
must use past experiences to direct
future experiences
a. Existentialism
c. Realism
b. Idealism
d. progressivism
28 This philosophy is a way of viewing and
thinking about life in the world so that
priority is given into individualism and
subjectivity. It believes that human beings
are the creator of their own experiences
a. Existentialism
c. Realism
b. Idealism
d.
Reconstruction
29 This emerging social values in education
stresses that man exist through the other
and for others for he becomes actuated
through relations
a. Existential Dialogue c. Socialization
b. Personalization
d.
Vigilance
30 To
what
particular
concept
must
education
be
based
according
to
contemporary philosophy?
a. Existential dialogue
b. Social Change
c. Supremacy of the human person
d. Synergy
31 This emerging social value in education
stresses the importance if the weexperience ad the converging of the
worlds
a. Authentic being
c.
Personalization
b. Existential dialogue d. Synergy
32 The essentialist would likely have this as
the ultimate aim of education
a. To fit man to perform, justly, skillfully
and magnanimous in all times in
peace and war
b. To develop the power of thought. To
search for an disseminate the truth
c. To give all pupils insight into their
traditions

d. To meet the needs of the growing


child
33 The philosophical belief that reality is
precisely what as it appear to be and
adheres to the belief that seeing is
believing
a. Coherence
c. Nave Realism
b. Consistency
d. Pragmatism
34 It is belief that when an idea agrees with
its object, it is proof of its truth. However,
it is definition of truth not a criteria
a. Coherence
c. Nave Realism
b. Consistency
d.
Correspondence
35 The belief that the ultimate criterion of
truth is if an idea works then it is true
a. Coherence
c. Nave Realism
b. Consistency
d. Pragmatism
36 It is a systematic consistent explanation
of all the facts of experience. Its technical
name is reason. This is believed to be the
ultimate criterion of truth
a. Coherence
c. Nave Realism
b. Consistency
d. Pragmatism
37 This educational philosophy encourages
accumulation of knowledge and thinking
and must apply criteria for moral
evaluation. Suggested methods are
questioning and discussion, lecture and
the project, whether done singly or in
group
a. Existentialism
c. Materialism
b. Idealism
d. Pragmatism
38 This theory views education as a
recurring process based on eternal truths;
thus, the schools curriculum should
emphasize the recurrent themes of
human life
a. Perennialism
c. Progressivism
b. Pragmatism
d.
Reconstructionism
39 The philosophical study of human mind
a. Axiology
c. Metaphysics
b. Epistemology d. Rational psychology
40 The philosophical study of being, its
nature and essence
a. Axiology
c. Metaphysics
b. Epistemology
d.
Rational
Psychology
41 A sociologist would be most interested in
studying
a. Artifacts of the fast civilization
b. Group behavior
c. Psychological problems
d. Economic institution
42 Interrelated social structures that fit
together to form an integrated whole
a. Structural functionalism
b. Conflict theory
c. Symbolic Interactionism
d. Disruptive Functionalism
43 Development of self and adaptation of
individual to society
a. Structural functionalism
b. Conflict theory
c. Symbolic Interactionism
d. Disruptive Functionism
44 Which is NOT a method in sociological
inquiry
a. Experimental study c.
Intuitive
study
b. Longitudinal Study d. Ex-post Facto
study

St. Louis Review Center-Inc-Davao Tel. no. (082) 224-2515

90

45 Which of the following give some


difficulties to a sociological researcher?
a. Complexity of phenomena
b. Constant change
c. Unpredictability of behavior
d. All of the above
46 A cluster of behavior patterns related to
the general culture of a society and yet
distinguishable from it
a. Sub-culture
c. Norm
b. Culture shock
d. mores
47 Which group would most likely have a
counter culture?
a. Musician
c. Singers
b. Drugs users
d. Dancers
48 The complex whole which includes
knowledge, beliefs, arts moral, laws,
customs and other capabilities and habits
acquired by man as a member of the
society
a. Norm
c. Culture
b. Folkways
d. Mores
49 When one consider the culture of another
group as inferior to his own, this connotes
a. Ethnocentrism
c.
Assimilation
b. Cultural relativism
d. Acculturation
50 The rightness or wrongness of what one
does depend on where he is doing it
a. Cultural integration c.
Cultural
relativism
b. Cultural lag
d. Cultural traits
51 The culture that includes undesirable
behavior of the member of society.
Formally condemned but widely practiced
a. Real culture
c. Sub-culture
b. Ideal culture
d.
Counter culture
52 The process by which a member learns
the norms of the group
a. Accommodation
c.
Compartmentalization
b. Aggregation
d. Socialization
53 A role or status assigned according to
heredity traits without regard to individual
preference, ability or performance
a. Achieved
c. Attached
b. Ascribed
d. Applied
54 An achieved position in a society
a. Heir to the throne
c. Movie actor
b. Presidential son
d.
First
lady
55 Refers to the difficulty people have in
meeting their role obligations
a. Role conflict
c. Role strain
b. Role set
d.
Role
ambiguity
56 When a supervisor is tasked to report
unsatisfactory performance of her coworkers who are also her friends. She
may experience
a. Role play
c. Role strain
b. Role conflict
d.
Role
ambiguity
57 Social control imposed on social deviants
to maintain
a. Social stigma
c. Social order
b. Social strata
d. Social mobility
58 Which of the following is NOT true?
a. The
deviant
behavior
of
one
generation may become the norm of
the next
b. All forms of deviant are bad
c. Deviant behavior is one way of
adapting a culture to a social change

59

60

61

62

63

d. Some deviation are due to the failure


of the socialization process to
integrate the cultural norms into
individuals personality
A teenager in a good neighborhood of
stable families and conventional people
who rejects middle class norms and
become a delinquent is an example of
a. Individual deviation c.
Counter
deviation
b. Group deviation
d.
Relative
deviation
A case of compulsive drug addict is an
example if a
a. Primary deviant
c.
Tertiary deviant
b. Secondary deviant d. All of the
above
Which is NOT a characteristic of a social
group?
a. Physical collection of people
b. Shares a common purpose and
conscious of each other
c. Share some common characteristics
d. Member interact with one another
Which is NOT a characteristic of the
relationship among members of a primary
group?
a. Personal
c. Segmental
b. Informal
d. Sentimental
Which is an example of members if a
primary group?
a. Peer group
c.
School
administration
b. Family
d. Neighbors

64 Which of the following statements is NOT


true about Filipino families today?
a. Male authority has declined
b. Division of labor has changed
c. Separation has become more common
d. Has cease to be socialization unit in
society
65 Which of the following is NOT an ethical
issue that must be confronted when doing
social research?
a. Are the subjects being paid enough?
b. What degree of risk, pain or harm is
involved?
c. To what extent are the subjects being
deceived?
d. Will there be disclosure of confidential
or personally harmful information?
66 In
most
instances
of
participant
observation the researchers:
a. Hide their true identity
b. Do not hide their true identity
c. Pay their informants for information
d. Acts as therapist to the subject
67 A research technique in which the
investigators enter to the activities of the
group at the same time they study the
groups behavior is
a. Participants observation
b. A semi-structured (open-ended)
Interview
c. A structured interview
d. A data discussion
68 Participant observation:
a. A research tool used to follow-up on
expected findings
b. A research interview in which the
investigators ash a list of questions,

St. Louis Review Center-Inc-Davao Tel. no. (082) 224-2515

91

69

70

71

72

73

but is free to vary them or make up


new ones that become important
during the course of the interview
c. A research technique in which the
investigators enter into activities of
the group at the same time they study
the groups behavior
d. A research interview determined
entirely in advance and followed
rigidly
A research interview determined entirely
in advance and followed rigidly is:
a. Participants observation
b. A semi structured (open-ended)
interview
c. A structured interview
d. A data discussion
A research interview in which the
investigators ask a list of questions, but is
free to vary them or make up new ones
that become important during the course
interview, that is
a. Participant observation
b. A semi structured (open-ended)
interview
c. Structured interview
d. A data discussion
A conversation between two or more
individuals in which one party attempts to
gain information from others(s) by asking
of questions is known as;
a. A survey
c.
Participant
observation
b. An interview d. Hypothesis testing
Sociological and anthropological be
misused because it:
a. Is hard to understand
b. Is often used selectively by groups or
individuals to support certain motives
c. Is often ambiguous
d. Has few applicants
A survey by their nature usually
a. Are cross-sectional
b. Are longitudinal
c. Involve secondary methodology
d. Involve participants observation

74 A cross-sectional study is
a. A research technique in which
investigators enter into activities of
the group at the same time they study
the groups behavior
b. Research examines a population at a
given point in a time
c. Research
which
examines
a
population, or portion thereof, is
questioned in order to reveal specific
facts about itself
d. Research
that
investigates
a
population at several intervals over
relatively a long period of time
75 Research that investigates a population at
several intervals over a relatively long
period of time is called
a. A cross-sectional study
c.
Secondary Analysis
b. A residual analysis d.
Longitudinal
research
76 Survey are used when:
a. Other measure do not produce
statistical significance
b. The findings of the study must be
repeatable
c. It is desirable to discover he
contribution and interrelations of

77

78

79

80

81

82
83
84

85

86

certain variables among large number


of people
d. Procedures
having
cross-cultural
applications needed
A research method is which a population,
or portion thereof, is questioned in order
to reveal specific facts itself is known as:
a. A survey
c. Experiments
b. Participant observation
d.
Residual analysis
Which of the following is NOT one of the
three main methods of research used by
sociologists?
a. A survey
c. Experiments
b. Participants observation
d.
Residual analysis
A statistical technique to make all
significant
groups
in
as
society
represented in a sample proportion to
their numbers in the large society is
known as:
a. Random sampling
b. A stratified random sample
c. Sampling
d. Randomized response
The goal in designing a research project
should be:
a. To prove a point
b. To provide a collection for all
necessary and sufficient data to test
the stated hypothesis
c. To selectively isolate the variables in
order to prove the hypotheses are
either correct or incorrect
d. To prove hypothesis false. If they
cannot proven false, they must be
true
An independent variable is
a. A testable statements about the
relationship between two or more
empirical variables
b. Anything that can be change
c. A variable that changes for reason
that have nothing to do with another
variable
d. A variable that changes in response to
changes in another variable
Which of the following is not a sign?
a. A clenched first
c. A flag
b. A knock on the door d. Yawning
The most important symbols are
a. Action
c. Actions
b. Words
d. Behavior
The principal means through which
culture is transmitted from generation to
generation is
a. Actions
c.
Language
b. Diffusion
d. Behaviors
If apes can be taught to use language, it
denies the views that:
a. Only human can think
b. Only human have culture
c. Apes do not have history to pass on
d. Apes are stupid
Which of the following statements about
culture is not true?
a. Every social group must have a
culture on its own in order to function
b. Every individual participates in a
number of different culture
c. Meeting the social expectation of
several cultures is often source of tree
d. Families do not have their own culture
but instead reflect the culture of the
larger society

St. Louis Review Center-Inc-Davao Tel. no. (082) 224-2515

92

87 Promised to make this country great


again. Ruled for about twenty years
a. Diosdado Macapagal c.
Ramon
Magsaysay
b. Ferdinand Marcos
d. Carlos Garcia
88 The form of government established by
Aguinaldo was change from dictatorial to:
a. Democratic
c. Republic
b. Sociologist
d. Revolutionary
89 Which provides he Tagalog shall be the
official language of the republic?
a. Constitution of 1935 c. Biyak na Bato
b. Makabula
d. Malolos
90 Which of the following were the greatest
naval battles in history during Japanese
occupation of the Philippines?
a. Battle of Leyte gulf c. Claveri
b. Surigao Strait
d. Dela Torre
91 The most cultured of the reformist, wrote
a socio-historical novel based on facts he
gathered in the Philippines
a. Marcelo H. Del Pilar c. Mariano Ponce
b. Jose Rizal
d.
Jose
Ma.
Paganiban
92 The editor of La Solaridad and a great
orator, he initiated the reform movement
a. Marcelo H. Del Pilar c. Emilio Jacinto
b. Jose Rizal
d.
Graciano
Lopez-Jaena
93 The generation of the spirits was common
among ancient Filipinos. Memories of
their dead relatives were kept alive
through craved idol. This practice was
called
a. Cult of the dead
c. Nature
Worship
b. Mass of the dead d. Divination
94 The practice of having surnames was
started during Spanish regime through
the order of
a. Gov. Gen Jaudenes c.
Gov. Gen
Claveria
b. Gov. Gen Dela Torre d.
Gov.
Gen
Polavejia
95 As promised by Gen. Mc Arthur- he would
return to liberate the Filipinos after he
could train troops, in Australia. The Leyte
landing signified the liberation of the
Philippines, was well as the return of the
commonwealth government. Who was the
commonwealth president who came with
Mc Arthur?
a. Romulo
c.
Osmea
b. Roxas
d. Quezon
96 Men and women of pre-colonial times had
weakness for personal adornment. A side
from jewels, tattooing of the body was
common. In the Visayas tattooed men
were called
a. Umalohokan
c. Abi
b. Pintados
d. Plista
97 Memories of brutality which was so in
human during the Japanese regime were
always associated with a particular
Japanese group called
a. Kalibapi
c.
Kamikaze
b. Kempetai
d. Samurai
98 Membership in the Katipunan come in
grades: first (katipons); second (kawal);
and third (bayani). Each grade designed
as well as password so they could
maintain their secrecy of the movement.
The password for the kawal grade was?

a. Rizal
c. Kawal
b. Gomburza
d. Sundalo
99 Pre-colonial trade in the Philippines was
prosperous. Business transactions made
use of:
a. Money system
c. Credit system
b. Barter system
d.
Banking
System
100
The use of magic charms was very
common among pre-Spanish Filipinos.
One of these is the anting-anting or
agimat which is meant to:
a. Insure a man against a weapon of
every kind
b. Made a man lovable to all ladies
c. Made a man invisible
d. Made him walk in storm or sea
without getting wet
***** THE END *****
WORK HARD,
Social science
1 C
2 A
3 B
4 B
5 C
6 B
7 D
8 A
9 A
10 C
11 D
12 C
13 A
14 A
15 B
16 C
17 D
18 A
19 A
20 C
21 C
22 B
23 C
24 B
25 d
26 b
27 d
28 a
29 c
30 c
31 d
32 a
33 c
34 c
35 d
36 a
37 b
38 a
39 d
40 c
41 b
42 a
43 c
44 c
45 d
46 c
47 b
48 c
49 a

St. Louis Review Center-Inc-Davao Tel. no. (082) 224-2515

DREAM HARDER
51
52
53
54
55
56
57
58
59
60
61
62
63
64
65
66
67
68
69
70
71
72
73
74
75
76
77
78
79
80
81
82
83
84
85
86
87
88
89
90
91
92
93
94
95
96
97
98
99

a
d
b
c
c
b
c
b
a
a
a
c
c
d
a
a
a
c
c
b
b
b
c
b
d
c
a
d
b
b
c
c
d
d
b
d
b
d
a
a
b
d
a
c
d
a
b
b
b
93

50

100

POST TEST SOCIAL SCIENCES


L.E.T APRIL 2010
101
What branch of philosophy is closely
related to education because it is
concerned with human knowledge in
general and the criteria of truth?
a. Metaphysics
c. Epistemology
b. Logic
d. Ethics
102
This is the philosophical study of the
universe and everything in it

a. Cosmology
c. Metaphysics
b. Ethics
d. Axiology
103
What philosophy posits that a social
teacher is a revered person, central to the
educative process and therefore must be
excellent mentally and morally in
personally conduct and conviction?
a. Realism
c. Essentialism
b. Idealism
d. Progressivism
104
What would the aim of a realist
education likely be?
a. Contribute to the development of
mind and self
b. Provide students with the knowledge
he will need to survive in the natural
world
c. Give students insights into their
traditions and adapting students to
society
d. Meet the need of a growing child
rather than emphasize the subject
matter
105
This philosophy emphasizes the
teaching of controversial issues to
prepare students for a progressive
structuring of the social order
a. Progressivism
c.
Reconstructionism
b. Social traditionalism d. Social
Experimentalism
106
The philosophy whose criterion of
truth is seeing is believing
a. Essentialism
c. Progressivism
b. Native realism
d.
Existentialism
107
This is the systematic consistent
explanation of all the facts of experience.
Its technical term is reason which is
considered as the best criterion of truth
a. Pragmatism
c.
Correspondence
b. Consistency
d.
Coherence
108
This philosophy contends that truth is
universal an unchanging and therefore a
good education is also universal and
constant
a. Perennialism
c. Existentialism
b. Progressivism
d. Essentialism
109
Which philosophy has for its
curriculum a body intellectual subject
matter that are conceptual on subject,
essential for the realization of mental and
moral development?
a. Realism
c. Existentialism
b. Idealism
d. Pragmatism
110
A branch of Philosophy that studies
the morality of human action
a. Epistemology
c. Ethics
b. Metaphysics
d. Cosmology
111
The area of Philosophy that
specifically deals with the problem of
human values
a. criteriology
c. Theodicy
b. Epistemology d. Axiology
112
The aim of this philosophy is to reduce
statements about education to empirical
terms
a. Reconstructionism
c. Philosophical
analysis
b. Existentialism
d. Social
traditionalism
113
The educational theory, in response to
progressivism, which concerns with the
revival of effort in the direction of
teaching the fundamental tools of

St. Louis Review Center-Inc-Davao Tel. no. (082) 224-2515

94

learning as the most indispensable type


of education
a. Essentialism c. Philosophical analysis
b. Perennialism d. Social traditionalism
114
This philosophy believes that the
schools should originate policies and
progress which would bring about reform
of the social order
a. reconstructionism
c. Existentialism
b. Progressivism
d. Perennialism
115
This is the logical conclusion if we are
to apply the philosophy of martin Buber
to education
a. Education will be characterize by
imposition and rules
b. Education will be characterize by
mutual unfolding of persons
c. Education will be characterize by
pretensions
d. Education will be characterize by self
motivation
116
Man as an embodied spirit means
a. Man is a body and soul
b. Man is a rational animal
c. Man is a totally, unique core or center,
source wellspring of initiative and
meaning
d. Man is subjective
117
Man as an embodied sprit implies that
education should
a. Develop the intellect / mind
b. Develop mans individuality
c. Develop essential physical and
intellectual skills
d. Develop the total man
118
human beings are living in a human
world implies that
a. Education must help man give
meaning to his world of others
b. Education must develop skills to
survive in this world
c. Education must be relevant to this
world
d. Education must be in the context of
societal needs
119
This philosophy contends that the
intellectual faculty of man is higher that
the physical faculties
a. Idealism
c.
Existentialism
b. Realism
d. Logical
Positivism
120
The philosophical study of the
essence and nature of man
a. Rational psychology
b. Epistemology
c. Philosophy of human person
d. Social Philosophy
121
The branch of philosophy that studies
the morality of human acts
a. Aesthetics
c. Ethics
b. Cosmology
d. Theodicy
122
This is the philosophical study of
human knowledge and the criteria of
truth
a. Axiology
c. Logic
b. Epistemology
d. Metaphysics
123
This contemporary philosophical
theme in education contends that
education must involve in its processes
the total human being-his subjectivity,
the rational, the material and the
emotional aspect for man is a unity and
not a duality
a. Genuine dialogue
b. Man as a being in the world

c. Man as an embodied spirit


d. The socials and the inter human being
124
This philosophy proclaims the spiritual
nature and the universe. It holds that the
good, the true and the beautiful are
permanently part of the structure of an
unchanging universe
a. Existentialism
c. Materialism
b. Idealism
d. Realism
125
This philosophy posits the know ability
of the world and everything in its as they
are in themselves and their existence
independent of human mind
a. Existentialism
c. Materialism
b. Idealism
d. Realism
126
The central aim of education should
be to develop the power of thought this
would likely be the aim of what
educational practitioner?
a. Existentialism
c. Progressivist
b. Perennialist
d.
Reconstructivist
127
This educational theory is based on
the philosophy grounded on experience
and the interaction of the person with his
environment. It posits that education
must use past experiences to direct
future experiences
a. Existentialism
c. Realism
b. Idealism
d. progressivism
128
This philosophy is a way of viewing
and thinking about life in the world so
that priority is given into individualism
and subjectivity. It believes that human
beings are the creator of their own
experiences
a. Existentialism
c. Realism
b. Idealism
d.
Reconstruction
129
This emerging social values in
education stresses that man exist through
the other and for others for he becomes
actuated through relations
a. Existential Dialogue c. Socialization
b. Personalization
d.
Vigilance
130
To what particular concept must
education be based according to
contemporary philosophy?
a. Existential dialogue
b. Social Change
c. Supremacy of the human person
d. Synergy
131
This emerging social value in
education stresses the importance if the
we-experience ad the converging of the
worlds
a. Authentic being
c.
Personalization
b. Existential dialogue d. Synergy
132
The essentialist would likely have this
as the ultimate aim of education
a. To fit man to perform, justly, skillfully
and magnanimous in all times in
peace and war
b. To develop the power of thought. To
search for an disseminate the truth
c. To give all pupils insight into their
traditions
d. To meet the needs of the growing
child
133
The philosophical belief that reality is
precisely what as it appear to be and

St. Louis Review Center-Inc-Davao Tel. no. (082) 224-2515

95

adheres to the belief that seeing is


believing
a. Coherence
c. Nave Realism
b. Consistency
d. Pragmatism
134
It is belief that when an idea agrees
with its object, it is proof of its truth.
However, it is definition of truth not a
criteria
a. Coherence
c. Nave Realism
b. Consistency
d.
Correspondence
135
The belief that the ultimate criterion
of truth is if an idea works then it is true
a. Coherence
c. Nave Realism
b. Consistency
d. Pragmatism
136
It is a systematic consistent
explanation of all the facts of experience.
Its technical name is reason. This is
believed to be the ultimate criterion of
truth
a. Coherence
c. Nave Realism
b. Consistency
d. Pragmatism

137
This educational philosophy
encourages accumulation of knowledge
and thinking and must apply criteria for
moral evaluation. Suggested methods are
questioning and discussion, lecture and
the project, whether done singly or in
group
a. Existentialism
c. Materialism
b. Idealism
d. Pragmatism
138
This theory views education as a
recurring process based on eternal truths;
thus, the schools curriculum should
emphasize the recurrent themes of
human life
a. Perennialism
c. Progressivism
b. Pragmatism
d.
Reconstructionism
139
The philosophical study of human
mind
a. Axiology
c. Metaphysics
b. Epistemology d. Rational psychology
140
The philosophical study of being, its
nature and essence
a. Axiology
c. Metaphysics
b. Epistemology
d. Rational
Psychology
141
A sociologist would be most interested
in studying
a. Artifacts of the fast civilization
b. Group behavior
c. Psychological problems
d. Economic institution
142
Interrelated social structures that fit
together to form an integrated whole
a. Structural functionalism
b. Conflict theory
c. Symbolic Interactionism
d. Disruptive Functionalism
143
Development of self and adaptation of
individual to society
a. Structural functionalism
b. Conflict theory
c. Symbolic Interactionism
d. Disruptive Functionism
144
Which is NOT a method in sociological
inquiry
a. Experimental study c. Intuitive
study
b. Longitudinal Study d. Ex-post Facto
study

145
Which of the following give some
difficulties to a sociological researcher?
a. Complexity of phenomena
b. Constant change
c. Unpredictability of behavior
d. All of the above
146
A cluster of behavior patterns related
to the general culture of a society and yet
distinguishable from it
a. Sub-culture
c. Norm
b. Culture shock
d. mores
147
Which group would most likely have a
counter culture?
a. Musician
c. Singers
b. Drugs users
d. Dancers
148
The complex whole which includes
knowledge, beliefs, arts moral, laws,
customs and other capabilities and habits
acquired by man as a member of the
society
a. Norm
c. Culture
b. Folkways
d. Mores
149
When one consider the culture of
another group as inferior to his own, this
connotes
a. Ethnocentrism
c.
Assimilation
b. Cultural relativism
d. Acculturation
150
The rightness or wrongness of what
one does depend on where he is doing it
a. Cultural integration c. Cultural
relativism
b. Cultural lag
d. Cultural traits
151
The culture that includes undesirable
behavior of the member of society.
Formally condemned but widely practiced
a. Real culture
c. Sub-culture
b. Ideal culture
d.
Counter culture
152
The process by which a member
learns the norms of the group
a. Accommodation
c.
Compartmentalization
b. Aggregation
d. Socialization
153
A role or status assigned according to
heredity traits without regard to individual
preference, ability or performance
a. Achieved
c. Attached
b. Ascribed
d. Applied
154
An achieved position in a society
a. Heir to the throne
c. Movie actor
b. Presidential son
d. First
lady
155
Refers to the difficulty people have in
meeting their role obligations
a. Role conflict
c. Role strain
b. Role set
d. Role
ambiguity
156
When a supervisor is tasked to report
unsatisfactory performance of her coworkers who are also her friends. She
may experience
a. Role play
c. Role strain
b. Role conflict
d. Role
ambiguity
157
Social control imposed on social
deviants to maintain
a. Social stigma
c. Social order
b. Social strata
d. Social mobility
158
Which of the following is NOT true?
a. The deviant behavior of one
generation may become the norm of
the next
b. All forms of deviant are bad
c. Deviant behavior is one way of
adapting a culture to a social change

St. Louis Review Center-Inc-Davao Tel. no. (082) 224-2515

96

d. Some deviation are due to the failure


of the socialization process to
integrate the cultural norms into
individuals personality
159
A teenager in a good neighborhood
of stable families and conventional people
who rejects middle class norms and
become a delinquent is an example of
a. Individual deviation c. Counter
deviation
b. Group deviation
d. Relative
deviation
160
A case of compulsive drug addict is an
example if a
a. Primary deviant
c.
Tertiary deviant
b. Secondary deviant d. All of the
above
161
Which is NOT a characteristic of a
social group?
a. Physical collection of people
b. Shares a common purpose and
conscious of each other
c. Share some common characteristics
d. Member interact with one another
162
Which is NOT a characteristic of the
relationship among members of a primary
group?
a. Personal
c. Segmental
b. Informal
d. Sentimental
163
Which is an example of members if a
primary group?
a. Peer group
c. School
administration
b. Family
d. Neighbors
164
Which of the following statements is
NOT true about Filipino families today?
a. Male authority has declined
b. Division of labor has changed
c. Separation has become more common
d. Has cease to be socialization unit in
society
165
Which of the following is NOT an
ethical issue that must be confronted
when doing social research?
a. Are the subjects being paid enough?
b. What degree of risk, pain or harm is
involved?
c. To what extent are the subjects being
deceived?
d. Will there be disclosure of confidential
or personally harmful information?
166
In most instances of participant
observation the researchers:
a. Hide their true identity
b. Do not hide their true identity
c. Pay their informants for information
d. Acts as therapist to the subject
167
A research technique in which the
investigators enter to the activities of the
group at the same time they study the
groups behavior is
a. Participants observation
b. A semi-structured (open-ended)
Interview
c. A structured interview
d. A data discussion
168
Participant observation:
a. A research tool used to follow-up on
expected findings
b. A research interview in which the
investigators ash a list of questions,
but is free to vary them or make up

new ones that become important


during the course of the interview
c. A research technique in which the
investigators enter into activities of
the group at the same time they study
the groups behavior
d. A research interview determined
entirely in advance and followed
rigidly
169
A research interview determined
entirely in advance and followed rigidly is:
a. Participants observation
b. A semi structured (open-ended)
interview
c. A structured interview
d. A data discussion
170
A research interview in which the
investigators ask a list of questions, but is
free to vary them or make up new ones
that become important during the course
interview, that is
a. Participant observation
b. A semi structured (open-ended)
interview
c. Structured interview
d. A data discussion
171
A conversation between two or more
individuals in which one party attempts to
gain information from others(s) by asking
of questions is known as;
a. A survey
c. Participant
observation
b. An interview d. Hypothesis testing
172
Sociological and anthropological be
misused because it:
a. Is hard to understand
b. Is often used selectively by groups or
individuals to support certain motives
c. Is often ambiguous
d. Has few applicants
173
A survey by their nature usually
a. Are cross-sectional
b. Are longitudinal
c. Involve secondary methodology
d. Involve participants observation

174
A cross-sectional study is
a. A research technique in which
investigators enter into activities of
the group at the same time they study
the groups behavior
b. Research examines a population at a
given point in a time
c. Research which examines a
population, or portion thereof, is
questioned in order to reveal specific
facts about itself
d. Research that investigates a
population at several intervals over
relatively a long period of time
175
Research that investigates a
population at several intervals over a
relatively long period of time is called
a. A cross-sectional study
c.
Secondary Analysis
b. A residual analysis d. Longitudinal
research
176
Survey are used when:

St. Louis Review Center-Inc-Davao Tel. no. (082) 224-2515

97

a. Other measure do not produce


statistical significance
b. The findings of the study must be
repeatable
c. It is desirable to discover he
contribution and interrelations of
certain variables among large number
of people
d. Procedures having cross-cultural
applications needed
177
A research method is which a
population, or portion thereof, is
questioned in order to reveal specific
facts itself is known as:
a. A survey
c. Experiments
b. Participant observation
d.
Residual analysis
178
Which of the following is NOT one of
the three main methods of research used
by sociologists?
a. A survey
c. Experiments
b. Participants observation
d.
Residual analysis
179
A statistical technique to make all
significant groups in as society
represented in a sample proportion to
their numbers in the large society is
known as:
a. Random sampling
b. A stratified random sample
c. Sampling
d. Randomized response
180
The goal in designing a research
project should be:
a. To prove a point
b. To provide a collection for all
necessary and sufficient data to test
the stated hypothesis
c. To selectively isolate the variables in
order to prove the hypotheses are
either correct or incorrect
d. To prove hypothesis false. If they
cannot proven false, they must be
true
181
An independent variable is
a. A testable statements about the
relationship between two or more
empirical variables
b. Anything that can be change
c. A variable that changes for reason
that have nothing to do with another
variable
d. A variable that changes in response to
changes in another variable
182
Which of the following is not a sign?
a. A clenched first
c. A flag
b. A knock on the door d. Yawning
183
The most important symbols are
a. Action
c. Actions
b. Words
d. Behavior
184
The principal means through which
culture is transmitted from generation to
generation is
a. Actions
c.
Language
b. Diffusion
d. Behaviors
185
If apes can be taught to use language,
it denies the views that:
a. Only human can think
b. Only human have culture
c. Apes do not have history to pass on
d. Apes are stupid
186
Which of the following statements
about culture is not true?
a. Every social group must have a
culture on its own in order to function

b. Every individual participates in a


number of different culture
c. Meeting the social expectation of
several cultures is often source of tree
d. Families do not have their own culture
but instead reflect the culture of the
larger society
187
Promised to make this country great
again. Ruled for about twenty years
a. Diosdado Macapagal c. Ramon
Magsaysay
b. Ferdinand Marcos
d. Carlos Garcia
188
The form of government established
by Aguinaldo was change from dictatorial
to:
a. Democratic
c. Republic
b. Sociologist
d. Revolutionary
189
Which provides he Tagalog shall be
the official language of the republic?
a. Constitution of 1935 c. Biyak na Bato
b. Makabula
d. Malolos
190
Which of the following were the
greatest naval battles in history during
Japanese occupation of the Philippines?
a. Battle of Leyte gulf c. Claveri
b. Surigao Strait
d. Dela Torre
191
The most cultured of the reformist,
wrote a socio-historical novel based on
facts he gathered in the Philippines
a. Marcelo H. Del Pilar c. Mariano Ponce
b. Jose Rizal
d. Jose Ma.
Paganiban
192
The editor of La Solaridad and a great
orator, he initiated the reform movement
a. Marcelo H. Del Pilar c. Emilio Jacinto
b. Jose Rizal
d. Graciano
Lopez-Jaena
193
The generation of the spirits was
common among ancient Filipinos.
Memories of their dead relatives were
kept alive through craved idol. This
practice was called
a. Cult of the dead
c. Nature
Worship
b. Mass of the dead d. Divination
194
The practice of having surnames was
started during Spanish regime through
the order of
a. Gov. Gen Jaudenes c. Gov. Gen
Claveria
b. Gov. Gen Dela Torre d. Gov. Gen
Polavejia
195
As promised by Gen. Mc Arthur- he
would return to liberate the Filipinos after
he could train troops, in Australia. The
Leyte landing signified the liberation of
the Philippines, was well as the return of
the commonwealth government. Who was
the commonwealth president who came
with Mc Arthur?
a. Romulo
c.
Osmea
b. Roxas
d. Quezon
196
Men and women of pre-colonial times
had weakness for personal adornment. A
side from jewels, tattooing of the body
was common. In the Visayas tattooed
men were called
a. Umalohokan
c. Abi
b. Pintados
d. Plista
197
Memories of brutality which was so in
human during the Japanese regime were
always associated with a particular
Japanese group called

St. Louis Review Center-Inc-Davao Tel. no. (082) 224-2515

98

a. Kalibapi
c.
Kamikaze
b. Kempetai
d. Samurai
198
Membership in the Katipunan come in
grades: first (katipons); second (kawal);
and third (bayani). Each grade designed
as well as password so they could
maintain their secrecy of the movement.
The password for the kawal grade was?
a. Rizal
c. Kawal
b. Gomburza
d. Sundalo
199
Pre-colonial trade in the Philippines
was prosperous. Business transactions
made use of:
a. Money system
c. Credit system
b. Barter system
d. Banking
System
200
The use of magic charms was very
common among pre-Spanish Filipinos.
One of these is the anting-anting or
agimat which is meant to:
a. Insure a man against a weapon of
every kind
b. Made a man lovable to all ladies
c. Made a man invisible
d. Made him walk in storm or sea
without getting wet
***** THE END *****
WORK HARD, DREAM HARDER
Social science
1
c
51
a
2
a
52
d
3
b
53
b
4
b
54
c
5
c
55
c
6
b
56
b
7
d
57
c
8
a
58
b
9
a
59
a
10
c
60
a
11
d
61
a
12
c
62
c
13
a
63
c
14
a
64
d
15
b
65
a
16
c
66
a
17
d
67
a
18
a
68
c
19
a
69
c
20
c
70
b
21
c
71
b
22
b
72
b
23
c
73
c
24
b
74
b
25
d
75
d
26
b
76
c
27
d
77
a
28
a
78
d
29
c
79
b
30
c
80
b
31
d
81
c
32
a
82
c
33
c
83
d
34
c
84
d
35
d
85
b
36
a
86
d
37
b
87
b
38
a
88
d
39
d
89
a
40
c
90
a

41
42
43
44
45
46
47
48
49
50

b
a
c
c
d
c
b
c
a
c

91
92
93
94
95
96
97
98
99
100

b
d
a
c
d
a
b
b
b
a

PRINCIPLES AND STRATEGIES OF


TEACHING

St. Louis Review Center-Inc-Davao Tel. no. (082) 224-2515

99

POST-TEST SEPTEMBER 2010


1.

It has reference to what teachers do in


planning, implementing and evaluating
instruction.
a. Teaching
c. Teaching strategies
b. Curriculum d. Instruction
2. The orderly process directing learners to
develop their skills and habits so that
they will be assisted in acquiring
knowledge and attitudes.
a. Instructional Media
b. Instructional Method
c. Teaching Techniques
d. Instructional System
3. Facial Expression, writing on the board,
and oral expression of the teacher is an
example of
a. Teaching Behavior
b. Technical Skills of teachers
c. Instruction
d. Instructional System
4. Learning to draw, drive a car, play
tennis, cook and type a poem often
taught in is an example of
a. Cognitive Learning c. Verbal
Learning
b. Motor Skill Learning d. Social
Learning
5. Responding to telephone calls, writing
ones name, reading a book orally is an
example
a. Cognitive Learning c. Verbal
Learning
b. Motor skill learning d. Serial
Learning
6. A process wherein the pupils attention
and interest are aroused and directed to
a definite purpose.
a. Learning
c. Method
b. Motivation
d. Principle
7. Contains a statement of results to be
accomplished and specific means by
which these results are to be attained
under direction and guidance.
a. Method
c. Technique
b. Lesson Plan
d. Principle
8. Could be the means of developing good
study habits and independence in work
as well as preparing the pupils for the job
to be done
a. Review
c.
Assignment
b. Drill
d. Recitation
9. The act of repeating from memory the
reciting of a lesson and often described
as a session lesson hearing
a. Review
c. Assignment
b. Recitation
d. Drill
10. A teaching procedure dealing with firsthand experiences pertaining to material
obtained from experimentation
a. Demonstration Method
b. Laboratory Method
c. Discovery Method
d. Deductive Method
11. Starts with generalization and principles
or from general to particular
a. Inductive Method
c. Classical
Method
b. Deductive Method d. Problem
Method
12. Students enact situations that arise in
daily living, where values may be
clarified, insights are developed and
decision-making is practiced

13.

14.

15.

16.

17.

18.

19.

20.

21.

22.

23.

24.

a. Simulation Game
c.
Demonstration
b. Role Playing d. Inquiry Process
Encouraging students to search for and
see relationships that are not obvious;
also it stretches the intellect of students
a. Open-ended Questions
b. Recall Questions
c. Explanatory Questions
d. Descriptive Question
It is control by enforcing obedience or
orderly conduct or training that corrects
and strengthens?
a. Management
c. Techniques
b. Discipline
d. Strategies
When students are asked to respond to
incomplete statements or questions that
are presented in oral/ written form
a. Open-ended Statement
b. Close-procedure
c. PAC Strategy
d. Structured Activity
These are all the experience which
children have under the direction of a
school
a. Curriculum
c. Learning
b. Instruction
d. Socialization
The subjects mater, not the child is
important in this type of curriculum
a. Correlated curriculum
b. Subject-centered curriculum
c. Experience curriculum
d. Fused curriculum
The child-instead of the subject-matter is
important in this kind of curriculum
a. Correlated curriculum
b. Core curriculum
c. Experience curriculum
d. Fused curriculum
It is a unified curriculum where subject
matters from different subject field are
treated unitary of the same curriculum
a. Core curriculum
b. Integrated Curriculum
c. Broad field curriculum
d. Fused curriculum
Teachers initiative, imagination, puppet
shows, play, reading and animated
cartoons can be examples of enriching
the curriculum under these resources
a. Specializing Resources
b. Creative Resources
c. Human Resources
d. Reading Resources
A curriculum considered basics for all
students, that all must get them
a. Broad field curriculum
b. Core curriculum
c. Integrated Curriculum
d. Experience Curriculum
The whole body of experience utilized by
the school to attain the aims of
education
a. Psychology c. Socialization
b. Curriculum d. Methods
Formal education starts when the child
a. begins to talk
b. reaches the age of six years old
c. first enters school
d. begins to be inquisitive
That aspect of curriculum that has to do
with the preservation of the best in our
culture, customs and traditions has been
borrowed from
a. Sociology
c. Psychology
b. Sociometry
d. Ethics

St. Louis Review Center-Inc-Davao Tel. no. (082) 224-2515

100

25. The curriculum must take into


consideration the
a. aim of education
b. learning process
c. motives and incentives
d. instincts
26. The curriculum is
a. all-embracing
c. all power
b. encompassing
d. selective
27. In the traditional school, the focus of
attention was on the
a. child
c. method
b. subject matter
d. book
28. Curriculum objects are formulated in the
light of our
a. past history
b. educational policy and philosophy
c. experience as a nation
d. needs in school
29. Which of the following questions
encourages reflective thinking?
a. What are the parts of a complete
flower?
b. What do we use to observe matter?
c. In what ways can help his community
d. Why are machine-made goods
cheaper than those made by hands?
30. The success of the pupils in formulating
generalization greatly depends on:
a. the interest of the pupils
b. the devices used
c. the subjects matter
d. the teachers skillful questioning
31. In the inquiry method, the initiation
phase calls for the teacher to set the
stage for:
a. finding solutions to problem
b. raising of problems
c. gathering data
d. formulating generalization
32. Method is dependent upon:
a. classroom techniques
b. teachers expectation
c. theoretical assumptions
d. available textbooks
33. To lead the students to the desired
behavior, method must be implemented
through:
a. selected technique
b. the curriculum
c. the discussion of the teacher
d. careful observation
34. Which is not true regarding the project
method?
a. Many worthwhile projects are
impossible because of the materials
needed
b. The project method should be used
occasionally but not regularly
c. The pupil or the class should carry
the chief responsibility of planning
the project
d. The project method is adaptable to
all units in the curriculum
35. The laboratory method is also called:
a. the research methods
b. the deductive method
c. the development method
d. the problem method
36. In the unit method, actual learning takes
place in:
a. orienting the pupils
b. collecting, discovering and recording
data
c. summarizing the unit

d. organizing the unit or study


37. A statement of objectives, learning
experience and the means of attaining
results of teaching is called
a. procedure
c. outcomes
b. lesson plan
d. strategy
38. Teaching aids which the teacher uses to
make learning meaningful, productive
and interesting is known as:
a. device
b. technique
c. method
d. learning continuum
39. Teaching method which proceeds from
the details of a lesson towards the
generalization is called:
a. Inductive
c. problemsolving
b. deductive
d. debate
40. A teaching method which proceeds from
a generalization, principle or rule is:
a. inductive
c. project
b. deductive
d. process
41. The recent approach in teaching Social
Studies is called
a. discovery
c. process
b. conceptual
d. formaleducation
42. A method of teaching which aptly applies
to lessons needing experiments is called:
a. problem-solving
c. observation
b. laboratory
d. demonstration
43. What type of lesson is presented wherein
the learner meets the learning
experience through understanding,
analysis, and generalizations of facts
presented?
a. review
c.
developmental
b. drill
d. deductive
44. What lesson is presented when the
teacher takes up the previous learning
experiences of the learners in a
recognized pattern of presentation?
a. Drill
b. developmental
c. review
d. discussion procedure
45. A lesson which aims to focalize skills to
make them fixed to the point of mastery
is
a. problem-type
c. review
b. drill
d. experimental
46. The law of exercise is aptly applied in a
a. review lesson
c. drill lesson
b. assignment
d. check-up
47. A type of review which presents the sumtotal of all activities previously presented
a. integrated
c. daily
b. cumulative
d. drill
48. What recent technique o teaching calls
for acting out of a situation where the
participants aim to uncover a problem of
great importance to the class?
a. panel
c. role-playing
b. debate-form
d. lecture-form
49. What technique of in-service training for
teachers involves the identification and
solution of common problems by them,
thru live-in sessions, conferences, and
speeches of consultants?
a. buzz session c. seminar
b. workshop
d. professional meeting
50. The non-verbal symbols used to
maximize learning are referred to as

St. Louis Review Center-Inc-Davao Tel. no. (082) 224-2515

101

51.

52.

53.

54.

55.

56.

57.

58.

59.

60.

a. Instructional devices
b. Classrooms techniques
c. Field trips
d. Educational media
Graphic material which are eye-catching
and which use slogans and topics
presented in bold letterings and strong
colors to serve as reminders of standards
and / or important events are called
a. poster
c.
projector
b. film strips
d. objects
What contemporary aid to teaching
utilizes carefully-planned materials
where each step of learning requires
repetition and practice until such step is
thoroughly learned?
a. programmed instruction
b. Keypunching
c. Educational Television
d. Educational hardware
The Stimulus-Response theory of
learning which involves the association
between a conditioned stimulus and a
response thru the repeated presentation
of the stimulus was advocated by whom?
a. Edward Thorndike c. Burrhus
Skinner
b. Ivan Pavlov
d. Wolfgang
Kohler
What plan of promoting pupils is
committed to encouraging the learners
to progress from grade to grade without
needless repetition
a. non-graded scheme
b. individualized
c. heterogeneous grouping
d. acceleration
Differentiated assignments, tutorial and
remedial work to would-be-failures are
not considered in the individualized
Instruction Scheme
a. Yes
c. Maybe
b. No
d. Sometimes
A part of a daily lesson which serves as
a carry-over for the next day of what has
been presented is the
a. review
b. drill
c. assignment or agreement
d. lesson proper
A good learning environment is one
a. free from distraction
c.
disturbing noise
b. aver decorated
d.
dilapidated
The proper handling of the physical
condition ad instructional materials in
the classroom to effect learning refers to
a. teaching method
b. Classroom management
c. Discipline grouping
d. Guidance-oriented
What refers to the process o directing
immediate personal desires, interests or
wishes for the purpose of achieving an
effective action?
a. discipline
c. supervision
b. teaching
d. management
What characteristics an effective type of
discipline?
a. vital, sympathetic, humane
b. formal and strict
c. inhibited
d. imposed

61. Which of these is not a quality of a good


teacher?
a. mastery o the subject matter
b. broad background of liberal
education
c. aims to enrich himself thru teaching
d. understand the nature of the learners
62. Which of these is a good personal
qualification of a teacher?
a. resourceful, creative and intelligent
b. rich, capricious and luxurious
c. complaining, demanding and scornful
d. materialistic
63. Which of these is included among the
professional ethics o school teachers?
a. professional jealousy
b. integrity
c. engaging in business pre-judicial to
his teaching duties
d. gossip mongering

64. What teaching method helps the learners


draw generalization from a discipline
with the end in view of applying the
same similar situations in the future?
a. discovery approach
b. process approach
c. conceptual approach
d. problem-solving approach
65. Which subjects is in the elementary and
secondary school levels mostly
concerned with the study of societal
problems and issues which are
significant to the learners as member of
society?
a. Modern Mathematics
b. Social Studies
c. Filipino
d. Character Education
66. Which of these are considered with two
essential dimension of science teaching?
a. observing and inferring
b. seeing and observing
c. reading and researching
d. knowledge and performance
67. Which of these is not a process in
science teaching?
a. Measurement
b. Communication skill
c. Controlling variables
d. None of these
68. Of the process involve in the modern
approach to science instruction, which
one utilizes the most number of scientific
processes.
a. prediction
c. inference
b. experimentation
d. hypothesis
69. Give the main difference of these two
objectives:
to teach the importance of proper
nutrition for good health
to give the importance of proper
nutrition for good health
a. The first objective is general while
the second is specific.
b. The first objective is hard to do while
the second is easy
c. The first objective needs a longer
time while the second doesnt
d. The first objective is teacher behavior
while the second pupil behavior.

St. Louis Review Center-Inc-Davao Tel. no. (082) 224-2515

102

70. Which of the objectives below show


overt behavior?
a. To appreciate the value of democracy.
b. To understand the importance of a
constitution
c. To recite he preamble of the
constitution
d. To show love to ones country
71. The basis by which content is outlined
and institutional procedures are
developed is the:
a. lesson plan c. objectives
b. basic text
d. instructional
materials
72. An objective MUST specify:
a. What the learner must do or say.
b. What the teacher must do or say
c. What projects are to be accomplished
d. What the learner must understand
73. Given ten photographs of biological
cells, the pupils will be able to identify
six of them as plant or animal cells. The
underlined phrase is a :
a. terminal behavior
b. standard or acceptable performance
c. condition for learning
d. an accomplishment to be realized.
74. To make statement as an objective in
an English Lesson that is:
a. specific
c. correct
b. vague
d. none of the above
75. What is the most fitting condition of
learning for this behavior: to conclude
that plants need sunlight in order to
live?
a. with the must of materials
b. given a set of pictures
c. after reading the book
d. realistic
76. Which of the following is not a criterion
of a well-formulated objective?
a. attainable
c. interesting
b. observable
d. realistic
77. Which task below is not in the
psychomotor domain?
a. imitation
c. manipulation
b. evaluation
d. articulation
78. The growth of attitudes or values is in
the:
a. cognitive domain
b. psychomotor domain
c. affective domain
d. behavioral domain
79. The domains of behavior do not come in
isolation. This statements is :
a. True
c. False
b. Acceptable
d. Partly true
80. Will a person do it freely without any
type of coercion? This is:
a. a cognitive question
b. an affective question
c. a psychomotor question
d. a behavioral question
81. To develop appreciation of poetry is a :
a. general aim
c. nature aim
b. specific aim
d. serious aim
82. Which aim below does not belong to the
group?
a. To enumerate the uses of common
garden tools
b. To express opinion politely
c. To explain the significance of the
story
d. To identify the parts of a flower.

83. Which objective below is not realistic?


a. To respect places of worship
b. To sing the national anthem correctly
c. To give the importance of cleanliness
d. To cite ways to show love ones
country
84. Which objective below is not specific?
a. To describe some of farming
procedures
b. To define terms comprehensively
c. To pay tax promptly
d. To know the life cycle of a moth.
85. What phrase below is a standard of
performance?
a. Solve the problem correctly within 10
minutes
b. Identify and sketch the curve
c. With the use of a ruler
d. After several examples
86. A visible activity shows :
a. overt behavior
b. covert behavior
c. confident behavior
d. artificial behavior
87. Which infinite below is not behavioral?
a. to describe
c. to compare
b. to select
d. To believe
88. Which objective below needs
improvement
a. To prepare a seed box
b. To develop skill in embroidery
c. To plan a noon meal
d. To make an apron
89. Which of the following statements is
correct?
a. Method is probably more important in
college than in the elementary
b. Method is more important in the
elementary than in high school or
college
c. Method is more important in college
than in high school
d. Method is less important than a
lesson plan
90. What encourages the child to think,
rationalize and make proper decisions?
a. drill
b. Appreciation lesson
c. Memorization
d. Problem- oriented strategies
91. The following except one are the factors
that determine the choice of a method.
Which is the exception?
a. nature o the learners
b. school equipment and facilities
c. educational background of the
teacher
d. Subject matter
92. How well a teacher tells a story depends
on:
a. Techniques c. the method used
b. the plot
d. classroom
93. Which of the following statements is
correct?
a. Method is synonymous with
technique
b. A device is a teaching method
c. Method can be standardized
d. There is no single best method
94. When a teacher reviews a lesson, she is
utilizing the law of:
a. Readiness
c. effect
b. exercise
d. multiple
response

St. Louis Review Center-Inc-Davao Tel. no. (082) 224-2515

103

95. In which situation is the law of readiness


best applied?
a. The teacher gives the aims of the
lessons to be taken up
b. The teacher announces he subject
matter at the start of the period
c. The teacher waits or the children to
be ready before teaching her lesson
d. The teacher presents a song, related
to the lesson
96. Which of the glowing is not an am in the
inductive method
a. To delay judgments until truth is
given
b. To enable pupils discover important
truths for themselves.
c. To help student/pupil to carry out an
investigation by themselves
independent of the teacher
d. To make relationship of ideas clear to
pupils
97. In the inductive method, what does the
child do during the comparison and
abstraction step?
a. Recalls information and directs
himself to the activities to be
accomplished
b. Perceives the common element
present in the cases given
c. Applies the principles learned to
other problems or exercises
d. Draw conclusion in his own words
98. The deductive method uses the following
steps:
a. statement of the problem,
generalization, inference, verification
b. statement of the problem, inference ,
generalization, verification
c. inference, statement of the problem,
generalization, verification
d. inference, statement of the problem,
verification, generalization

14
15
16
17
18
19
20
21
22
23
24
25
26
27
28
29
30
31
32
33
34

b
a
a
b
c
b
b
b
b
c
a
b
a
b
b
d
d
b
c
a
d
35
36
37
38
39
40
41
42
43
44
45
46
47
48
49
50

64
65
66
67
68
69
70
71
72
73
74
75
76
77
78
79
80
81
82
83
84
a
b
b
a
a
b
a
b
c
c
c
b
b
c
a
b

b
b
a
d
b
d
c
c
a
c
b
d
d
b
c
a
b
a
b
a
d
85
86
87
88
89
90
91
92
93
94
95
96
97
98
99
100

a
a
d
b
b
d
c
a
d
b
d
a
b
a
a
a

99. In reality, the type of study method is:


a. an inductive procedure
b. a deductive procedure
c. a traditional method
d. a question and answer method
100.
The Herbartian formal steps
corresponds to the steps of:
a. the inductive method
b. teaching an appropriate lesson
c. the deductive method
d. the project method

A thing becomes wasted when you fail to


take advantage of it
Principle
1
2
3
4
5
6
7
8
9
10
11
12
13

and strategies
c
51
b
52
a
53
b
54
c
55
b
56
b
57
c
58
b
59
b
60
b
61
b
62
a
63

a
a
b
a
c
a
a
b
c
c
a
a
b

St. Louis Review Center-Inc-Davao Tel. no. (082) 224-2515

104

PROFESSIONAL EDUCATION
Pre-board EXAM
April 2010
MULTIPLE CHOICE
1. You are very much interested in a quality
professional development program for
teachers. What characteristic should you
look for?
a. Prescribe by top educational teachers
b. Dependent on the availability of funds
c. Required for renewal of professional
license
d. Responsive to identified teachers
needs.
2. To ensure high standards of teachers
personal and professional development,
which of the following measures must be
implemented?
I. A school head plans the professional
development of his/her teachers.
II. Every teacher formulates his/her own
professional development plan
III. The implementation of what is leaned
in a training must be monitored.
a. I only
II and III
b. I and III
d. II only
3. As a community leader, which of the
following should a teacher NOT do?
a. Support effort of the community to
improve their status in life.
b. Make herself aloof to ensure that her
decisions will not be influenced bu the
community politics.
c. Solicit donation from philanthropists in
the community.
d. Play an active part in the activities of
the community.

4. In a highly pluralistic society, what type of


learning environment is the responsibility
of the teacher?
I. Safe
II. Gender-biased
III. Secure
a. I and II
c. II only
b. I, II and III
d. I and III
5. A teacher is said to be trustee of the
cultural and educational heritage of the
nation and is under obligation to transmit
to learners such heritage. Which practice
makes the teacher fulfill such obligation?
a. Use interactive teaching strategies.
b. Use the latest educational technology.
c. Observe continuing professional
education
d. As a class, study the life of Filipino
heroes.
6. Which actions show that political factors
affect schools as agents of change?
I.
The strengthening of the teaching of
English in Philippines school.
II.
The introduction of mandated subjects
such as Rizal in the curriculum
III.
The practice of mainstreaming
IV.
The turnover of day care centers for
DSWD to DepEd for supervision.
a. I and III
c. II and III
b. I and II
d. II and IV
7. For more efficient and effective
management of school as agents of
change, one proposal is for the DepEd to
cluster remote stand-alone schools under
one lead school head. Which factor has
the strongest influence on this proposal?
a. Psychological
c. Geographical
b. Historical
d. Social

8. What does the acronym EFA imply for


schools?
a. The acceptance of exclusive schools
for boys and for girls.
b. The stress on the superiority of formal
education over that of alternative
learning system.
c. Practice of inclusive education
d. The concentration on formal
education system
9. The wide acceptance of bottom up
management style has influenced schools
to practice which management practice?
a. Exclusion of politicians from the pool
of guest speakers during graduation
exercises.
b. Prescription of what ought to be done
from the Center Office.
c. Involvement of students, parents,
teachers, and community in school
planning
d. Allowing schools to do what they think
is best
10. Large class size in congested cities is a
common problem in our public schools.
Which measure/s have schools taken to
offset the effects of large class?
I.
The deployment of more teachers
II.
The implementation of 1:1 pupil
textbook ratio
III.
The conduct of morning and
afternoon sessions
a. I, II and III
c. III only
b. I and II
d. II only

St. Louis Review Center-Inc-Davao Tel. no. (082) 224-2515

105

11. The failure of independent study with


most Filipino students may be attributed
to students
a. unpreparedness fro schooling
b. ambivalence
c. high degree of independence
d. high degree of independence on
authority
12. The schooling incidents in school
campuses abroad have made school to
rethink the curriculum. Which is believed
to counteract such incidents and so is
being introduced in schools?
I. Inclusion of socio-emotional teaching
II. The emphasis on the concept of
competition against self and not
against others
III. Focus on academic achievement and
productivity
a. I and III
c. I and II
b. II and III
d. I, II and III
13. Widespread abuse of Mother Earth
prompted schools to teach sustainable
development. which one does this prove
about schools?
a. The curricula of schools are centered
on Mother Earth.
b. Schools can easily integrate sustained
development in their curriculum.
c. Sustained development cannot be
effectively taught in the classroom.
d. Environment factors influence the
school as an agent of change.
14. A father tells his daughter You are a
woman. You are meant for the home and
so for you, going to school is not
necessary. Is the father CORRECT?
a. It depends on the place when the
daughter and the father live.
b. Yes, women are meant to be a mother
only.
c. No. today women can take on the jobs
of men.
d. No, there is gender equality in
education.

15. Is there a legal basis for increasing the


teachers starting salary to P18,000 an
months?
a. No, it is a gift to teachers from
Congress
b. Yes, R.A 7836
c. No, it is simply an act of benevolence
from President GMA
d. Yes, the Phil. Constitution
16. As provided for the Educational Act of
1982, how are the institutions of learning
encourage to set higher standards of
equality over the minimum standards
required for state recognition?
a. Granting of Special Permit
b. Academic freedom
c. Continuing Professional Education
d. Voluntary accreditation
17. Despite of opposition from some school
official, DepEd has continuously enforced
the no collection of fees policy during
enrolment period in public schools. In this
policy in accordance with EFA goals?
a. No, it violates the mandate of equality
education

b. Yes, it somewhat eliminates gender


disparities
c. Yes, it supports equitable access to
basic education
d. No. it does not support parent of
adult education
18. Specialization is knowing more and more
about less and less. Hence, it is better to
be a generalist, claims Teacher F. Which
Philosophy does Teacher F. subscribe to?
a. Existentialism
c. Essentialism
b. Perennialism
d. Progressivism
19. Mencius believed that all people are born
good. This thought on the innate
goodness of people makes it easier to
our pupils.
a. teach
c. like
b. Respect
d. motivate
20. The specialization requires of every
professional teacher for him/her to be
competent is in line with which pillar of
learning?
a. Learning to know
b. Learning to be
c. Learning to live together
d. Learning to do
21. Which pillar of learning is aimed at the
wholistic development of man and his
complete fulfillment?
a. Learning to be
b. Learning to know
c. Learning to live together
d. Learning to do
22. Material development at the expense of
human development points to the need to
do more in school.
a. Learning to do
b. Learning to know
c. Learning to live together
d. Learning to be
23. A student complains to you about his
failing grade. When you recomputed you
found out that you committed an error in
this grade computation. Your decision is
not accept the erroneous computation
before the student and so leave the
failing grade as is for fear that you may
lose credibility. Is this morally right?
a. No, the reason for not accepting the
error before the students in flimsy.
b. No, the end does not justify the
means
c. Yes, the end justifies the means
d. Yes, as a teacher you must maintain
your credibility
24. Which violate(s) the principle of respect?
I.
Teacher A tells her students that what
Teacher B taught is wrong.
II.
To retaliate, Teacher B advises
students not to enroll in Teacher As
class.
III.
Teacher C secretly gives way to a
special favor (e.g. add 2 points to
grade) requested by student A who is
vying for honors.
a. II and III
c. I and II
b. I, II and III
d. I and III
25. Which is/are in accordance with the
principle of pedagogical competence?
I. Communication of objectives of the
course to students
II. Awareness of alternative instruction
strategies
III. Selection of appropriate methods of
instruction

St. Louis Review Center-Inc-Davao Tel. no. (082) 224-2515

106

a. I and III
c. III only
b. I, II and III
d. II and III
26. With a death threat over his head,
Teacher D is directed to pass an
undeserving student. Which will a
utilitarianist do?
a. Pass the student, why suffer the
threat?
b. Dont pass him; live by your principle
of justice. You will get reward, if not in
this life, in the next.
c. Pass the student. That will be of use to
the student, his parents and you.
d. Dont pass him. You surely will not like
someone to give you a death threat in
other to pass.
27. Teacher A knows of the illegal activities of
a neighbor but keeps quiet in order not to
be involved in any investigation. Which
foundational principle of morality does
Teacher A fail to apply?
a. The end does not justify the means
b. The end justifies the means
c. Always do what is right
d. Between two evils, do the lesser evil
28. To earn units for promotion, a teacher
pays her fee but does not attend class at
all. Does this constitute professional
growth?
a. Not immediately but yes after
promotion
b. It depends on the school she is
enrolled in
c. No, it is simply earning MA units for
promotion
d. Yes, just enrolling in an MA program is
already professional growth
29. If a teacher asks more higher-order
questions, he has to ask more questions.
a. fact
c. convergent
b. close
d. concept
30. Misdemeanor has a ripple effect. This
implies that as a classroom manager, a
teacher
a. reinforces positive behavior
b. responds to misbehavior promptly
c. is consistent in her classroom
management practice
d. count 1 to 10 before she deals with a
misbehaving student
31. Based on Edgar Dales Cone of
Experience, which activity is farthest
from the real thing?
a. Watching demo
c. Video disc
b. Attending exhibit
d. Viewing
images
32. The students of Teacher Y scan an
electronic encyclopedia, view a film on
subject, or look at related topics at the
touch of a button right there in the
classroom. Which device/s des teacher
Ys class have?
a. Teaching machines
b. CD
c. Video disc
d. Videotaped lesson
33. Which is an INAPPROPIATE way to
manage off-task behavior?
a. Redirect a childs attention to task and
check his progress to make sure he is
continuing work
b. Make eye contact to communicate
what you wish to communicate
c. Move closer to the child to make him
feel your presence

d. Stop your class activity to correct a


child who is no longer on task
34. To be an effective classroom manager, a
teacher must be friendly but must at the
same time be
.
a. confident
c. analytical
b. businesslike
d. buddy-buddy
35. Which software is needed when one
wants to perform automatic calculations
on numerical data?
a. Database
b. Spreadsheet Program
c. Microsoft Word
d. Microsoft Powerpoint
36. Which of the following questions must be
considered in evaluating teacher-made
materials?
a. In the material new?
b. Does the material simulate
individualism?
c. Is the material expensive?
37. Kounin claims that with-it-ness is one of
the characteristics of an effective
classroom manager. What is one sign of
with-it-ness?
a. Giving attention to students who are
having difficulty with school work
b. Seeing only a portion of the class but
intensively
c. Knowing where instructional materials
are kept
d. Aware of whats happening in all parts
of the classroom
38. Which of these is one of the ways by
which the internet enables people to
browse documents connected by
hypertext links?
a. URL
b. Browser
c. Welcome page
d. World Wide Web
39. Which characteristics must be primary
considered as a choice of instructional
aids?
a. Stimulate and maintain students
interests
b. Suited to the lesson objectives
c. Updated and relevant to Filipino
setting
d. New and skillfully made
40. You can exhibit referent power on the first
day of school by
a. telling them the importance of good
grades
b. giving your students a sense of
belongingness and acceptance
c. making them feel you know what you
are taking about
d. reminding your students your
authority over them again and again
41. I would like to use a model to emphasize
particular part. Which of these would be
MOST appropriate?
a. Regalia
c. Stimulation
b. Audio recording
d. Mock up
42. What must a teacher do to ensure orderly
transitions between activities?
a. Allow time for the students to
socialize in between activities
b. Have the materials ready at the start
of the activity
c. Assign fewer exercise to fill the
allotted time
d. Wait for students who lag behind

St. Louis Review Center-Inc-Davao Tel. no. (082) 224-2515

107

43. The task of setting up routine activities


for effective classroom management is a
task that a teacher should undertake
a. as soon as the students have adjusted
to their schedules
b. on the very first day of school
c. every day at the start of the session
d. every homeroom day
44. Teacher S uses the low-profile classroom
control technique most of the time. what
does this imply?
a. She is reactive in her disciplinary
orientation
b. She manages pupils personalities
c. She reacts severely to a misbehaving
student
d. She stops misbehaving without
disrupting lesson flow
45. When teacher tries to elicit clarification
on a student response or solicits
additional information, which of these
should be use?
a. Directing
c. Structuring
b. Probing
d. Cross
examining
46. Which priority criterion should guide a
teacher in the choice of instructional
devices?
a. Novelty
c. Attractiveness
b. Cost
d.
Appropriateness
47. Which learning activity is MOST
appropriate if a teachers focus is
attitudinal change?
a. Fieldtrip
c. Role play
b. Exhibit
d. Game
48. Teacher H strives to draw participation of
every student into her classroom
discussion. Which of these student needs
is she trying to address? The need to
.
a. shoe ones oral abilities to the rest of
the class
b. feel significant and be part of a group
c. get everything and be part of a group
d. be creative
49. Instead of teacher giving this comment a
student response. Youre on the wrong
track!, what should be teacher do?
a. Change the question to an easier one
b. Redirect the question by calling
another student to recite
c. Probe to redirect the response into a
more productive area
d. Pause, ask the question, lecture, then
ask the question again
50. If curriculum is designed following the
traditional approach, which feature(s)
apply(ies)?
I.
The aims of the curriculum are set by
professionals and experts
II.
Interested groups (teachers,
students, communities) are assumed
to agree with the aims of the
curriculum
III.
Consensus building in not necessary
a. III only
c. I, II
b. I, II, III
d. I, III
51. I want my student to develop the ability
to look at a problem from various
perspectives. Which approach will be
MOST fit?
a. Behaviorist approach
b. Computer-based Education
c. Modular approach
d. Cognitive approach

52. Ones approach to teaching is influenced


by Howard Gardners MI Theory. What is
he/she challenged to do?
I.
To come up with 9 different ways of
approaching lesson to cater to the 9
multiple intelligence
II.
To develop all students skill in all nine
intelligences
III.
To provide worthwhile activities that
acknowledge individual difference in
children
a. I, II and III
c. II only
b. II, III
d. III only
53. If my approach to my lesson is
behaviorist, what features will dominate
my lesson?
I.
Copying notes
III. Lecturing
II.
Reasoning
IV.
Demonstration
a. III, IV
c. I, II, III, IV
b. I, III, IV
d. II, III, IV
54. You practice inclusive education. Which of
these applies to you?
I. You accept every student as full and
valued member of the class and
school community
II. Your special attention is on learners
with specific learning or social needs
III. Your address the needs of the class as
a whole within the context of the
learners with specific learning or
social needs
a. II only
c. I only
b. I and II
d. I and III
55. School curriculum reflects the worlds
economic and political integration and
industrialization. What does this point in
curriculum development?
a. The trend towards the classical
approach to curriculum development
b. The trend toward the globalization
and localization
c. The trend toward participatory
curriculum development
d. The shift in the paradigm of
curriculum development from a
process-oriented to a product-oriented
one
56. You choose cooperative learning as a
teaching approach. What thought is
impressed on your students?
a. Interaction is a must, but not
necessarily face to-face interaction
b. Students success depends on the
success of the group
c. Students individuality evaluate how
effectively their group worked.
d. The accountability for learning is on
the group not on the individual
57. What principle is violated by overusing
the chalkboard, as though it is the only
education technology available?
a. Isolated use
c. Variety
b. Flexibility
d. Uniformity
58. Which statement applies a CORRECTLY to
Edgar Dales Cone of Experience
a. The farther you are from the base, the
more direct the learning experience
becomes.

St. Louis Review Center-Inc-Davao Tel. no. (082) 224-2515

108

b. The farther you are from the bottom,


the more direct the learning
experience becomes.
c. The closer you are from the base, the
more indirect the learning experience
becomes
d. The closer you are from the base, the
more direct the learning experience
becomes

59. When more senses are stimulated,


teaching and learning become more
effective. What is an application of this
principle?
a. Appeal to students sense of
imagination
b. Use multisensory aids
c. Make your students touch the
instructional material
d. Use audiovisual aids because the eyes
and the eras are the most important
senses in learning.
60. Which is a classroom application of the
theory of operant conditioning?
a. Help student see the connectedness
of facts, concepts, and principles
b. Create a classroom atmosphere that
elicits relaxation
c. Reinforce a good behavior to increase
the likelihood that the learner will
repeat the response
d. Make students learn by operating
manipulatives
Read the following teacher-student
situation
61. TEACHER: Why is the process called
photosynthesis?
STUDENT: I dont know
Which questioning technique should
be the teacher use?
a. Clarification
c. Prompting
b. Multiple response
d. Concept
review
62. Here is the test item.
From the data presented in the
table, from generalizations that
are supported by the data.
Under what type of question
does this item fall?
a. Convergent
c. Application
b. Evaluative
d. Divergent
63. I want to teach concepts, patterns and
abstractions. Which method will be MOST
appropriate?
a. Discovery
c. Direct
instruction
b. Indirect instruction d. Problem
solving
64. Teacher A teaches English as a Second
Language. She uses vocabulary cards, fillin-the-blanks sentences, dialogues,
dictation and writing exercises in teaching
a lesson about grocery shopping. Based
on this information, which of the following
is a valid conclusion?
a. The teacher wants to make her
teaching easier by having les talk
b. The teacher emphasizing reading and
writing skills
c. The teacher is teaching in a variety of
ways because not all students learn in
the same manner
d. The teacher is applying Blooms
hierarchy of cognitive learning

65. Teacher A an experienced teacher, does


daily review of past lessons in order to
a. introduce a new lesson
b. reflect on how he presented the
previous lessons
c. provide his pupils with a sense of
continuity
d. determine who among his pupils are
studying
66. I combined several subject areas in order
to focus on a single concept for
interdisciplinary teaching. Which strategy
did I use?
a. Reading-writing activity
b. Thematic introduction
c. Unit method
d. Problem-centered learning
67. To teach the democratic process to the
pupils, Biag Elementary School decided
that the election of class officers shall be
patterned after local elections. There are
qualifications set for candidates, limited
period for campaign and rules for posting
campaign materials, etc. Which of the
following did the school use?
a. Symposium
c. Pole playing
b. Simulation
d. Philips 66
68. Which are effective methods in teaching
student critical reading skills?
I. Interpret editorial
II. Read and interpret three different
movie reviews
III. Read a position paper and deduce
underlying assumptions of the
position papers
a. II and III
c. I and II
b. I and III
d. I, II and III
69. Here is a test item
The improvement of basic
education should be the top
priority of the Philippine
government. Defend or refute
this position.
Under what type of question
does this test item fall?
a. Low-level
c. Analysis
b. Evaluative
d. Convergent
70. When I teach, I often engage in
brainstorming. Which do I avoid?
a. Break down barriers
b. Increase creativity
c. Generate many ideas
d. Selectively involves pupils
71. Teacher S teaches a lesson in which
students must recognize that is the
same 0.25. They use this relationship to
determine that 0.15 and 0.20 are slightly
less than . Which of the following
concept/s is/are being taught?
a. Numeration skills
b. Place value of decimals
c. Numeration skills of decimals and
relationships between fractions and
decimals
d. Relationship between fraction and
decimals
72. What is the best way to develop math
concept?
a. Solving problems using multiple
approaches
b. Solving problems by looking for
correct answer

St. Louis Review Center-Inc-Davao Tel. no. (082) 224-2515

109

c. Learning math as applied to


situations, such as being a tool of
science
d. solving problems by applying learned
formulas
73. After the reading of a selection in the
class, which of these activities can
enhance students creativity.
I.
Readers theater
II.
Reading aloud
III.
Silent reading
a. I and II
c. I only
b. II only
d. III only
74. Teacher C, a Reading teacher, advised he
class to read between the lines. What
does she want her pupils to do?
a. Make an educated guess
b. Determine what is meant by what is
stated
c. Apply the information read
d. Describe the characters in the story
75. To nurture students creativity, which
activity should a teacher AVOID?
a. Ask hat if questions
b. Ask divergent thinking questions
c. Emphasize the need to give right
answers
d. Be open to out-of-this-world ideas
76. Teacher R wants to develop his students
creativity. Which type of questions will be
MOST appropriate?
a. Synthesis questions
b. Fact questions
c. What if. questions
d. Analysis questions
77. In my attempt to develop creative
thinking skills, I want to test fluency of
ideas. Which activity for my students will
be MOST appropriate?
a. Solve this math problem
b. List animals covered with hair in 1
minute
c. Solve this puzzle
d. Compare pictures 1 and 2. Where are
the differences?
78. You want your students to answer the
questions at the end of a reading lesson.
What did I learn did?,What still puzzle
me?, What did I enjoy, hate accomplish
in the class today?,How did I learn from
the lesson?.Which of the following are
you asking them to do?
a. Work on an assignment
b. Make journal entry
c. Work on a drill
d. Apply what they learned
79. After reading an essay. Teacher B wants
to help sharpen her students ability to
interpret. Which of these activities will be
most appropriate?
a. Drawing conclusions
b. Making inferences
c. Getting the main idea
d. Listing facts separately from opinion
Read the following then answer the question
80. A man and his son are driving in a car.
The car crashes into a tree, killing the
father and seriously injury his son. At the
hospital, the boy needs to have surgery.
Looking at the boy, the doctor says
(telling the truth),I cannot operate on
him. He is my son. How can this be?

ASWER: The doctor is the boys


mother.
The above brain twister helps
develop critical reading skills. Which
activity was used?
a. Comparing
b. Classifying
c. Inferring meaning
d. Looking for cause and effect
81. Research says that mastery experiences
increase confidence and willingness to try
similar or more challenging tasks as
reading. What does this imply for
childrens reading performance?
a. Children who have not mastered the
basic skills are more likely to be
motivated to read in order to gain
mastery over basic skills.
b. Children who have mastered basic
skills are more likely to be less
motivated to read because they get
fed up with too much reading.
c. Children who have a high sense of
self-confidence are not necessarily
those who can read
d. Children who have gained mastery
over basic skills are more motivated
to read
82. The value that students put on reading is
critical to their success. In what way/s can
teachers inculcate his value for reading?
I. Sharing the excitement of read-aloud
II. Showing their passion for reading
III. Being rewarded to demonstrate the
value of reading
a. II and III
c. I, II and III
b. I and II
d. II only
83. Bruners theory on intellectual
development moves from enactive to
iconic and symbolic stages. Applying
Bruners theory. How would you teach?
a. Be interactive in approach
b. Begin with the abstract
c. Begin with the concrete
d. do direct instruction
84. A person who has painful experiences at
the dentists office may become fearful at
the mere sight of the dentists office
building. Which theory can explain this?
a. Generalization
b. Operant Conditioning
c. Attribution theory
d. Classical conditioning
85. Which is/are the basic assumption/s of
behaviorists?
I. The mind of newborn child is a blank
state.
II. all behaviors are determined by
environmental events
III. The child has a certain degree of
freedom not to allow himself to be
shaped by his environment.
a. III only
c. II only
b. I and II
d. I and II
86. If a student is encourage to develop
himself to the fullest and must satisfy his
hierarchy of needs, the highest needs to
satisfy according to Maslow is
.
a. psychological need c. belongingness
b. self-actualization
d. safety needs
87. In a Social studies class. Teacher I
presents a morally ambiguous situation
and asks student what they would do. On
whose theory is Teacher Is technique
based?
a. Bandura
c. Kohberg

St. Louis Review Center-Inc-Davao Tel. no. (082) 224-2515

110

b. Piaget
d. Bruner
88. Teacher F is convinced that whatever a
student performs a desired behavior,
provide reinforcement and soon the
student learns to perform the behavior on
his own. On which principle is Teacher Fs
conviction based?
a. Environmentalism
c. Cognitivism
b. Behaviorism
d.
Constructivism
89. Banduras social learning theory, states
that children often imitate those who
I.
have substantial influence over their
lives
II.
belong their peer group
III.
belong to other races
IV.
are successful and seem admired
a. IV only
c. I and II
b. I and IV
d. II and IV
90. According to Erikson, what years are
critical for the development of selfconfidence?
a. High school years
b. College years
c. Preschool years
d. Elementary school years
91. Which of the following does NOT describe
the development of children aged 11 to
13?
a. They exhibit increased objectivity in
thinking
b. They shift from impulsivity to adaptive
ability
c. Sex difference in IQ become more
evident
d. They show abstract thinking and
judgment

92. Teacher H begins a lesson on tumbling,


demonstrating front and back
somersaults in slow motion and physically
guiding his students through the correct
movements. As his students become
more skillful, he stands back from the
man and gives verbal feedback about
how to improve. With Vygotskys theory in
mind, what did Teacher H do?
a. Apprenticeship
b. Guided participation
c. Peer interaction
d. Scaffolding
93. What does Gagnes hierarchy theory
propose for effective instruction?
a. Be concerned with the socioemotional climate in the classroom
b. Teach beginning with the concrete
c. Sequence instruction
d. Reward good behavior
94. William Glassers control theory states
that behavior in inspired by what satisfies
a persons want at any given time. What
then must a teacher do to motivate
students to learn?
a. Make teaching-leaning interactive
b. Avoid giving assignments
c. Organize a curriculum in a spiral
manner
d. Make schoolwork relevant to students
basic human needs.
95. Soc exhibit fear response to freely
roaming dogs but does not show fear

when a dog is on a leash or confined to a


pen. Which conditioning process is
illustrated?
a. extinction
c. acquisition
b. generalization
d. discrimination
96. Based on Freuds theory, which operate/s
when a student strikes a classmates at
the height of anger?
a. Ego
c. Id and Ego interact
b. Id
d. Superego
97. Bernadette enjoyed the roller coaster
when he and her family went to
Enchanted Kingdom. The mere sight of a
roller coaster gets her excited. Which
theory explains Bernadettes behavior?
a. Operant conditioning
b. Social learning theory
c. Attribution theory
d. Pavlovian conditioning
98. According to Frued, with which should one
be concerned if he/she has to develop in
the students a correct sense of right and
wrong?
I.
Super-ego II. Ego
III. Id
a. I and II
c. I
b. II
d. III
99. When small children call animals dog,
what process is illustrated on Piagets
cognitive development theory?
a. reversion
c.
accommodation
b. assimilation
d. conservation
100. Researchers found that when a child is
engaged in a learning experience a
number of areas of the brain are
simultaneously activated. Which of the
following is/are implication/s of this
research finding?
I. Make use of field trips, guest speakers
II. Do multicultural units of study
III. Stick to the left brain and right brain
approach
a. I and III
c. I and II
b. I only
d. II only
101. Which appropriate teaching practice
flows from this research finding on the
brain: The brains emotional center is
tied into its ability to learn.
a. Establish the discipline of being
judgmental in attitude
b. Come up with highly competitive
games where winners will feel happy
c. Tell the students to participate in class
activities or else wont receive plus
points in class recitation
d. Create a learning environment that
encourages students to explore their
feeling and ideas freely
102. Research on Piagetian tasks indicates
that thinking becomes more logical and
abstract as children reach the formal
operations stage. What is an educational
implication of this finding?
a. Engage children in analogical
reasoning as early as preschool to
train them for higher order thinking
skills (HOTS)
b. Learners who are not capable of
logical reasoning from ages 8 to 11
lag behind in their cognitive
development
c. Let children be children

St. Louis Review Center-Inc-Davao Tel. no. (082) 224-2515

111

d. Expect hypothetical reasoning for


learners between 12 to 15 years of
age
103. Research says: People tend to
attribute their successes to internal
causes and their failures to external
causes.Based on this finding, what
should be taught to students for them to
be genuinely motivated to succeed?
a. Tell them the research finding when
applied will make them genuinely
motivated
b. Convince them that genuine
motivation is the only factor that
matters for a person to succeed
c. Make them realize that failure is a part
of life
d. Make them realize that both success
and failure are more a function of
internal causes.
104. Which characterize/s a learning
environment that promotes fairness
among learners of various cultures, family
background and gender?
I. Inclusive
II. Exclusive
III. Gender-sensitive
a. I only
c. I and III
b. III only
d. II and III
105. Which of the following steps should be
completed first in planning an
achievement test?
a. Define the instructional objective
b. Set up a table of specialization
c. Select the types of test items to use
d. Decide on the length of the test
106. The computed r fro scores in Math and
Science in 0.92. What does this mean?
a. Math score is positive related to
Science score
b. The higher the Math score, the lower
the Science score
c. Math score is not in any way related
to Science score
d. Science score is slightly related to
math score
107. Which types of test is most
appropriate if Teacher Y wants to measure
students ability to organize thoughts and
ideas?
a. Short answer type of test
b. Extended response essay
c. Modified alternative response
d. Limited response essay
108. With assessment of affective learning
in mind, which does NOT belong to the
group?
a. Cloze test
c. Reflective
writing
b. Moral dilemma
d. Diary entry
109. I want to test students synthesizing
skills. Which has the highest diagnostic
value?
a. multiple choice test c. Essay test
b. Performance test
d. Completion
test
110.
is an example of a leafy
vegetable.
I.
II.

Why is this test item poor?


The test item does not pose a problem
to the examinee
There are variety of possible correct
answer to this item

III.

the language used in the question is


not precise
IV. The blank is near the beginning of a
sentence
A. I and III
B. II and IV
C. I and IV
D. I and II
111.
What follows is a multiple choice type of
test.
Some test items
.
a. are too difficult
b. are objective
c. are poorly constructed
d. have multiple defensible answers
What makes the multiple choice type
of test poor?
a. The options are not grammatically
connected to the stem
b. The stem fails to present a problem
c. There are grammatical clues
d. The options are not parallel
112. If a teacher wants to measure her
students ability to discriminate, which of
these is an appropriate type of test item
as implied by the direction?
a. Outline the Chapter on The Cell.
b. Summarize the lesson yesterday.
c. Group the following items according
to shape.
d. State a set of principle that can
explain the following events.
113. A test item has a difficult index of
0.89 and a discrimination index of 0.44.
What should the teacher do?
a. Reject the item
b. Retain the item
c. Make it a bonus item
d. Make it a bonus item and reject it
114. Which form of assessment is
consistent with the saying The proof of
the pudding is in the eating.
a. Contrived
c. Traditional
b. Authentic
d. Indirect
115.
Who is best admired for outstanding
contribution to world peace?
a. Kissinger
c. Kennedy
b. Clinton
d. Mother Teresa
What is WRONG with this item?
a. Item is overly specific
b. Content is trivial
c. Test item is option-based
d. There is a cue to the right answer
116. Students score were as follows: 82,
83, 84, 86, 88, 84, 83, 85. The score 86 is
the.
a. mode
c. median
b. average
d. mean
117. Which text form would you choose if
you want to have a valid and reliable test
based on the table below?
Test Form
Validity Index
A
.47
B
.87
C
.20
D
.40
E
.63
a. A only
b. B only

St. Louis Review Center-Inc-Davao Tel. no. (082) 224-2515

c. A and D
d. B and E
112

118. A mathematicians test was given to


all Grade V pupils to determine the
contestants for the Math Quiz Bee. Which
statistical measure should be used to
identify the top 15?
a. Mean percentage score
b. Quartile Deviation
c. Percentile Rank
d. Percentage Score
119. Nellies score is within x1 SD. To
which of the following groups does she
belong?
a. Below average
b. Average
c. Needs Improvement
d. Above average
120. Use the inbox below to answer the
question that follows:
Percentage Grades for Final
Examination

40

100

70

80

90

Which of the following statement is


TRUE about the plot of grades above?
a. The median is a score of 80 and the
range is 60.
b. The median is a score of 70 and the
range is 60.
c. The median is a score of 80 and the
range is 20.
d. The median is a score of 70 and the
range is 20.
121. Which can be said of Arielle who
obtained a score of 75 out of 100 items in
a Grammar objective test?
a. She performed better than 25% of her
classmates
b. She answered 75 items in the test
correctly
c. Her rating is 75
d. She answered 75% of the test items
correctly
122. The criterion of success in Teacher Ds
objective is that the pupils must be able
to spell 90% of the words correctly. Ana
and 24 others in the class spelled only 40
out of 50 words correctly while the rest
scored 45 and above. This means that
Teacher D
a. attained her lesson objective
b. did not attain her lesson objective
because of the pupils lack of
attention
c. failed to attain her lesson objective as
far as the 25 pupils are concerned
d. attained her lesson objective because
of her effective spelling drill
123. If the scores of your test follow a
negatively skewed score distribution,
what should you do?
Find out
?
a. why your items were easy
b. why most of the scores are high
c. why most of the scores are low
d. why some pupils scored high
124. Principal A is talking about grading
on the curve in a faculty meeting. What
does this expression refer to?
a. A student mark compares his
achievement to his effort.

b. A students grade or mark depends on


how his achievement compares with
the achievement of other students in
a class.
c. A students grade determines whether
or not a student attains a defined
standard of achievement
d. A student mark tells how closely he is
achieving to his potential.
125. Which tests determine whether
students accept responsibility for their
own behavior or pass on responsibility for
their own behavior to other people?
a. Thematic tests
b. Sentence-completion tests
c. Stylistic test
d. Locus-of-control tests
126. Which of the one weakness of selfsupporting personality checklists?
a. Many personality measures have
built-in lie scales
b. They lack stability
c. They may not get true information
because individuals can hide or
disguise feelings
d. They have poor internal consistency
127. Which of these can measure
awareness of values?
a. Sociogram
b. Moral dilemmas
c. Projective techniques
d. Rating scales
128. Marking on a normative basis means
that
a. the normal distribution curve should
be followed
b. some should fall
c. some get high marks
d. the grading is based on a present
criteria
129. Which process enhances the
comparability of grades?
a. Using a table specifications
b. Determining the level of difficulty of
the tests
c. Giving more HOTS (higher order
thinking skills)
d. Constructing departmentalized
examinations for each subject area.
SITUATIONAL
Situation 1- In a faculty meeting, the
principle told his teacher: We need to
improve our school performance in the
National Achievement Test. What should we
do?
The teacher gave varied answers as
follows:
1. Lets give incentives and rewards to
students who get a rating of 85%
2. Lets teach them to accept
complete responsibility for their
performance
3. Lets make the school environment
conducive for learning
130. Which response/s come/s from a
behaviorist?
a. #2 and #4
c. #3 and #4
b. #1 and #2
d. #1 and #3
131. On which educational philosophy is
response #1 anchored?
a. Existentialism
c. Progressivism
b. Essentialism
d. Bahaviorism

St. Louis Review Center-Inc-Davao Tel. no. (082) 224-2515

113

132. If you learned toward a progressivist


philosophy, with which response would
you agree?
a. #2
b. #3
c. #4
d. #1
Situation 2- One principle in the utilization of
technology of the classroom is appropriateness
of material or activity.
133. Teacher C wants his students to
master the concept of social justice.
Which series of activities will be most
effective?
a. Pretest-teaching-posttest
b. Pretest-teaching-posttest-re-teaching
for unlearned concepts-posttest
c. Review-pretest-teaching-posttest
d. Teaching-posttest
134. Teacher A likes to show how the
launching of spaceships takes place.
Which of the following materials available
is most fit?
a. Model
c. Replica
b. Mock-up
d. Realia
135. Teacher B likes to concretize the
abstract concepts of an atom. She came
up with a concrete presentation of the
atom by using wire and plastic balls. How
would you classify Teacher Bs visual
aids?
a. Chart
c. Model
b. Replica
d. Realia
Situation 3- After reading and paraphrasing
Robert frosts Stopping by the Wood on a
snowy Evening. Mr. Sales asked the class to
share any insight derived from the poem.
136. The class was asked to share their
insights about the poem. The ability to
come up with a n insight stems from the
ability to
a. analyze the parts of a whole
b. evaluate the worthiness of a thing
c. relate and organize things and ideas
d. comprehend the subject that is being
studied
137. To ask the class any insight derived
from the poem is based on the theory of
a. realism
c. conditioning
b. behaviorism
d. constructivism
138. On which assumption about the
learner is Mr. Marquezs act of asking the
class to share their insight based?
a. Learners are like empty receptacles
waiting to be filled up
b. Learners are meant to interact with
one another
c. Learners have multiple intelligence
and varied learning styles
d. Learners are producers of knowledge
not only passive recipients of
information
Situation 4- Principal E wants her teachers
to apply constructivism in teaching

139. On which assumption/s is the


principals action anchored?
I.
Students learn by personally
constructing meaning of what is
taught.

II.

Students are construct and


reconstruct meaning based on
experiences
III.
Students derive meaning from the
meaning that the teacher gives
a. II only
c. I, II, and III
b. I and II
d. I only
140. Which materials will her teachers
LEAST prefers?
a. Controversial issues
b. Open-ended topics
c. Unquestionable laws
d. Problem or cases
141. Which concept/s of the learner will
Principal E NOT accept?
I.
Empty vesse!
II.
Tabula rasa
III.
Candle to be lighted
a. III only
c. II only
b. I only
d. I and II
Situation 5- Study the matching type of test then
answer the 3 questions that follow:
Column A
Column B
1. equilateral triangle
A. With 3 equal sides
2. right triangle
B. With 5 equal sides
3. octagon
C. Has 90- degree
angle
4. pentagon
D. Means many
5. heptagon
E. with 7 sides
6. poly
F. with 8 sides
142. How can you make the items
homogeneous?
a. Increase the number of items in
Column B
b. All items should be on polygons
c. Remove the word triangle in items #1
and #2 in column A
d. The word gon must be included in
column B
143. What is the main defect of this
matching test?
a. the matching type is an imperfect
type
b. the items are NOT homogeneous
c. the items quite easy
d. an obvious pattern is followed in the
answering
144. Which should be done to improve the
matching type of test?
a. Capitalize the items in Column A
b. Items in Column A and B should be
exchanged
c. Drop #6 item in Column A
d. The item in Column A should be
increased
Situation 6- Below the template for Scoring
Rubric.

5-Demonstrate complete understanding of the problem.


in response
4-Demonstrate considerable understanding of the pro
included
3- Demonstrate partial understanding of the problem. M
2- Demonstrate little understanding of the problem. Ma
1- Demonstrate no understanding
0-No response/task not att

St. Louis Review Center-Inc-Davao Tel. no. (082) 224-2515

114

145. Which of these is/are essential in


constructing a scoring rubric?
I.
Description of criteria to serve as
standard
II.
Clear descriptions of performance at
each level
III.
Levels of achievement (mastery)
IV.
Rating scheme
a. I, II, III c. I, II, III, IV
b. I, II
d. I only
146. Which statement is TRUE of the
rubric?
a. It is developmental
b. It is analytical
c. It is both holistic and developmental
d. It is holistic
147. Which is TRUE of the scoring rubric?
I.
It describes criteria of levels of
achievement
II.
It has a rating scheme
III.
It limit itself to 4 levels of
achievement
a. I and II
b. I and III
c. II and III
d. I, II and III
Situation 7- Study the table on item
analysis for non-attractiveness and nonplausibility of distracters based on the
results of a try-out test in Science. The
letter marked with a asterisk is the correct
answer.
Item No. 1
Upper 27%
Lower 27%

A
10
6

B
4
5

C
1
2

D
1
2

148. The table shows that the test item


analyzed
.
a. has a positive discrimination index
b. has a negative discrimination index
c. is extremely easy
d. is extremely difficult
149. Based on the table, which is the most
effective distracter?
a. Option D
b. Option A
c. Option C
d. Option B
150. Based on the table, which group got
more correct answer?
a. Upper group
b. It cannot be determined
c. Lower group
d. Data are not sufficient to give an
answer

When you get right down to the root of


the meaning of the word "succeed," you
find that it simply means to follow
through.

E
0
0

51

52

53

54

55

56

57

58

9
1
0
1
1
1
2
1
3
1
4
1
5
1
6
1
7
1
8
1
9
2
0
2
1
2
2
2
3
2
4
2
5
2
6
2
7
2
8
2
9
3
0
3
1
3
2
3
3
3
4
3
5

59

60

61

62

63

64

65

C
C

66
67

B
B

68

69

70

71

72

73

74

75

76

77

78

79

80

81

82

83

84

85

St. Louis Review Center-Inc-Davao Tel. no. (082) 224-2515

10
1
10
2
10
3
10
4
10
5
10
6
10
7
10
8
10
9
11
0
11
1
11
2
11
3
11
4
11
5
11
6
11
7
11
8
11
9
12
0
12
1
12
2
12
3
12
4
12
5
12
6
12
7
12
8
12
9
13
0
13
1
13
2
13
3
13
4
13
5

D
D
C
C
A
A
B
A
B
D
B
C
B
B
D
D
A
C
D
A
B
C
B
B
D
C
B
D
D
B
D
B
B
D
B
115

3
6
3
7
3
8
3
9
4
0
4
1
4
2
4
3
4
4
4
5
4
6
4
7
4
8
4
9
5
0

86

87

88

89

90

91

92

93

94

95

96

97

98

99
10
0

13
6
13
7
13
8
13
9
14
0
14
1
14
2
14
3
14
4
14
5
14
6
14
7
14
8
14
9
15
0

A
D
B
B
C
A
B
B
C
C
C
A
D
D
C

GENERAL SCIENCE
POST TEST
Multiple Choices:
1. These are living things that use sunlight,
chlorophyll, water and carbon dioxide to
produce food.
a. Autotrophs c. Heterotrops
b. Consumers d. Food chain
2. It is describe as the lifeline of the body. IT
is the bodys pickup and delivery
system.
a. Blood
c. circulatory system
b. nervous system
d. heart
3. It carries the oxygen-rich blood to the
head, arms, chest and down to the waist
and the legs.
a. heart
c. aorta
b. ventricles
d. arteries
4. They are the transmitters of message
from the different parts of the body to the
brain and vice versa.
a. spinal cord
c. brain
b. neurons or nerve cells
d.
arteries
5. Which part of the brain controls the
following activities: breathing, blood
pressure, heart rate, alertness.
a. brainstem
c. cerebrum
b. hypothalamus d. spinal cord
6. Refers to a sequence of organism in a
community that constitutes a feeding
chain.
a. photosynthesis
c. consumers
b. ecosystem
d. food chain

7. Is a group of interacting plants, animals


and human in a particular area.
a. ecological community
c. living
organism
b. environment
d. food
chain
8. Excessive presence of carbon dioxide in
the air, trapping heat near the earths
surface causing a rise in temperature in
the environment.
a. El Nino
c. Deforestation
b. Greenhouse Effect d. Weather
disturbance
9. The Earths shield against suns harmful
radiation.
a. Atmosphere
c. Ozone layer
b. Air
d. Forest
10. The use of product containing ______ is
discouraged because they contribute to
the depletion of_____.
a. Chlorofluorocarbon-solar radiation
b. Gas-ozone layer
c. Ozone layer air
d. Chlorofluorocarbon-ozone layer
11. What causes high and low tides?
a. Earths rotation on its axis
b. Moons gravitational pull
c. Suns solar energy
d. Earths gravitational pull
12. How is coral a tool formed?
a. Volcanic eruption
b. Corals growing around a volcanic
island
c. Underwater bedrock formations
d. Earthquake
13. What is a long shore drift?
a. Movement o sand and shingles along
the coast
b. Sand bars
c. Accumulation o sad at the river mouth
d. Island formed by volcanic eruptions
14. How does an occlusion form?
a. Cold air moving up from the ground
b. Cold front pushing warm air up of the
ground
c. Unbalance electrical reaction in the air
d. Cold and warm air mixing in the
atmosphere
15. What is a eat haze?
a. A reflection caused by pollutants in the
air
b. A distorted image resulting from the
bending o suns light rays by changes
in air temperature
c. A movement o warm air over a vast
expanse of land
d. Caused by extremely high temperature
common in dessert areas
16. What sort of rock formation do the worlds
greatest mountain ranges consist of?
a. Magma
c. Fold eruptions
b. Chalk deposit d. Slip formation
17. What is the fore that wears down
mountains?
a. Earthquake
c. Volcanic eruptions
b. Erosion
d. Deforestation
18. How are volcanic island formed?
a. Collision of two oceanic plates
b. Cooling of lava by seawater
c. Volcanic eruptions
d. Accumulation of corals
19. When the Theory of plate Tectonics was
generally accepted?
a. 1900s
c. 1950s
b. 1930s
d. 1980s

St. Louis Review Center-Inc-Davao Tel. no. (082) 224-2515

116

20. The weathering away of rocks by water,


wind and ice.
a. Denudation
c. Volcanic rock
b. Erosion
d. Metamorphic
rock
21. How do hormones work?
a. By releasing adrenaline
b. By controlling cell chemistry
c. By regulating water loss
d. By controlling blood pressure
22. Name the male and female sex hormones
a. Sperm cell & Ovum c. Chromosomes
b. Testosterone
d. Red & white
blood cells
23. The unit of measurement of energy in a
given an mount of food
a. Pound
c. Olfactory system
b. Kilo
d. calorie
24. Nitrogen compounds known as the
building blocks of proteins
25. The growth of roots towards water is an
example of?
a. Chemotropism
c. Hydrotropism
b. Geotropism
d. Phototropism
26. DNA means
a. Data nurturing analysis
c.
Deoxyribonucleic acid
b. Deoxytribonucleic acid
d.
Deotrixyl nucleic acid
27. What are the three products of oxygen
when it has been burned?
a. Water, carbon dioxide and air
b. Energy, water and carbon dioxide
c. Energy, carbon and oxide
d. Energy, air and water
28. In flowering plants, fertilization happens
in the?
a. Pollen tube
c. Ovules
b. Stamen
d. Pollen grain
29. The development of egg without
fertilization
a. Mitosis
c. Spermatogenesis
b. Parthenogenesis d. Mitochondria
30. Which of the following is a source of
energy needed for photosynthesis?
a. Water
c. Light
b. Soil
d. Fertilizer
31. Chemistry is primarily concerned with the
composition and changes of?
a. Nature
c. Man
b. Matter
d. Earth
32. A scientific theory is
a. A hypothesis not yet subjected to
experimental test
b. An idea that correctly predict the result
c. An imagination
d. A guess
33. Which of the following units of measure is
equivalent to cubic centimeter?
a. Milligram
c. Millimeter
b. Milliliter
d.
Centiliter
34. Which of the following is NOT a
compound?
a. acetic acid
c. magnesium
b. alcohol
d. Zinc Oxide
35. The easier the atom to receive electrons
is measured by its?
a. Elecrtonegativity
c. Number of
shells
b. Atomic radius
d. Valence
electrons
36. The willingness o an atom to receive
electron is measured by its?
a. Electronegativity
c. Atomic
size

b. Ionization potential
d. Electron
affinity
37. A molecule is said to be polar or dipole if?
a. Its positive and negative charges are
at different places
b. It possesses polar bonds
c. Its polar bond have unsymmetrical
charge distribution
d. All of the above
38. Which of the following is NOT a physical
property of water?
a. freezing point at 0 degree C
b. boiling point at 100 degrees C
c. its heat of fusion at 80 cal/g.
d. its density at 4 degrees at 1 lb/cu. Ft
39. Which of the following statements is
true?
a. Molecular weight does not influence
boiling and melting point of a
substance
b. Boiling and melting point tend to
increase with molecular weight
c. Boiling and melting point tend to
decrease with molecular weight
d. None o the above
40. A Mole is the amount of substance or a
mass of a substance that contains?
a. 6.02 x 1023 particles c. 6.02 x 1023
particles
b. 60.2 x 1023 particles d. 60.2x 1023
particles
41. It is the measure of the amount of matter
in an object
a. Weight
c. Volume
b. Mass
d. Quantity
42. It is the distance traveled by the body per
unit time and tell how fast or slow the
body moves
a. Velocity
c. Acceleration
b. Speed
d. None of the above
43. The rate of change of the distance
traveled per unit time in a stated
direction
a. Velocity
c. Acceleration
b. Speed
d. None of the above
44. This law states that the force acting upon
an object is equal to the product o the
mass and acceleration of the object
a. Newtons 2nd law of motion
b. Newtons 3rd law of motion
c. Newtons 1st law of motion
d. None of the amount

45. When a force is applied to a body, several


effects are possible. Which one of the
following effect CANT occur?
a. the body rotates
b. the body changes direction
c. the body increase its mass
d. the body changes shape
46. It is the reluctance of the object to
change either its state of rest or uniform
motion in a straight line
a. Force
c. Inertia
b. Friction
d. Motion
47. This law states that energy cannot be
created nor destroyed but only changes
from one form to another
a. Energy law
b. Kinetic Theory of Matter
c. Law of Conservation Energy
d. None of the above

St. Louis Review Center-Inc-Davao Tel. no. (082) 224-2515

117

48. This law states that matter is made up of


a large number of molecules which are in
continuous motion
a. Boyless Law c. Law of Conservation
Energy
b. Kinetic Theory d. None of the above
49. The lowest possible temperature that a
substance can reach
a. Freezing point
c. Steam point
b. Absolute Zero
d. Threshold
50. It is the amount of heat required to raise
the temperature of kg. of a substance by
degree C
a. Calorie
c. Specific heat capacity
b. Watt
d. Joule
51. The relationship of give-and-take of living
organism in the biosphere is a balance of
nature called________.
a. universal relationship
b. symbiotic relationship
c. spontaneous relationship
d. abiogenetic relationship
52. Process of removing excess odor in water.
a. sedimentation
c. distillation
b. chlorination
d. aeration
53. Which of the following statement is
CORRECT?
a. As altitude increases, atmospheric
pressures corresponding
b. Throughout the available space, gas
tends to contract
c. Equal chances are always given to all
in life
d. Shadow is formed when a colored
object is projected against the wall
54. The earth rotates on its axis from west to
east. This causes the sun to _______?
a. appear with a fiery orange color
b. cause the appearance of solar eclipse
c. rise room the east and sets in the west
d. emit solar radiation
55. One of these planets has the greatest
gravitational pull. Which one is it?
a. Mars
b. Earth c. Mercury d.
Jupiter
56. It is the law which explains why one can
pull a piece without topping a glass in a
quick motion.
a. energy in motion
c. law of inertia
b. gravity
d. force
57. Which of the following is NOT a source of
energy?
a. water
c. geothermal heat
b. nuclear
d. inertia at rest
58. Which instrument will one use to convert
mechanical energy to electrical energy?
a. rotor
b. generator c. motor d.
circuit

59. Suns energy is generated by?


a. nuclear fission
b. sun enacting with gravity
c. nuclear fusion reaction
d. reaction with magnetic field
60. Application of energy is called _______?
a. work
c. kinetics
b. inertia
d. simulation
61. The falling of any form of water from the
air to the earths surface
a. Condensation c. Water vapor
b. Precipitation d. Rainwater
62. The part of the atmosphere that filters
the ultraviolet rays of the sun

a. stratosphere c. Ozone layer


b. Troposphere d. Ionosphere
63. The point in the earths orbit nearest to
the sun
a. Solstice
c. Aphelion
b. Eclipse
d. Perihelion
64. A property of minerals which gives off
rays of light when exposed to ultraviolet
light.
a. Luminescence c. Radiation
b. Phosphorescence d. Fluorescence
65. Which process involves chemical
weathering?
a. Carbonation c. Hydration
b. Oxidation
d. All of the
above
66. How long does it take for the earth to
complete one rotation
a. 365 days
c. 24 hours
b. 30 days
d. 12 hours
67. What is the principal function of gravity in
the universe?
a. Provision for energy
b. Keeps the stars and other heavenly
bodies in orbit
c. Causes movement in space
d. Part of universal design
68. What does the word monsoon mean?
a. Moon will soon come
c.
Seasons
b. Rains
d. Wet
weather
69. Its discovery enable geologist to date
rocks accurately
a. layering
c.
Radioactivity
b. Evolutionary staging
d.
Carbon-dating
70. It occurs when the earth is between the
sun and the moon, with the earths
shadow cast over the moon.
a. Total eclipse
c. Eclipse
b. Lunar eclipse
d. Partial Eclipse
71. Male and female reproductive part of a
flower
a. Pollen grains and ovules c. Pollen
grains and pistil
b. Stamen and pistil
d. Stamen and
ovules
72. In the human body, the cell that most
nearly resembles a one-celled animals
a. Red Blood cell
c. Nerve cell
b. White Blood cell
d.
Antibodies
73. The main energy source of a plant-eating
animals
a. Glucose
b. Starch c. Cellulose
d. Glycogen
74. These are cellular secretions which help
regulate the breakdown and buildup of
various substance in the body
a. Enzymes
c. Plasma
b. Amino Acids
d. Hormones
75. It is the energy source of the cell which it
uses for growing, reproducing and other
activities
a. Adenosine Triphosphate
c.
Chloroplast
b. Amino Acids
d.
Sunlight
76. It is a segment of DNA molecule which
controls the appearance of a given trait
a. Chromosomes
c.
Gametes
b. Genes
d.
Zygotes

St. Louis Review Center-Inc-Davao Tel. no. (082) 224-2515

118

77. Group of similar cells performing similar


functions together
a. Organs b. System c. Nucleus
d.
Tissue
78. The diffusion of water through a semi
permeable membrane
a. osmosis
c.
Transfusion
b. permeability
d.
Capillary
79. It shows the complex food relationship of
organism in a given area and the cyclic
flow of food through organisms
a. Food chain
c. Food pyramid
b. Food web
d. Biological
cycle
80. Which of the following does NOT occur to
both respiration and fermentation?
a. energy is released
b. sugar is broken down
c. carbon dioxide is produced
d. alcohol is formed
81. Energy removal is best illustrated in
a. boiling of liquid substances
b. changing water to ice
c. changing water to stem
d. none of the above
82. Refers to the maximum amount of solute
expressed in grams that can be dissolved
in 100 grams of water at a specific
temperature
a. Solubility
c. Molarity
b. Stability
d.
Molality
83. Compounds with the same molecular
formula but with different structural
formulas
a. Cellulose
c. Polymers
b. Isomers
d. Monomers
84. The most penetrating type of radiation
given of by radioactive elements
a. Alpha particle c. Gamma particle
b. Beta particle d. None of the above
85. The basic unit for expressing the masses
o individual atoms
a. Atomic number
c.
Nucleus
b. Atomic mass unit
d. Atomic
weight
86. A substance that speeds up a chemical
reaction without itself undergoing a
chemical change
a. catalyst
c.
Electrolytes
b. Enhancer
d. Ionizer
87. The temperature at which the vapor
pressure of the liquid is equal to the
pressure of the surroundings atmosphere
a. Melting point c. Boiling Point
b. Critical point d. None of the above
88. the warming of the earths surface due to
an increase in atmospheric carbon
dioxide
a. Greenhouse effect c. Atmospheric
pressure
b. Ozone
d. El Nino
phenomena
89. Describe the force of gravity on an object
a. mass
c. capacity
b. weight
d. pressure
90. When gaseous molecules are
compressed, they tend to?
a. increase in volume c. repel each other
b. decrease in volume d.attract and
liquefy

91. It is the union of two light nucleus to form


a heavier nucleus, resulting in a mass
defect and release of energy
a. Radiation
c. Nuclear Fusion
b. Nuclear Fission
d.
Radioactivity
92. Which of the following statements is a
characteristic of an electronic spectrum?
a. They all travel at the same speed in
fire space
b. The exhibit diffraction and interference
phenomena
c. They follow the laws of refraction and
reflection
d. All of the above
93. The term RADAR is derived from the
phrase?
a. Radiation Detection and Ranging
b. Radiation Diffusion and Ranging
c. Radio Diffraction and Resolution
d. Radiation Diffraction and Resolution
94. A material whose ability to conduct
electricity lies between those of
conductors and insulators
a. Integrated Circuits
c.
Semiconductors
b. Silicon Chips
d. Insulators
95. LASER is derived from the phrase?
a. Light Amplification by Stimulated
Emission of Radiation
b. Light Application by Simulated
Emission of Radiation
c. Light Amplification by Simulated
Ejection of Radiation
d. None of the above
96. What is the color of a transparent
substance?
a. The color of the light it absorbs
b. The color of light it reflects
c. The color of light it transmit
d. The color of light it refracts
97. What is a rotating electromagnetic called?
a. Motor b. Rotor
c. Phasor d.
Sensor
98. What happens with the centripetal force
when sped is doubled?
a. remains the same
c. triples
b. force is increase 4x d. force is
doubled
99. What is an electrochemical cell in which
the reacting materials can be renewed by
the use o reverse current
a. Storage cell
c. Fuel cell
b. Primary cells
d. Chemical cell
100. What will make an object move in a
circular path?
a. Central force
c. Frictional force
b. Gravitational
d.
Electromagnetic force
***** THE END *****
WORK HARD, DREAM HARDER

St. Louis Review Center-Inc-Davao Tel. no. (082) 224-2515

119

science and health


1
a
51
2
c
52
3
d
53
4
b
54
5
a
55
6
d
56
7
a
57
8
b
58
9
c
59
10
d
60
11
b
61
12
b
62
13
a
63
14
b
64
15
b
65
16
c
66
17
b
67
18
a
68
19
c
69
20
b
70
21
b
71
22
b
72
23
d
73
24
d
74
25
c
75
26
c
76
27
b
77
28
c
78
29
b
79
30
c
80
31
b
81
32
a
82
33
b
83
34
c
84
35
d
85
36
d
86
37
b
87
38
b
88
39
b
89
40
c
90
41
b
91
42
b
92
43
c
93
44
a
94
45
c
95
46
d
96
47
b
97
48
b
98
49
b
99
50
c
100

b
d
a
c
d
c
d
b
c
a
b
c
d
b
d
c
b
c
c
b
b
b
c
d
a
b
d
a
b
d
b
a
b
c
b
a
c
a
b
a
c
d
a
c
a
c
b
d
a
a

St. Louis Review Center-Inc-Davao Tel. no. (082) 224-2515

120

You might also like